Questions and Answers about Allopathic Medical Schools

This forum made possible through the generous support of SDN members, donors, and sponsors. Thank you.
Status
Not open for further replies.

imrep1972

Senior Member
15+ Year Member
Joined
Jan 3, 2005
Messages
388
Reaction score
1
We need volunteers!!
We would like one or more people from each medical school to take a few moments and answer as many of AAMC's 31 questions as they can. When we all do this, we will have a tremendous resource for every student applying or thinking of applying to medical schools.

This is how this will work:

1. Examine the thread to see if your school has been added. If not, email the answers to as many of the questions as you'd like to [email protected] from your SCHOOL email account.

2. If your school is already listed, please complete whatever questions have not been answered by other students. Email your answers to [email protected] from your SCHOOL email account.

3. A moderator will post your answers to the thread.

Now let's see that school pride and help out the SDN members who are thinking of applying!


Can I participate in this thread anonymously?
Not completely. This IS a verified identity thread, after all. You will have to disclose your name and school affiliation to the Mentor Forum staff. But, if you'd still like to help out and protect your identity from general posters on this forum, we can accomodate that.

Email your answers to [email protected] and make sure you tell us NOT to reveal your name when we post your answers. Tell us how you'd like to be credited: by first name only, by initials, by a nickname, or as "anonymous student."

Is there a downloadable form of the 31 Questions?
Sure. It is attached.

Members don't see this ad.
 

Attachments

  • SDN 31 Questions Allopathic.doc
    28 KB · Views: 856
Table of Contents

Albany Medical College
Ben Gurion Medical School for International Health
Cleveland Clinic Lerner College of Medicine of Case Western Reserve University
Columbia University, College of Physicians and Surgeons
Duke University
Georgetown University
George Washinton University
Jefferson Medical College
Medical College of Virginia
Medical College of Wisconsin
New York Medical College
Robert Wood Johnson - Camden
Uniformed Services University of the Health Sciences
University of Iowa
University of Kansas
University of Maryland
University of Miami
University of Miami, Boca
University of New Mexico
University of North Carolina
University of Oklahoma
University of Pennsylvania
University of Southern California
University of Texas, Houston
University of Toledo
University of Wisconsin
Weill Cornell Medical College


Standard Disclaimer
Remember, everything posted on any SDN forum, once posted becomes permanently available to the general public. Members are encouraged to avoid posting anything that they would not want to be associated with publicly. Use caution in sharing identifiable information or overly personal details.

Mentors and students posting in this thread are individual volunteers. Advice given by mentors and/or students does not necessarily represent the views or opinions of the Student Doctor Network or CRG. The mentor forum should not be used as your only source of advice when making career or academic decisions.
 
Curriculum
1. Are there any special programs for which this medical school is noted?
2. Describe this school's curriculum in the pre-clinical and clinical years. Are there any innovations, like Problem-Based Learning?
3. Are there opportunities for students to design, conduct, and publish their own research?
4. Is there a note-taking service? If so, is it university-run or student-run?
5. Is there flexibility in the coursework (the number of electives) and the timing of the courses (accelerating, decelerating, and time off) during the pre-clinical and clinical years?
6. Has this medical school, or any of its clinical departments, been on probation or had its accreditation revoked?
7. How do students from this medical school perform on the National Board Examinations? How does the school assist students who do not pass?

Evaluations
8. How are students evaluated academically? How are clinical evaluations performed?
9. Is there a formal mechanism in place for students to evaluate their professors and attending physicians? What changes have been made recently as a result of this feedback?

Counseling/Student Support
10. What kind of academic, personal, financial, and career counseling is available to students? Are these services also offered to their spouses and dependents/children?
11. Is there a mentor/advisor system? Who are the advisors—faculty members, other students, or both?
12. How diverse is the student body? Are there support services or organizations for ethnic minorities and women?

Facilities
13. Tell me about the library and extracurricular facilities (i.e., housing and athletic/recreational facilities).
14. Are there computer facilities available to students? Are they integrated into the curriculum/learning?
15. What type of clinical sites—ambulatory, private preceptors, private hospitals, rural settings—are available or required for clerkships? Does this school allow for students to do rotations at other institutions or internationally?
16. Is a car necessary for clinical rotations? Is parking a problem?

Financial Aid
17. What is the current tuition and fees? Is this expected to increase yearly? If so, at what rate?
18. Are there stable levels of federal financial aid and substantial amounts of university/medical school endowment aid available to students?
19. Are there students who have an "unmet need" factor in their budget? If so, where do these students come up with the extra funds?
20. Are spouses and dependents/children covered in a student's budget?
21. Is someone available to assist students with budgeting and financial planning?
22. Does this school provide guidance to its students, and to its graduates/alumni, on debt management?

Student Involvement
23. What medical school committees (e.g., curriculum committee) have student representation?
24. Are students involved in (required or voluntary) community service?
25. How active is the Student Council/Government? Are there other student organizations?
Policies
26. Is there an established protocol for dealing with student exposure to infectious diseases? Is disability insurance provided to cover this exposure?
27. Does this school provide, or does the student pay for, vaccinations against Hepatitis B or prophylactic AZT treatment in case of a needle-stick or accident?
28. Is there a school Honor Code? Is there a grievance process/procedure? Are the students involved?
Residency
29. May I see a list of residency programs to which this school's recent graduates were accepted?

Questions to Ask Yourself
30. Does this school have strengths in the type of medicine (primary versus specialized care, urban versus rural practice environment, academic medicine versus private practice) that I will want to practice?
31. Would I be happy at this school (for at least the next four years)?
 
Members don't see this ad :)
Curriculum
1. Are there any special programs for which this medical school is noted?
For the basic science years, Maryland students are only in lecture for two hours a day, and in small group (for the most part) for two hours a day. This is a departure from a lot of school's lecture system.

2. Describe this school's curriculum in the pre-clinical and clinical years. Are there any innovations, like Problem-Based Learning?
The pre-clinical were discussed above, but the small groups can be many things including PBL, clinical information workshops, laboratories and more. Clinical years are typical.

3. Are there opportunities for students to design, conduct, and publish their own research?
Absolutely.

4. Is there a note-taking service? If so, is it university-run or student-run?
There is no note-taking service, but all lectures are recorded on video and mp3 audio and made available for download. The lecturer's Powerpoint file and Noteset are also posted.

5. Is there flexibility in the coursework (the number of electives) and the timing of the courses (accelerating, decelerating, and time off) during the pre-clinical and clinical years?
Not particularly. A couple of electives are offered that can give credit for fourth year, but that's about it.

6. Has this medical school, or any of its clinical departments, been on probation or had its accreditation revoked?
Not that I am aware of.

7. How do students from this medical school perform on the National Board Examinations? How does the school assist students who do not pass?
About average. A mid-90s pass rate, I believe. Students who do not pass are given time away from third year rotations to study for a second go.

Evaluations
8. How are students evaluated academically? How are clinical evaluations performed?
Honors-A-B-C scale. Anything below a C is a fail. Honors are done by class, but are generally given to those in the top 10%, sometimes with a project required.

9. Is there a formal mechanism in place for students to evaluate their professors and attending physicians? What changes have been made recently as a result of this feedback?
Evaluations are done online and anonymously, and are required after each exam to receive those scores. Numerous professors (including some section heads!) have been replaced after student feedback.


Counseling/Student Support
10. What kind of academic, personal, financial, and career counseling is available to students? Are these services also offered to their spouses and dependents/children?
Tutoring and academic counseling is available.

11. Is there a mentor/advisor system? Who are the advisors—faculty members, other students, or both?
We have a mentor system in place that students can register for with a faculty member in their specialty of choice.

12. How diverse is the student body? Are there support services or organizations for ethnic minorities and women?
We have a very diverse student body. Probably one of the most diverse classes in the country. Minority and women's organizations exist.

Facilities
13. Tell me about the library and extracurricular facilities (i.e., housing and athletic/recreational facilities).
The library is excellent, is fully stocked with anything you might need. It is not open 24 hours, but the Pods, a set of multidisciplinary labs are in the main medical school and are open 24 hours (with card access). The student center is poor, but a new one is being constructed. The gym is outfitted with all of the necessary equipment, including racquetball courts, but can get crowded and is poorly ventilated. Some equipment is also quite old. There are two housing options on campus - Pascault Row and University Suites. The former is quite old and cramped, the latter are nice, but very expensive compared to apartments and houses in the area.

14. Are there computer facilities available to students? Are they integrated into the curriculum/learning?
All students are given laptops during orientation. All exams except for wet practicals are taken on them. They are absolutely crucial to the curriculum. There are also separate computer labs on campus.

15. What type of clinical sites—ambulatory, private preceptors, private hospitals, rural settings—are available or required for clerkships? Does this school allow for students to do rotations at other institutions or internationally?
All types of sites - a rural setting is usually done in the 4th year AHEC rotation. Away rotations are allowed, as far as I know.

16. Is a car necessary for clinical rotations? Is parking a problem?
From what I understand, cars are essential for rotations, but parking is not an issue.

Financial Aid
17. What is the current tuition and fees? Is this expected to increase yearly? If so, at what rate?
For the 2007/2008 year: 21,702 for in state, 39,957 for out of state.

18. Are there stable levels of federal financial aid and substantial amounts of university/medical school endowment aid available to students?
Yes.

19. Are there students who have an "unmet need" factor in their budget? If so, where do these students come up with the extra funds?

20. Are spouses and dependents/children covered in a student's budget?

21. Is someone available to assist students with budgeting and financial planning?


22. Does this school provide guidance to its students, and to its graduates/alumni, on debt management?
Yes.

Student Involvement
23. What medical school committees (e.g., curriculum committee) have student representation?
Admissions committee, curriculum committee.

24. Are students involved in (required or voluntary) community service?
Yes. We were required to do two separate community service events this year, but this may change, as it was new.

25. How active is the Student Council/Government? Are there other student organizations?
Student organizations are very active on campus.

Policies
26. Is there an established protocol for dealing with student exposure to infectious diseases? Is disability insurance provided to cover this exposure?

27. Does this school provide, or does the student pay for, vaccinations against Hepatitis B or prophylactic AZT treatment in case of a needle-stick or accident?
This was a recent issue, and it was not provided in the past but should be provided for now.

28. Is there a school Honor Code? Is there a grievance process/procedure? Are the students involved?
Yes, there is an honor code, but I don't know about the process.

Residency
29. May I see a list of residency programs to which this school's recent graduates were accepted?
The last 4 years match lists are here.

Questions to Ask Yourself
30. Does this school have strengths in the type of medicine (primary versus specialized care, urban versus rural practice environment, academic medicine versus private practice) that I will want to practice?
We are in a very urban environment, so this is a key part of the clinical curriculum.

31. Would I be happy at this school (for at least the next four years)?
 
Curriculum
1. Are there any special programs for which this medical school is noted?
I don't really know of any, although the school has a fantastic loan repayment program if one is going into primary care in underserved Kansas. Its called KMSL, and it pays for tuition in its entirety (for in state or out of state students) plus gives a generous stipend. The commitment is four years in primary care (defined as general IM, FM, Peds, EM) in underserved Kansas, which is all but four counties.
2. Describe this school's curriculum in the pre-clinical and clinical years. Are there any innovations, like Problem-Based Learning?
We've just recently began a new curriculum (my class was the first class to be a part of it) Its a system's based approach in which we learn all of the relevant information (pathology, physiology, histology, biochemistry, pharmacology, etc) based on systems rather than taking the traditional courses that cover these topics individually. We have some PBL, small groups, and other group like activities worked into our curriculum. We also have some clinical experience in our first year. We have basically learned how to do the entire history and physical in our first year (other than Ear, eyes, nose, throat.) We also do monthly preceptor visits with a physician as a part of our curriculum.
3. Are there opportunities for students to design, conduct, and publish their own research?
Yes, we're sent emails quite often detailing research opportunities, although I haven't taken advantage of any of them so I don't know more information.
4. Is there a note-taking service? If so, is it university-run or student-run?
There was prior to the change in curriculum, but isn't one currently. We are given the power points and are given podcasts of the lectures. I think this is sufficient and so a note taking service is not necessary.
5. Is there flexibility in the coursework (the number of electives) and the timing of the courses (accelerating, decelerating, and time off) during the pre-clinical and clinical years?
I'm not aware of details. I'm pretty sure people take years off for research or personal reasons but I don't know anything more about it.
6. Has this medical school, or any of its clinical departments, been on probation or had its accreditation revoked?
not that I'm aware of
7. How do students from this medical school perform on the National Board Examinations? How does the school assist students who do not pass?
I think we were around the national average last year. I don't know what they do for students that don't pass

Evaluations
8. How are students evaluated academically? How are clinical evaluations performed?
We have a test over every block (usually 4 week long blocks) and are allowed to retake the final if we desire. (its a different test but similar) We have the standard Honor, High Sat, Sat, low sat, Unsat grading levels. (I'm not sure why they don't just call it A,B,C,D,F)
9. Is there a formal mechanism in place for students to evaluate their professors and attending physicians? What changes have been made recently as a result of this feedback?
Yes, with the new curriculum they are constantly trying to get our feedback and large changes have been made in response to that.

Counseling/Student Support
10. What kind of academic, personal, financial, and career counseling is available to students? Are these services also offered to their spouses and dependents/children?
We have a student center that has counseling in all of the above areas. I haven't used it yet, so I can't really comment on it.
11. Is there a mentor/advisor system? Who are the advisors—faculty members, other students, or both?
Yes, we have multiple mentor systems set up. Big Brother/Big Sister system with the second year students, the preceptors we work with monthly, a mentor set up through our society, and a mentor set up through the school, oh, I almost forgot, our PBL instructor is a physician who has been a valuable resource to our group this year. I haven't really been impressed with the mentors set up for me (they haven't tried to keep in contact with me, in fact I haven't even met them). I think this is a combination of my bad luck and the fact that I haven't really tried either. I think its a situation where if I had put in more effort, I would have found it more rewarding. I have really enjoyed getting to know my preceptor and have also really enjoyed the time we've spent with our PBL instructor. All of the mentors are both physicians and faculty members. (other than the second year students I mentioned)
12. How diverse is the student body? Are there support services or organizations for ethnic minorities and women?
I guess we're ethnically diverse. I'm sure there are some support services available but I'm not aware of them.

Facilities
13. Tell me about the library and extracurricular facilities (i.e., housing and athletic/recreational facilities).
The library contains our computer based testing center. Thats about all the experience I have with it. We have a "gym" with a basketball court, pool, hot tub, sauna, indoor track, and various machines. They also have trainers for various workout routines. Its maybe a little small and a little old, but it works for those that use it.
14. Are there computer facilities available to students? Are they integrated into the curriculum/learning?
We have computers available throughout the school buildings and library, but my class doesn't use them much. We are required to purchase a notebook computer through the school during orientation. We have a lot of stuff available to us through this computer and use it almost completely (if we choose) as a part of our education process. (We have many of our books saved on the computer and also take notes on it during class)
15. What type of clinical sites—ambulatory, private preceptors, private hospitals, rural settings—are available or required for clerkships? Does this school allow for students to do rotations at other institutions or internationally?
I know we have a requirement to do a rural clerkship. I don't know much more. 1/3 of the class does the clinical years at an off campus site (Wichita, KS). I've heard this is actually quite a great experience and would do it myself except for the fact that I have children and we own a home in KC.
16. Is a car necessary for clinical rotations? Is parking a problem?
I think its probably necessary. Parking isn't terrible, but it just depends on what your comparison is. I thought it was bad compared to what I was used to prior to med school, but have been told its quite good compared to large state universities.

Financial Aid
17. What is the current tuition and fees? Is this expected to increase yearly? If so, at what rate?
In state tuition is something like $19K. OOS tuition was $36K last year. I'm sure it will continue to increase but have no idea at what rate.
18. Are there stable levels of federal financial aid and substantial amounts of university/medical school endowment aid available to students?
We have the normal allotments for stafford loans. I'm not aware of a large amount of scholarships that are given away each year, but it could be that I'm just not that special. They do have a neat program for individuals who want to work in underserved primary care. They pay for tuition and give a generous stipend every month for a four year commitment to Kansas underserved primary care (includes FM, general IM, EM, and Peds.) Initially I was a part of this program. The program gave me $55K my first year. I got out of it before the end of the year due to an interest in surgery.
19. Are there students who have an "unmet need" factor in their budget? If so, where do these students come up with the extra funds?
I don't have a clue
20. Are spouses and dependents/children covered in a student's budget?
just dependant daycare needs. I think our budget was generous, but wouldn't have been able to support my husband and children if my husband didn't work.
21. Is someone available to assist students with budgeting and financial planning?
yes, we have a helpful financial aid department and they often have seminars about such things
22. Does this school provide guidance to its students, and to its graduates/alumni, on debt management?
seminars are available, I don't know what else they do.

Student Involvement
23. What medical school committees (e.g., curriculum committee) have student representation?
I know there are lots of committees.... don't have more knowledge
24. Are students involved in (required or voluntary) community service?
yes, we do a community service project as a part of our orientation. there is also a community clinic that our students are very involved in. everything is voluntary as far as I am aware of.
25. How active is the Student Council/Government? Are there other student organizations?
don't know.
Policies
26. Is there an established protocol for dealing with student exposure to infectious diseases? Is disability insurance provided to cover this exposure?
not sure
27. Does this school provide, or does the student pay for, vaccinations against Hepatitis B or prophylactic AZT treatment in case of a needle-stick or accident?
i'm not sure
28. Is there a school Honor Code? Is there a grievance process/procedure? Are the students involved?
yes there is, i'm not sure of much more. I think students run the committee that reviews grievances.

Residency
29. May I see a list of residency programs to which this school's recent graduates were accepted?
http://www.kumc.edu/som/ResidencySpecialtyLocation2007Graduates.html

Questions to Ask Yourself
30. Does this school have strengths in the type of medicine (primary versus specialized care, urban versus rural practice environment, academic medicine versus private practice) that I will want to practice?
I think that one could really go in any direction with an education at KUMed.
31. Would I be happy at this school (for at least the next four years)?
I certainly am happy with my choice to attend there.
 
Curriculum
1. Are there any special programs for which this medical school is noted?
Duke has a unique curriculum - the first year (11 months) is preclinical/basic science coursework. The second year (13 months) is made up of clinical rotations. The third year is devoted to research or the pursuit of a dual degree. The research can be in any field that is related to medicine (basic science, clinical, public health, etc). The fourth year is similar to other medical schools, and is mostly elective rotations with a few required rotations (Sub-I).
2. Describe this school's curriculum in the pre-clinical and clinical years. Are there any innovations, like Problem-Based Learning?
The preclinical years are divided into 4 blocks - Molecules and Cells (6 weeks - genetics, biochemistry, cell biology and histology), Normal Body (10 weeks - anatomy, physiology and microanatomy), Brain and Behavior (4 weeks - neuroscience/neuroanatomy and psych) and Body and Disease (20 weeks - immunology, microbiology, pathology and pharmacology). The majority of work is lecture-based with labs and some small group activities. There are some clinical presentations made by doctors to tie the basic science work with medicine. Sometimes patients accompany the doctors to talk about their disease. There is no problem based learning, but we do have CPCs (Clinical Pathological Correlates) during Body and Disease that are case studies (done with the whole class, not in small groups).
Second year has 6 mandatory rotations - 4 weeks of Family Med, 4 weeks of Psych, 8 weeks of Internal Med, 8 weeks of Surgery, 6 weeks of Ob/Gyn and 6 weeks of Peds. There are also two 2 weeks electives and one 4 week elective. The electives are offered in all areas of medicine and students have the option of doing out of the country electives during the 4 week elective block. Fourth year is mostly electives, with a required Sub-I and a required Critical Care rotation (EM, ICU, CCU). There is also a "capstone course" at the end that is required.
3. Are there opportunities for students to design, conduct, and publish their own research?
All of third year is devoted to research. Students do research (or pursue a dual degree) for 10-12 months in the area of their interest. Some scholarships and grants are available for this time.
4. Is there a note-taking service? If so, is it university-run or student-run?
There is no official note taking service. However, the first year class often puts together a "notes group" in which each student takes notes for one lecture every month or so, on a rotating basis. The student taking notes for the lecture is supposed to re-stream the lecture to ensure that they get all the pertinent information. This has worked really well in the past.
5. Is there flexibility in the coursework (the number of electives) and the timing of the courses (accelerating, decelerating, and time off) during the pre-clinical and clinical years?
6. Has this medical school, or any of its clinical departments, been on probation or had its accreditation revoked?
The family medicine residency program closed it's doors last year, but is re-opening again with new ideas about the training program they would like to use. The program was NOT under probation and did NOT have it's accreditation revoked, it was just felt to be not working correctly.
7. How do students from this medical school perform on the National Board Examinations? How does the school assist students who do not pass?
The average Step 1 score is around 235. Step I is generally taken at some point during third year.

Evaluations
8. How are students evaluated academically? How are clinical evaluations performed?
Preclinical courses are graded H/P/F, with the first block (Molecules and Cells) being graded strictly P/F. Passing is considered a 70. In Normal Body, any exam grade under a 60 MUST be retaken. In Body and Disease, the only grade that counts is your cumulative test average. Lab tests must be passed in order to pass the block. Honors is generally considered a 90 or above. Clinical rotations are graded H/HP/P/F. The grading system for each rotation is different, with the shelf exam counting for a different percentage of the grade depending upon the rotation.
9. Is there a formal mechanism in place for students to evaluate their professors and attending physicians? What changes have been made recently as a result of this feedback?
Students, at any point, can contact the course director with concerns. Feedback is solicited multiple times during each block, with the ability to write comments about any lecture or lecturer.

Counseling/Student Support
10. What kind of academic, personal, financial, and career counseling is available to students? Are these services also offered to their spouses and dependents/children?
11. Is there a mentor/advisor system? Who are the advisors—faculty members, other students, or both?
Each student has an advisory dean who is available for any kind of help. Students also have "big sibs" - someone in the class above them who is available to answer questions and give advice.
12. How diverse is the student body? Are there support services or organizations for ethnic minorities and women?

Facilities
13. Tell me about the library and extracurricular facilities (i.e., housing and athletic/recreational facilities).
There is a medical center library, but most students tend to prefer the undergraduate library, which is nicer and very close. There is a medical student gym, which is very small, but restricted to medical students/residents. Students can also use the undergraduate athletic facilities. Housing is generally off-campus - some students live in apartment complexes, some live in townhouses and some buy their own house.
14. Are there computer facilities available to students? Are they integrated into the curriculum/learning?
There are computers available in the library, and there is a computer room for medical students. There is also free printing available. Students also receive their own laptop, which is set up with all of the programs needed, and is used during class to take notes. Students also get a PDA during second year. Both the laptop and the PDA are part of tuition and are owned by the student.
15. What type of clinical sites—ambulatory, private preceptors, private hospitals, rural settings—are available or required for clerkships? Does this school allow for students to do rotations at other institutions or internationally?
There are several hospitals that students can rotate at in Durham. In addition, for the family medicine and ob/gyn rotations, students may rotate at hospitals/clinics in different parts of the state. Students can choose to do these rotations in rural areas if they would like - there is usually more one on one time and students usually get more hands on experience this way. Students can rotate internationally during the 4 week elective second year, or during fourth year. Students can rotate at other institutions in the US during fourth year.
16. Is a car necessary for clinical rotations? Is parking a problem?
Yes, a car is necessary. Parking at Duke is not a problem, all students are giving parking passes for the parking garage.

Financial Aid
17. What is the current tuition and fees? Is this expected to increase yearly? If so, at what rate?
It's approximately $63,000 a year.
18. Are there stable levels of federal financial aid and substantial amounts of university/medical school endowment aid available to students?
Both merit scholarships and need-based scholarships are available.
19. Are there students who have an "unmet need" factor in their budget? If so, where do these students come up with the extra funds?
All student's needs are met with scholarship money or loans.
20. Are spouses and dependents/children covered in a student's budget?
21. Is someone available to assist students with budgeting and financial planning?
22. Does this school provide guidance to its students, and to its graduates/alumni, on debt management?

Student Involvement
23. What medical school committees (e.g., curriculum committee) have student representation?
24. Are students involved in (required or voluntary) community service?
Yes, there are lots of voluntary community service opportunities available, including work at the Ronald McDonald House, nearby shelters and soup kitchens, animal shelters, rural clinics, and student-run education programs at local middle schools.
25. How active is the Student Council/Government? Are there other student organizations?
Policies
26. Is there an established protocol for dealing with student exposure to infectious diseases? Is disability insurance provided to cover this exposure?
27. Does this school provide, or does the student pay for, vaccinations against Hepatitis B or prophylactic AZT treatment in case of a needle-stick or accident?
28. Is there a school Honor Code? Is there a grievance process/procedure? Are the students involved?
Residency
29. May I see a list of residency programs to which this school's recent graduates were accepted?

Questions to Ask Yourself
30. Does this school have strengths in the type of medicine (primary versus specialized care, urban versus rural practice environment, academic medicine versus private practice) that I will want to practice?
31. Would I be happy at this school (for at least the next four years)?
 
Curriculum
1. Are there any special programs for which this medical school is noted?
CCLCM is a five year research MD program affiliated with Case Western Reserve University and administered by the Cleveland Clinic Foundation. You can either get a MD with Biomedical Research Distinction or a full MD/MS. Getting the MS requires taking some extra classes.
2. Describe this school's curriculum in the pre-clinical and clinical years. Are there any innovations, like Problem-Based Learning?
First Year: We have about 20 contact hours per week. Of that, we spend about 2 hours in anatomy, 6 hours in PBL, 6 hours of other seminars, 1.5 hours of Foundations of Medicine, 1 hour of research seminar, and 4 hours of physical diagnosis class or clinic each week.
Second Year: Almost the same as the first, except that we have an extra four hours per week of clinic or physical diagnosis class. Also, we have fewer anatomy sessions.
Third-Fifth Years: Flexible. We each do a year of rotations, a year of research, and a year of electives and vacation.
3. Are there opportunities for students to design, conduct, and publish their own research?
Definitely. In fact, research is heavily emphasized and also required here. We all do 2 summers of lab rotations (one basic science, one clinical science) and one year of lab with a required MS-level thesis. During our first year, the 32 people in my class published 11 papers and presented 11 posters. I would guess by the time we all graduate that we'll all be published.
4. Is there a note-taking service? If so, is it university-run or student-run?
No note-taking service.
5. Is there flexibility in the coursework (the number of electives) and the timing of the courses (accelerating, decelerating, and time off) during the pre-clinical and clinical years?
Pre-clinical: you can take optional MS coursework; several of us are doing that.
Clinical: very flexible. We can do rotations at any of the Case hospitals as well as here at the Clinic.
As far as I know, there is no option to accelerate or decelerate.
6. Has this medical school, or any of its clinical departments, been on probation or had its accreditation revoked?
No.
7. How do students from this medical school perform on the National Board Examinations? How does the school assist students who do not pass?
So far, the one class that has taken the boards had a 100% pass rate. Their average score was 229.

Evaluations
8. How are students evaluated academically? How are clinical evaluations performed?
This school is P/F, unranked all five years. We do not take science exams here, but we have weekly multiple choice and essay homework assignments for years 1 and 2. We have frequent clinical exams (observed histories and physicals and OSCEs).
9. Is there a formal mechanism in place for students to evaluate their professors and attending physicians? What changes have been made recently as a result of this feedback?
Yes. We evaluate each class and each tutor at the end of each block. We also are regularly asked to serve on feedback committees to improve the blocks. Changes are made for future classes based upon student feedback.

Counseling/Student Support
10. What kind of academic, personal, financial, and career counseling is available to students? Are these services also offered to their spouses and dependents/children?
Each student has a physician advisor who is responsible for helping them with academic counseling. There is employee counseling available at the clinic and student counseling available over at Case. Financial counseling is provided by the financial aid office. Career counseling is provided by the student dean. I am not sure whether any of these services are available to student dependents.
11. Is there a mentor/advisor system? Who are the advisors—faculty members, other students, or both?
Yes. Each student is assigned to a physician advisor for the duration of their time here. This person is a member of the faculty. We also have research and clinical preceptors.
12. How diverse is the student body? Are there support services or organizations for ethnic minorities and women?
Very diverse. We have multiple international students and URMs. My class is half female, half male. There is a Women in Medicine club and support services for URMs.

Facilities
13. Tell me about the library and extracurricular facilities (i.e., housing and athletic/recreational facilities).
The CCF library is fairly small, but we also have access to all of the Case libraries. I rarely have to physically go to the library anyway, since most items are available on line. There are no housing facilities on the Clinic campus. There are three gyms: two at Case and one at the Clinic. The one on the Clinic campus is free to students, and so is one of the Case gyms. The other Case gym requires you to purchase a membership, but most of us don't because the two free ones are more than adequate.
14. Are there computer facilities available to students? Are they integrated into the curriculum/learning?
Yes. There are computers in the library and in the student lounge. We are also each given a free laptop (well, it's part of your tuition!) on the first day of school that we get to keep when we graduate. This curriculum is highly computer-integrated.
15. What type of clinical sites—ambulatory, private preceptors, private hospitals, rural settings—are available or required for clerkships? Does this school allow for students to do rotations at other institutions or internationally?
CCF has its own hospital, and we are a mile away from the Case campus. Case is affiliated with University Hospital, the Veteran's hospital, Rainbow Babies Children's Hospital, and Metro Hospital. CCLCM students can rotate at any of these sites. My understanding is that it will be possible to do away electives during the fourth or fifth year, but the first class of students is just starting their fourth year now, so no one has done this yet.
16. Is a car necessary for clinical rotations? Is parking a problem?
Most people have cars, but I've been assured that it is possible to get by without one. A few people in each class do not have cars. Parking is always a problem, as it is on any urban campus.

Financial Aid
17. What is the current tuition and fees? Is this expected to increase yearly? If so, at what rate?
Our fees and tuition is pretty expensive. We pay the same as Case, which is on the order of around $41,000 for tuition and another $20,000 for living expenses. I don't know what the plan is for increasing that. Tuition has increased slightly this year over last.
18. Are there stable levels of federal financial aid and substantial amounts of university/medical school endowment aid available to students?
Yes. On average, CCLCM students get half grants/scholarships, half loans. The individual experience varies. Some students get a full tuition scholarship plus stipend for the five years (like what an MSTP student gets), and some get very little grant or scholarship aid. The school plans to ultimately make the school tuition free for all students.
19. Are there students who have an "unmet need" factor in their budget? If so, where do these students come up with the extra funds?
Sorry, I don't know how to answer this.
20. Are spouses and dependents/children covered in a student's budget?
I don't know.
21. Is someone available to assist students with budgeting and financial planning?
Yes, Liz Myers in the financial aid office does this.
22. Does this school provide guidance to its students, and to its graduates/alumni, on debt management?
Yes, Liz Myers does this.

Student Involvement
23. What medical school committees (e.g., curriculum committee) have student representation?
I'm not sure. I know the adcomm has student members.
24. Are students involved in (required or voluntary) community service?
Yes. We are supposed to do service through our FCM class, and we also have a student-founded and student-run free health clinic called Community Health Initiative (CHI). Several other student clubs and organizations are available on both the Case and CCF campuses.
25. How active is the Student Council/Government? Are there other student organizations?
There are two student governments: one for just CCLCM, and one for all of Case. I'm not on either one, so I don't know too much about them beyond that they send out newsletters and organize parties every so often. There are several other organizations. Some include specialty interest groups (ex. neuroscience/psych, cardiology, HIV/AIDS), religious groups, and informal groups that meet for fun or to discuss journal articles.
Policies
26. Is there an established protocol for dealing with student exposure to infectious diseases? Is disability insurance provided to cover this exposure?
There is a protocol. I don't know how the insurance end of it works.
27. Does this school provide, or does the student pay for, vaccinations against Hepatitis B or prophylactic AZT treatment in case of a needle-stick or accident?
My understanding is that if you report the exposure, you will be treated. I don't know who pays for it though.
28. Is there a school Honor Code? Is there a grievance process/procedure? Are the students involved?
We have a class oath for all Case students. There is a grievance process. I don't know if students are involved with it.
Residency
29. May I see a list of residency programs to which this school's recent graduates were accepted?
No, because none of the CCLCM students have graduated yet. The first class will finish in 2009.

Questions to Ask Yourself
30. Does this school have strengths in the type of medicine (primary versus specialized care, urban versus rural practice environment, academic medicine versus private practice) that I will want to practice?
Yes. This school is geared toward people who want to do research and go into specialty care. It's still possible to do primary care, but that's not the school's focus. If you want to do primary care research (ex. health outcomes, epidemiology, etc.), either Case program would be a great choice.
31. Would I be happy at this school (for at least the next four years)?
This is a mandatory five year program. It's not for everyone, but I like it. I guess I'd say that if you're interested, come interview, and see what you think.
 
I'll answer what I can. I just finished my M2 year.

Curriculum
1. Are there any special programs for which this medical school is noted?
2. Describe this school's curriculum in the pre-clinical and clinical years. Are there any innovations, like Problem-Based Learning?
The first 2 years are in the lecture hall and in block format (4 blocks/semester). Some classes have occasional small groups -- PBL for psych, Case-Based Discussions for Biochem, Labs and Team-Based learning for path. Pharm has one small group thing at the very beginning of the year. Anatomy obviously has gross lab, neuroanatomy has gross lab and micro lab on the computer, and cell and tissue anatomy's lab is all on the computer. Phys has wet lab and computer lab sessions. Micro has a bacteriology lab. The vast majority of all classes are in large lectures.
After first semester M1 year, the courses start getting students used to taking exams on computers versus paper format.
M1 year
Semester 1: Biochem, Anatomy, Human Development, Psychiatry
Psych PBL + Med Interviewing OR Medical Information Management (epidemiology)
Semester 2: Physiology, Neuroanatomy, Cell and Tissue Bio
switch PBL/interviewing and MIM
M2 year
Semester 1: Pathology, Microbiology, Abnormal Psychiatry, Med Ethics, Clinical Exam and Reasoning (CER)
Semester 2: Pathology, Pharmacology, CER, Human Sexuality, Health Policy

3. Are there opportunities for students to design, conduct, and publish their own research?
The summer after M1 year there is the Medical Student Summer Research Program. My summer 101 students particpated (out of a class of 200). This summer I think 60 participated. There is a stipend, and opportunities for 8, 10, 12 week summer research and a 16 week (12 + 4 weeks rotation elective or your own time) honors in research.
A lot of students got publications, abstracts, or presentations at national specialty meetings through their summer research.
4. Is there a note-taking service? If so, is it university-run or student-run?
The co-op is student run. I wasn't a part of it, but I think people receive emails of the notes and hard copies. I heard the price ranged from $20-50/semester.
5. Is there flexibility in the coursework (the number of electives) and the timing of the courses (accelerating, decelerating, and time off) during the pre-clinical and clinical years?
There is the 5 year plan. The first year is spread over 2 years, and the M2 year is taken as it normally would be.
6. Has this medical school, or any of its clinical departments, been on probation or had its accreditation revoked?
Not that I'm aware of.
7. How do students from this medical school perform on the National Board Examinations? How does the school assist students who do not pass?
I've heard we do around the national average for the pass rate and the score. People who do not pass use their 3rd year elective to study more, and arrangements have been made to go take an extensive board prep class.

Evaluations
8. How are students evaluated academically? How are clinical evaluations performed?
We have H, HP, P, LP, F for preclinical and clinical grades.
Preclinical grades are based on exam scores and we sometimes receive comments from preceptors of small groups.
We are technically not ranked, but records of each student's performance compared to class performance for each exam is kept, and that's used for AOA and stuff.
No experience with clinical evals yet.
9. Is there a formal mechanism in place for students to evaluate their professors and attending physicians? What changes have been made recently as a result of this feedback?
We have mandatory course evals that if we don't fill out we don't get our grades (they are anonymous). The micro program was revamped a few years ago (micro was extremely well organized), and the neuroanatomy course director was changed this year to hopefully change the class.

Counseling/Student Support
10. What kind of academic, personal, financial, and career counseling is available to students? Are these services also offered to their spouses and dependents/children?
11. Is there a mentor/advisor system? Who are the advisors—faculty members, other students, or both?
1st year each student is assigned a mentor to go to their clinic and begin working with patients -- taking a history. 2nd year the mentor helps with the history and physical.
3rd year each student needs to find an advisor. I don't know more about that.
12. How diverse is the student body? Are there support services or organizations for ethnic minorities and women?
The student body is mostly white. There are a lot of Asians, African Americans and some Latinos. There's a Minority Affairs office. AMWA is active on campus.

Facilities
13. Tell me about the library and extracurricular facilities (i.e., housing and athletic/recreational facilities).
The library isn't open 24 hours. During the school year, it opens at 7:30am every weekday, and stays open til about 10:30. On Saturdays, it closes at 6pm, and on Sundays, it opens at 10 and closes at about 10:30. There are extended hours for finals, but not for boards studying.
There's a gym in the basement of the floor, and some of the colleges in the area give a discount to MCW students -- I've been told Marquette does, and Wisconsin Lutheran (across the street from MCW) advertises a discount at M1 orientation.
There's no campus housing, but there are lots of places near campus.
14. Are there computer facilities available to students? Are they integrated into the curriculum/learning?
There's a 24 hour computer lab that's closed basically the night before computerized exams. Cell/Tissue Bio, Neuroanatomy and Phys and (path has a couple) have lab sessions on the computer.
Phys, CTB, Path, and Micro have computerized exams.
15. What type of clinical sites—ambulatory, private preceptors, private hospitals, rural settings—are available or required for clerkships? Does this school allow for students to do rotations at other institutions or internationally?
For a partial answer:
If you set up an international rotation or rotation somewhere else, you can do it.
16. Is a car necessary for clinical rotations? Is parking a problem?
Yeah -- I just got assigned to a rotation 17 miles from my home.

Financial Aid
17. What is the current tuition and fees? Is this expected to increase yearly? If so, at what rate?
M3 tuition -- $35,920
It's gone up every year by about $1000.
18. Are there stable levels of federal financial aid and substantial amounts of university/medical school endowment aid available to students?
I think so.
19. Are there students who have an "unmet need" factor in their budget? If so, where do these students come up with the extra funds?
Yes. Some go to parents, some private loans.
20. Are spouses and dependents/children covered in a student's budget?
21. Is someone available to assist students with budgeting and financial planning?
Yes, the financial aid office is very helpful.
22. Does this school provide guidance to its students, and to its graduates/alumni, on debt management?
My friends who just graduated said they were told to call the office if they ever had any questions.

Student Involvement
23. What medical school committees (e.g., curriculum committee) have student representation?
Curriculum, Library, Admissions, there's a lot. Students are voted in. There's a rep from each class.
24. Are students involved in (required or voluntary) community service?
It's voluntary. A lot participate though
25. How active is the Student Council/Government? Are there other student organizations?
There are a lot of student organizations -- interest groups for specialties, and associations. The different groups have variable levels of activity. Some do a lot, and others basically just exist.
Policies
26. Is there an established protocol for dealing with student exposure to infectious diseases? Is disability insurance provided to cover this exposure?
Yeah. Occupational health comes and talks to the students at orientation/registration each year. I think there's disability insurance.
27. Does this school provide, or does the student pay for, vaccinations against Hepatitis B or prophylactic AZT treatment in case of a needle-stick or accident?
I think so.
28. Is there a school Honor Code? Is there a grievance process/procedure? Are the students involved?
Not sure. I think there's an honor code.
Residency
29. May I see a list of residency programs to which this school's recent graduates were accepted?
I don't have it.

Questions to Ask Yourself
30. Does this school have strengths in the type of medicine (primary versus specialized care, urban versus rural practice environment, academic medicine versus private practice) that I will want to practice?
31. Would I be happy at this school (for at least the next four years)?
 
1. Are there any special programs for which this medical school is noted?

RWJ offers a distinction in research program as well as a distinction in community service program.

2. Describe this school's curriculum in the pre-clinical and clinical years. Are there any innovations, like Problem-Based Learning?

The pre-clinical years are mostly subject based. We have classes in the usual med school courses, such as Gross Anatomy, Histology, Biochem, Genetics, Pathology, Medicine, Psychiatry, ect. There is also a new class that spans the entire first two years that is a combination of physical diagnosis, ethics, epidemiology and actual patient contact.

In the Camden program, there is special course during the third year. Every Wednesday we are relieved from our clerkships at 2 pm. From 2 - 8 pm, we are scheduled different lectures and are allowed to go off and run our student run clinics, as well as get involved in a variety of community service projects (such as teaching CPR, mentoring kids, ect.)

3. Are there opportunities for students to design, conduct, and publish their own research?

Yes. I was a participant in the Students Interested in Research (SIR) Elective. The way it works is that you join a lab during the second semester of your first year to plan a research project for the summer. If you are selected, you recieve a 2700$ stipend for the summer and get to present your results at the annual Poster Day. There are also opportunities to do research outside of SIR.

4. Is there a note-taking service? If so, is it university-run or student-run?
During my first two years, there was no offical note-taking service. That being said, the students got together and scribed many of the lectures in our core classes and passed them out to the entire student body. Every lecturer also provides the powerpoint slides to each lecture, and we also have the audio to each lecture available online.

5. Is there flexibility in the coursework (the number of electives) and the timing of the courses (accelerating, decelerating, and time off) during the pre-clinical and clinical years?

Yes. A few students that were in my class have deceled. RWJ also offers a student scholars program, that allows you take time off as long as you are doing something scholarly. Many of my classmates have gone off to do research, or go to the NIH.

6. Has this medical school, or any of its clinical departments, been on probation or had its accreditation revoked?

Not that I know of. Of course everyone knows about the UMDNJ scandal, but that has had no effect on student education.

7. How do students from this medical school perform on the National Board Examinations? How does the school assist students who do not pass?

I honestly have no clue. They do not tell us how we do on the USMLE. I don't know of any of my classmates who have failed the exam, therefore I can not tell you how they assist those who do not pass.

Evaluations

8. How are students evaluated academically? How are clinical evaluations performed?

I'm pretty sure that the new entering class with now be pass/fail for their first two years. When I was a first year, we had Honors/high pass/pass/low pass/fail. It really wasn't as bad as you might think. The classes were not curved, so your grade was your grade.

The last two years are still graded on the five letter scale. Basically, all of the attendings that you work with fill out an evaluation on you. The shelf exam can count anywhere from 20% of your grade, up to 60% of your grade depending on the rotation.

9. Is there a formal mechanism in place for students to evaluate their professors and attending physicians? What changes have been made recently as a result of this feedback?

Yes. Course and clerkship feedback is required before we can access our grades. The Camden campus is excellent at making changes based on what the students have to say.

Counseling/Student Support

10. What kind of academic, personal, financial, and career counseling is available to students? Are these services also offered to their spouses and dependents/children?
The student affairs deans are there if you have academic/career concerns. There is a student wellness program that can help you handle any personal crises (they are also confidential). The financial aid office would handle any financial concerns. Also, there are occasional lectures on financial/career issues during lunch, or during the Camden Continuity of Care course.

11. Is there a mentor/advisor system? Who are the advisors—faculty members, other students, or both?

Yes, you get assigned peer mentors (2nd year students) at the beginning of orientation during first year. They stick with you, answer any questions that you have, for the entire first year. You are also assigned a faculty advisor in the beginning of orientation.

After that, it is very easy to get in touch with faculty members in your area of interest. RWJ has a program called a "A Day in the Life" where you are excused from your classes/clerkship duties to go spend a day with a physician in any specialty you are interested in knowing more about.


12. How diverse is the student body? Are there support services or organizations for ethnic minorities and women?

Very diverse. There are groups for everyone. African American students, Hispanic students, Jewish students, women, Muslim students, Asian/Indian students, gay and lesbian students. I'm sure I'm missing some groups, but you get my point.

Facilities

13. Tell me about the library and extracurricular facilities (i.e., housing and athletic/recreational facilities).

The facilities are OK. There is no "real" library on the Piscataway campus, only a small media library, however we are on the Rutgers campus and are allowed to use their Science and Medicine library. The inside of the main building is kind of dull, not alot of windows, a little dreary. We have access to the Rutgers gym (for a fee) and the student center (they make good chinese food).

There is no housing provided. This isn't so bad though, Piscataway/NB is a college town, so there are plenty of places to live. Most students live in townhomes right next to the school, others choose to live in New Brunswick. I lived in NB for the first two years, and it was fine, but the traffic can get bad.

On the Camden campus, you also have to find your own housing. Half of the class lives in Philadelphia, the other half lives in South Jersey. No one lives in Camden, lol.

14. Are there computer facilities available to students? Are they integrated into the curriculum/learning?

There is a 24 hour computer area, and a more formal computer lab that closes at some point (not sure of hours).

On the Camden campus, there is a formal computer lab that is available 24 hours a day in the hospital, and a couple of computers in our student lounge.

15. What type of clinical sites—ambulatory, private preceptors, private hospitals, rural settings—are available or required for clerkships? Does this school allow for students to do rotations at other institutions or internationally?

Since I'm on the Camden campus, I can only speak for my location. Most of the rotations are at Cooper University Hospital. We also spend time in outpatient offices across the street from the hospital, as well as other outpatient offices in South Jersey (esp. during family medicine). We are encouraged to do international electives, as well as electives at other schools.

16. Is a car necessary for clinical rotations? Is parking a problem?
Yes. A car is also necessary for the first two years. Parking is not a problem on the Camden campus.

Financial Aid

17. What is the current tuition and fees? Is this expected to increase yearly? If so, at what rate?

About 22,000 per year. Keep in mind this is not including fees, housing costs, personal expenses, ect. The tuition increases every year, for the most part. What school does not increase their tuition every year?

18. Are there stable levels of federal financial aid and substantial amounts of university/medical school endowment aid available to students?

You will be covered by federal loans. I have not recieved any grants or scholarships, but maybe I'm not special enough. :D

19. Are there students who have an "unmet need" factor in their budget? If so, where do these students come up with the extra funds?

I had to use loans to cover everything. The federal loans covered all of my expenses, I did not need any private loans.

20. Are spouses and dependents/children covered in a student's budget?

Sorry, but I don't know the answer to this.

21. Is someone available to assist students with budgeting and financial planning?

The financial aid office, I suppose.

22. Does this school provide guidance to its students, and to its graduates/alumni, on debt management?

I'm pretty sure they do. I know the fourth years have to go through a full day of exit counseling.

Student Involvement

23. What medical school committees (e.g., curriculum committee) have student representation?

There is a curriculum committee, a student rights person. Honestly, there are more, but I don't know them all.

24. Are students involved in (required or voluntary) community service?

YES! This is one of the strengths of the school! It is very easy to get involved with community service. During the first two years we have a organization called HIPHOP (the homeless and indigent population health outreach project) where you can get involved in blood pressure screenings, teaching high school students about HIV, working with patients with cancer, helping out pregnant moms, helping families who may have lead exposures and many more projects. There is also a student run clinic in NB.

In Camden, we have many student run clinics. We have an Adult HOP clinic, and Women's Hop Clinic, and a Peds Hop Clinic. All of these clinics are fully run by the students, with faculty coming in to oversee us. We also teach CPR to the community, mentor local high school and elementary kids, teach parents about important health topics at a local daycare. Just about every Camden student is involved in some kind of community service.

25. How active is the Student Council/Government? Are there other student organizations?

They are pretty active. There are alot of student organizations. I mentioned some in the diversity question. There are also organizations for just about every specialty imaginable, and also a global health club, AMSA, AMA, AMWA, ect.

Policies

26. Is there an established protocol for dealing with student exposure to infectious diseases? Is disability insurance provided to cover this exposure?
We are given a card that lists where we go if whe are exposed to anything. I'm not sure about disability insurance, but I know that we don't pay anything out of pocket.

27. Does this school provide, or does the student pay for, vaccinations against Hepatitis B or prophylactic AZT treatment in case of a needle-stick or accident?

Yes, they do pay for vaccinations. I'm pretty sure they also pay for treatment.

28. Is there a school Honor Code? Is there a grievance process/procedure? Are the students involved?

Yes, there is an honor code. In fact, we sign it before every test. There is also a grievance process/procedure, and I do believe there is a student rep.

Residency

29. May I see a list of residency programs to which this school's recent graduates were accepted?

http://rwjms.umdnj.edu/admissions/our_students_match_list_2007.htm

If you go to the RWJMS website, there are additional match lists from previous years under the FAQ area.

Questions to Ask Yourself

30. Does this school have strengths in the type of medicine (primary versus specialized care, urban versus rural practice environment, academic medicine versus private practice) that I will want to practice?

Am I suppose to answer this? :D I feel like every specialty is open to me as a RWJ graduate.

31. Would I be happy at this school (for at least the next four years)?

I'm pretty happy. Of course there are days I hate medicine, but I think I would feel that anywhere. The best thing I did was choosing the Camden campus!
 
Curriculum
1. Are there any special programs for which this medical school is noted?

MSTP (MD/PhD); MD/MPH; MD/JD; Research Distinction Track; Service Distinction Track; Gold Humanism Society; Master's degree in medical education; very strong PA program, which is somewhat integrated in the medical curriculum.

2. Describe this school's curriculum in the pre-clinical and clinical years. Are there any innovations, like Problem-Based Learning?
Preclinical is mostly lecture, with multiple small groups. There are 4 semesters, with 3-5 classes. The "credit load" is 19-23 credits per semester. Lectures and scheduled small groups typically concentrated in the mornings, with labs, small group time, and study time in the afternoons. We do have case-based learning (first year) and patient-centered learning (second year) as part of our longitudinal "Foundations of Clinical Practice" course, which also includes ethics, physical exam, interviewing, human sexuality, and an introduction to clinical disciplines. Labs include gross anatomy, neuroanatomy, histology, pathology, and principles of infectious diseases.

The schedule is:
First semester – gross anatomy, biochemistry, cell biology, genetics, FCP I
Second semester – neuroscience, immunology, human organ systems, FCP II
Third semester – pathology I, pharmacology, infectious diseases, FCP III
Fourth semester – pathology II, health law, FCP IV

Our clinical curriculum is nice. We have standard 6 week blocks of required core clerkships (ob-gyn, medicine, surgery, and pediatrics). Also during third year, you take 4 weeks of psychiatry and a twelve week block called the Ambulatory Practice Module (family medicine, outpatient internal medicine, and community-based primary care). Additionally, neurology (4 weeks) and eight two-week clerkships are required either during third or fourth years (dermatology, radiology, orthopedics, ophthalmology, urology, otolaryngology, anesthesiology, and lab medicine / EKG). Fourth year includes a sub-internship, ICU or ER, and various electives.

3. Are there opportunities for students to design, conduct, and publish their own research?
Tons. Aside from the structured offerings (dual degree programs and the research distinction tract = http://www.medicine.uiowa.edu/imsrp/components/rdt.html ), many students do either basic science or clinical research, either before M1, between M1 & M2 (as part of the Summer Research Fellowship = http://www.medicine.uiowa.edu/imsrp/components/srf/) , or as an elective in fourth year. We also are a host school for the Doris Duke Fellowship (http://www.medicine.uiowa.edu/programs/dorisduke/) .

4. Is there a note-taking service? If so, is it university-run or student-run?
Yes – student-run

5. Is there flexibility in the coursework (the number of electives) and the timing of the courses (accelerating, decelerating, and time off) during the pre-clinical and clinical years?
Yes – the pre-clinical years can be decelerated to 3 years. There are some electives offered across class years (such as intro to US healthcare systems). There are also a variety of senior electives to choose from. Also, many global electives are offered, either between M1 & M2, or during M4 years.

6. Has this medical school, or any of its clinical departments, been on probation or had its accreditation revoked?
Not that I am aware of.

7. How do students from this medical school perform on the National Board Examinations? How does the school assist students who do not pass?
Our scores are above average on the step exams. We had school sponsored review course prior to Step I and an informal review network to prepare for Step II CK. Most of our core clerkships have Patient-Based Assessments, aimed at preparing us for Step II CS. We have a counseling center and tutors available to help students who fail (I've also heard of faculty members specifically volunteering to help students who are having difficulty).


Evaluations
8. How are students evaluated academically? How are clinical evaluations performed?

Grades are Honors; Near Honors; Pass; or Fail in all four years (with a few exceptions which are strictly Pass / Fail). Courses aim to have roughly the top 15% in Honors, and the next 15% in Near Honors, with the overall majority in Pass (sometimes there will be more in the upper grades, if the course has predefined percentages, like > 90% correct for Honors, etc). Clinical grades usually come from approximately 50% clinical evaluations and 50% exams / assignments (varies by clerkship, of course). Small group and clinical evaluations are included in the dean's letter process.

9. Is there a formal mechanism in place for students to evaluate their professors and attending physicians? What changes have been made recently as a result of this feedback?
Evaluation is taken seriously at the school. Lecturers, attendings, and courses/clerkships are all evaluated anonymously. Additionally, a percentage of the class meets at scheduled luncheons through the clinical years to discuss how things are going as a whole. We also have student-run liaison committees for each preclinical course, which is a small group of students who meets with the course director throughout the semester to voice the concerns of the students and disseminate information back to the class.

Counseling/Student Support
10. What kind of academic, personal, financial, and career counseling is available to students? Are these services also offered to their spouses and dependents/children?

Our counseling center includes two full time counselors who help with academic, personal, and career issues. As mentioned above, the counseling center also organizes tutor groups, which are available to anyone interested (but devised specifically for those who are having difficulties). The counseling center organizes wellness events throughout the year. More in-depth mental health services are available at the student health center, which is easily accessible to the health campus. Two financial aid officers provide all financial counseling, and they also offer various workshops throughout the year for specific issues (debt, budgeting, financial planning, etc). Spouses are welcome at financial seminars / workshops. There is also an active core of individuals who are working on improving access to services like childcare for students who are parents.

11. Is there a mentor/advisor system? Who are the advisors—faculty members, other students, or both?
We have a community structure. Each student is assigned to one of four learning communities, which help vertically integrate students across the four years. Each community has a faculty head, as well as mayors and a council that form the basis for the student government. Other faculty members are also members of communities, and several programs have been formed to match newer students with peer or faculty mentors. As part of FCP I, we have a small group entitled "Personal and Professional Development", which also serves as a way to link to volunteer faculty members. And, each clinical department has multiple faculty members who offer guidance to individuals interested in their specific field. Students typically seek out their assistance during the third and fourth years.

12. How diverse is the student body? Are there support services or organizations for ethnic minorities and women?
I don't have the exact numbers, but I would say our proportions are in tune with national averages. Counseling services and career advising is available from faculty. There are multiple student organizations aimed at diversity (Student National Medical Association, American Medical Women's Association, Med Iowa's Queer Students, Medical Spanish Class). There are also volunteer initiatives aimed at diverse patient populations (Proteus clinic for migrant workers, the student-run Mobile Clinic, the Victory Temple Minority Health Initiative, etc). Finally, there is also a non-credit elective called "Conversations", which includes weekly meetings to discuss cultural competency and diversity.

Facilities
13. Tell me about the library and extracurricular facilities (i.e., housing and athletic/recreational facilities).

The Health Sciences library (Hardin) is located very near the new teaching facility (where the preclinical courses, teaching clinical suites, and communities are located), as well as the University of Iowa Hospitals and Clinics (where many of our clinical rotations are). The library includes reference texts and journals, of course, but most resources are also available online. The UI field house (the fitness center) is adjacent to the hospital and readily accessible. Most students live off campus, although there are two medical fraternities. The cost-of-living is typical of college towns (higher than the surrounding areas in the Midwest, but not nearly as bad as bigger cities).

14. Are there computer facilities available to students? Are they integrated into the curriculum/learning?
There are large teaching computer classrooms in the library, but we often do not use those. Our new building ("MERF") has 3 large lecture halls, many conference rooms for small group instruction, and a mid-sized computer lab (which is also used for instruction). Many exams are computer-based, administered in either the lab or on laptops in a lecture hall. Each community also has small computer workrooms, as well as mailboxes and plenty of furniture and work tables. Several classes have required activities on the computer (like histology, pathology, etc). However, we also have recommended computer-based materials available in most courses.

15. What type of clinical sites—ambulatory, private preceptors, private hospitals, rural settings—are available or required for clerkships? Does this school allow for students to do rotations at other institutions or internationally?
Most required rotations are done at UIHC (locally). Some can be completed at the VA (which is literally across the street – 2 minute walk from UIHC). Pediatrics, Ob-Gyn, and Internal Medicine can be taken at Des Moines if preferred. Pediatrics also requires a week in a private practice clinic. As part of the ambulatory block, family medicine and community-based primary care must be taken outside of UIHC at clinical locations across the state of Iowa. Stipends are available for family medicine rotations in rural / underserved areas. Fourth year electives can be arranged at other schools. International electives are encouraged, and receive credit through the Global studies program but must be scholarly in nature.

16. Is a car necessary for clinical rotations? Is parking a problem?
A car is required during the clinical years. There are parking ramps at the hospital and by MERF, but parking during the day is a pain and somewhat costly. There are free spots available in the evening. Commuting students can take the free campus buses, local Iowa City / Coralville buses, or park in commuter lots (which are about 10-15 minutes away on foot, but have shuttles). During the clinical years, students are sometimes offered low-cost parking passes for the parking ramps for the extent of their rotation.


Financial Aid
17. What is the current tuition and fees? Is this expected to increase yearly? If so, at what rate?

Ok – the total budget is scary, but keep in mind that this includes cost of living and supplies: http://www.medicine.uiowa.edu/osac/financial/0708coa.pdf
The cost of attendance has increased for residents at a rate above the national average, but I think the rate for non-residents is comparable to other schools.

18. Are there stable levels of federal financial aid and substantial amounts of university/medical school endowment aid available to students?
I believe so.

19. Are there students who have an "unmet need" factor in their budget? If so, where do these students come up with the extra funds?
I believe so but I don't know the details. I'd start here: http://www.medicine.uiowa.edu/osac/financial/CostofAttendance.htm

20. Are spouses and dependents/children covered in a student's budget?
Yes

21. Is someone available to assist students with budgeting and financial planning?
Yes – there are two financial aid officers who help with this. Also, there are financial / debt consultants available from the UI community as a whole.

22. Does this school provide guidance to its students, and to its graduates/alumni, on debt management?
Yes, as explained in #10 above.

Student Involvement
23. What medical school committees (e.g., curriculum committee) have student representation?

Admissions, Curriculum (focuses more on content of the curriculum), Medical Education (focuses more on the process of teaching), Clerkships, Promotions, Honor Council, Caduceus (runs liaison committees), Semester committee (provides input on scheduling, exams, etc), Continuing Medical Education, Student Awards and nominations, Research in Medical Education (evaluates new teaching initiatives)

24. Are students involved in (required or voluntary) community service?
Many voluntary projects. There are student-initiated, student-run clinics. First was the Mobile Clinic (http://www.healthcare.uiowa.edu/Programs/MobileClinic/). Recently, a Free Mental Health clinic was developed. There are also opportunities through many of the student organizations, the learning communities, and local agencies (notably, the Free Medical Clinic, the local hospice, Emma Goldman clinic, and the Crisis Center). There is a service-learning based elective called Community Health Outreach, available during the first two years.

25. How active is the Student Council/Government? Are there other student organizations?
Student government is active and tries to provide a conduit of communities between the students and with the administration. The medical school government actively integrates the learning communities, and has also been working with the student governments from the other health professions. Student organizations, based on common interests, backgrounds, and/or career goals, are also plentiful (http://www.medicine.uiowa.edu/osac/studentorg/index.htm).

Policies
26. Is there an established protocol for dealing with student exposure to infectious diseases? Is disability insurance provided to cover this exposure?]

Yes – there is an exposure protocol through student health.

27. Does this school provide, or does the student pay for, vaccinations against Hepatitis B or prophylactic AZT treatment in case of a needle-stick or accident?
Had to look this one up, but Hep B series must be completed by the end of the first year and is available through student health. Post-exposure prophylaxis is initiated immediately, when appropriate by the needle-stick protocol.

28. Is there a school Honor Code? Is there a grievance process/procedure? Are the students involved?

Yes – the honor code was established BY the medical students (http://www.medicine.uiowa.edu/osac/administration/honorcodecurrent.pdf) . It takes the form of the Honor Council, which provides an alternate manner for dealing with grievances. When complaints are levied, individuals have the option to go through the Promotions committee (faculty driven, with some student representation), or through the Honors Council. Either the promotions committee or the Honors Council makes recommendations to the Dean's office, who takes the final action.

Residency
29. May I see a list of residency programs to which this school's recent graduates were accepted?

Match results are posted each year: http://www.medicine.uiowa.edu/osac/programsrecords/match/matchresults.htm

Questions to Ask Yourself
30. Does this school have strengths in the type of medicine (primary versus specialized care, urban versus rural practice environment, academic medicine versus private practice) that I will want to practice?

Iowa is a good school for those interested in primary care and provides many opportunities (both required and voluntary). For those interested in specialties, some of our programs are highly regarded (otolaryngology, ophthalmology, orthopedics to name a few). One of the limitations is patient diversity (Iowa is mostly white, largely rural). However, there are lots of opportunities for those interested in geriatrics and rural medicine. There are urban centers available (Des Moines, Cedar Rapids, and the "QuadCities"). We don't, however, do much in private practice settings. Academically, Iowa is a strong school and there is a solid research tradition, but Iowa may lack of the name recognition compared to the coasts (and the big guns in the Midwest). One of the benefits of being a tertiary care center is that we get patients from across the state, so you get to see a great deal of interesting pathology as well as a disproportionate percentage of needy patients.

31. Would I be happy at this school (for at least the next four years)?
I'm from the East Coast and had no connections to the area, but I was pleasantly surprised. Iowa City is a fun college town with lots of culture and history (check out the Iowa Writer's Workshop). It's a big ten school, too – so sports (and drinking) are also predominant in the area. I have a lot of respect for the administration – it's very supportive, compared to other institutions in the region (I'm basing this on information gleaned from regional meetings with representatives from other medical schools). The student body may appear a little cliquish at first, since many instate students come from similar schools. However, students are generally quick to find their own niche.
 
Curriculum
1. Are there any special programs for which this medical school is noted?
Supposedly pretty good primary care emphasis. Very popular MPH program designed specifically for med students that is able to be completed in 1 year.

2. Describe this school's curriculum in the pre-clinical and clinical years. Are there any innovations, like Problem-Based Learning?
I can't comment on the first year experience personally as it changed the following year. From my understanding there are 4 blocks (Host Defense and Microbial Pathogens, Integrative Function and its Cellular Basis, Molecules to Cells, and Structure and Development) . The second year is divided into organ systems which each take approximately one month complete. Pharmacology is integrated (for better or for worse) into the individual organ systems. The pre clinical years are mostly lecture style although for each block there tends to be some form of small group sessions worked in.

3. Are there opportunities for students to design, conduct, and publish their own research?
Between first and second years there is a popular research grant that you can apply for. There is also a special 2 year research program you can apply to after your third year if you're really keen on research. The faculty are also receptive to doing research pretty much anytime.

4. Is there a note-taking service? If so, is it university-run or student-run?
No, although there were occasionally emails floating around offering folks money for their notes specifically for other classmates with some learning disabilities.

5. Is there flexibility in the coursework (the number of electives) and the timing of the courses (accelerating, decelerating, and time off) during the pre-clinical and clinical years?
I know a number of classmates who decelerated after first year (I don't know if this was by choice.) Many, many people take a year off after their third year to purse either an MPH or research. There is not too much flexibility in the coursework the first two years; however, there are classes you can take primarily during lunch hours or in the evenings (social medicine type stuff) that you can put towards fourth year credit. Typically 2 of these classes equal 1 month of 4 year, and you are only allowed to get one month's worth of credit.

6. Has this medical school, or any of its clinical departments, been on probation or had its accreditation revoked?
Not during my time there.

7. How do students from this medical school perform on the National Board Examinations? How does the school assist students who do not pass?
We're told that typically 1-2 students do not pass each year. I do not know what help is offered to these students.

Evaluations
8. How are students evaluated academically? How are clinical evaluations performed?


9. Is there a formal mechanism in place for students to evaluate their professors and attending physicians? What changes have been made recently as a result of this feedback?
There is currently a huge emphasis placed on the feedback given by students both in the pre-clinical and clinical years. During the first 2 years, we could not receive our grade for the block until we had completed our evaluations. The administration does appear to be receptive to thoughtful responses that represent a majority of the class. For instance, one class eloquently explained their problems with the anatomy class and the individuals leading the class. The next year the entire system was completely revamped, and the current students appear to enjoy the class much more. The school also spoke with the administration about the hours the library was open, and these were greatly expanded because of our requests.

Counseling/Student Support
10. What kind of academic, personal, financial, and career counseling is available to students? Are these services also offered to their spouses and dependents/children?
Financial aid office is incredibly difficult to work with. However, I think they have recently made some changes to the staff who I have not yet worked with. There is a

11. Is there a mentor/advisor system? Who are the advisors—faculty members, other students, or both?
During the latter half of your third year you choose an advisor in the field in which you are interested.

12. How diverse is the student body? Are there support services or organizations for ethnic minorities and women?
Fairly diverse. SNMA and AMWA are both prominent organizations on campus.

Facilities
13. Tell me about the library and extracurricular facilities (i.e., housing and athletic/recreational facilities).
Library was redone within the last 3 years. My one complaint about the library is that UpToDate will only be available on campus starting July 1st. Medical students have access to all UG recreational and athletic facilities.

14. Are there computer facilities available to students? Are they integrated into the curriculum/learning?
You are required to buy a (pricey) laptop prior to starting your first year. The syllabus and powerpoints are all available on line. There is a computer lab in the library; however you have to pay for printing and there is typically a long line of undergraduates using the medical sciences library.

15. What type of clinical sites—ambulatory, private preceptors, private hospitals, rural settings—are available or required for clerkships? Does this school allow for students to do rotations at other institutions or internationally?
Huge variety of sites available to rotate through. The state is divided into AHEC regions (Charlotte, Greensboro, Wilmington, Area L, Wake, and Chapel Hill) and for many rotations you rank these sites and are magically assigned one or the other.

16. Is a car necessary for clinical rotations? Is parking a problem?
A car is definitely necessary. People have been scattered throughout the state; Wilmingtonone month and Asheville the next. While at UNC, you have to pay for parking. However, this is the one site where I think this is the case.

Financial Aid
17. What is the current tuition and fees? Is this expected to increase yearly? If so, at what rate?
I don't know the exact dollar amount, but its ~10,000 for tuition a year. There have been some (relatively) huge increases recently but I don't know if this is expected to continue. I believe I read somewhere about a cap being placed on how much the UNC system tuition can increase by.

18. Are there stable levels of federal financial aid and substantial amounts of university/medical school endowment aid available to students?
There are small scholarships that you don't have to apply to that vary in amount based on the amount of money the medical school received that year.

19. Are there students who have an "unmet need" factor in their budget? If so, where do these students come up with the extra funds?
I don't think anyone has "unmet needs."

20. Are spouses and dependents/children covered in a student's budget?
Spouses are not. I do not know about children.

21. Is someone available to assist students with budgeting and financial planning?
Not that I know of.

22. Does this school provide guidance to its students, and to its graduates/alumni, on debt management?
Yes. Not a lot, but some.

Student Involvement
23. What medical school committees (e.g., curriculum committee) have student representation?
I don't know

24. Are students involved in (required or voluntary) community service?
There is no required community service. Many students are involved in voluntary service. A popular option is SHAC a student run free clinic in the neighboring town of Carrboro.

25. How active is the Student Council/Government? Are there other student organizations?
Lots of active student organizations that change based on the interests of a particular class.

Policies
26. Is there an established protocol for dealing with student exposure to infectious diseases? Is disability insurance provided to cover this exposure?
Yes, there is an established protocol. I don't know about whether disability insurance covers this.

27. Does this school provide, or does the student pay for, vaccinations against Hepatitis B or prophylactic AZT treatment in case of a needle-stick or accident?
I don't know. I think the school does.

28. Is there a school Honor Code? Is there a grievance process/procedure? Are the students involved?
There is an honor code and an Honor Court composed of students with faculty involved as well. There is a grievance process in place.

Residency
29. May I see a list of residency programs to which this school's recent graduates were accepted?
Match Lists for 2001-2007 UNC Graduates



Questions to Ask Yourself
30. Does this school have strengths in the type of medicine (primary versus specialized care, urban versus rural practice environment, academic medicine versus private practice) that I will want to practice?
31. Would I be happy at this school (for at least the next four years)?
 
Curriculum
1. Are there any special programs for which this medical school is noted? We are noted for a PBL-based curriculum. It is not completely PBL based like many people think though. It is more like 1/3 PBL, 1/3 lecture, 1/3 small groups. Class is from 8-1 most days in the first two years. One afternoon a week you shadow a clinical preceptor to get experience talking to patients and doing physical exams. In addition, a new clinical skills center has just been built which has exam rooms equipped with cameras to give feedback to students on their physical exam and interview skills. In addition, there are dummies that can simulate a variety of medical emergency situations so students can get some experience in different medical scenarios.
2. Describe this school's curriculum in the pre-clinical and clinical years. Are there any innovations, like Problem-Based Learning? In the first year, there are three integrated courses. The first block lasts from September to December, and consists of biochemistry, molecular biology, cell biology, and genetics. The second block beings in January and lasts until April; it consists of anatomy, physiology, histology, and embryology. This is different than a lot of schools because we don't start anatomy until January. The last block goes from April to June; it has general pathology, immunology, introduction to pharmacology and microbiology. In the second year, we take neuro for the first 10 weeks and pathophysiology for the rest of the year. We have 8 weeks to study for boards. The clinical years are similar to other schools with required clerkships and some elective time. Many people do an abroad program in the fourth year because our school has a ton of different international opportunities.
3. Are there opportunities for students to design, conduct, and publish their own research? There are a lot of research opportunities. We are affiliated with Rockefeller University, Memorial Sloan Kettering Cancer Center (one of the top cancer centers in the country), and Hospital for Special Surgery (the top orthopedic and rheumatology hospital in the country). People in my class have worked in each of these places in the summer or taken a year off to do research fellowships. Most people either do research in the first year summer or go abroad. Many people end up with publications during medical school.
4. Is there a note-taking service? If so, is it university-run or student-run? There is a university-run note-taking service. Every lecturer has to provide the school with detailed notes and references about their topic. This is very convenient for the students here as all our notes are in one place.
5. Is there flexibility in the coursework (the number of electives) and the timing of the courses (accelerating, decelerating, and time off) during the pre-clinical and clinical years? There is not really much flexibility in the preclinical years as far as elective courses. The school does have people take time off for personal reasons but I don't know anyone on a decelerated course. Electives can be taken in the third year if desired.
6. Has this medical school, or any of its clinical departments, been on probation or had its accreditation revoked? I know of no departments on probation.
7. How do students from this medical school perform on the National Board Examinations? How does the school assist students who do not pass? Students do very well on the boards here. The average is 235 which is almost one standard deviation above the mean. There are people who get in the 260s and 270s which are among the top scores in the country. The school doesn't really do anything to help us with the boards (they basically don't acknowledge its existence), but everyone does well anyway (probably because they give us so much time to prepare; you could theoretically take more than the 8 weeks given because we have a 1 month "intro to clerkships" after the board study period which doesn't require much work. In reality you could take 12 weeks to study for the boards if you wanted). I don't know anyone who didn't pass.

Evaluations
8. How are students evaluated academically? How are clinical evaluations performed? Academically, we have a H/P/F system in the preclinical years. This is based on quiz average, PBL evaluations, and the grade on the final exam (which is a clinical case that we have to write about). Clinical years are H/HP/P/F (the majority of people receive either H or HP though). This grade is based on evaulations and the shelf at the end of the rotation.
9. Is there a formal mechanism in place for students to evaluate their professors and attending physicians? What changes have been made recently as a result of this feedback? We fill out evaluations at the end of all our courses. The school is actually very responsive to this feedback, and things change every year as a result of it (lectures are eliminated or added, the structure of the courses change, ect.). This year, instead of having a clinical preceptor all year, the 1st year students got to have some elective time instead because the 2nd year class expressed a desire for this.
Counseling/Student Support
10. What kind of academic, personal, financial, and career counseling is available to students? Are these services also offered to their spouses and dependents/children? Career counseling is not very formalized at this school. Most people identify a mentor from research or shadowing and use them as their guide.
11. Is there a mentor/advisor system? Who are the advisors—faculty members, other students, or both?
12. How diverse is the student body? Are there support services or organizations for ethnic minorities and women? The student body is pretty diverse. There are people from all parts of the country, many different backgrounds, and many different ages. I think our school tends to like non-traditional students and our class is definitely not the "straight out of college" type.

Facilities
13. Tell me about the library and extracurricular facilities (i.e., housing and athletic/recreational facilities). Our facilities are great (especially for New York City). We have a very nice library with many computers. Our gym has a basketball court and a recently remodeled weight room (it is in the dorm so it is very convenient). Most 1st years live in a dorm right across the street from the school (heavily subsidized). 2nd years move into 1-2-3 bedroom apartments which are also right across the street. They have all been remodeled in the past year or two. They are also heavily subsidized. There are studio options as well although there is a lottery system for this.
14. Are there computer facilities available to students? Are they integrated into the curriculum/learning? There is a whole floor of computer rooms for the students. We use computers for pathology, pathophys, presentations, ect. All powerpoint lectures are also online.
15. What type of clinical sites—ambulatory, private preceptors, private hospitals, rural settings—are available or required for clerkships? Does this school allow for students to do rotations at other institutions or internationally? Clinical clerkships can be taken at NY Presbyterian (the big teaching hospital), Lincoln Hospital (Bronx), NY Hospital Queens (Queens), St. Barnabas (Brooklyn), Westchester for psychiatry. We do some of our rotations at these sites away from NY Presbyterian. These away hospitals are private hospitals with a large underserved population so it is a different experience than a NY Presbyterian. You can do elective rotations at other institutions and many people do go abroad here (30%) for elective rotations (the school has many different programs and you can also design your own).
16. Is a car necessary for clinical rotations? Is parking a problem? Cars are not necessary (the school with provide you with shuttle service or black car service to any away site).

Financial Aid
17. What is the current tuition and fees? Is this expected to increase yearly? If so, at what rate? Tuition is $35,125 this year. It increases every year in line with other comparable schools.
18. Are there stable levels of federal financial aid and substantial amounts of university/medical school endowment aid available to students? The school gives need-based aid. It seems pretty generous to most people.
19. Are there students who have an "unmet need" factor in their budget? If so, where do these students come up with the extra funds?
20. Are spouses and dependents/children covered in a student's budget?
21. Is someone available to assist students with budgeting and financial planning?
22. Does this school provide guidance to its students, and to its graduates/alumni, on debt management?

Student Involvement
23. What medical school committees (e.g., curriculum committee) have student representation? Students are on all committees.
24. Are students involved in (required or voluntary) community service? There are many community service activities, and many people paricipate in them. It can be everything from teaching elementary school kids from disadvantaged backgrounds about diabetes to volunteering at homeless shelters. Most people do some kind of volunteer work here even though it is not required.
25. How active is the Student Council/Government? Are there other student organizations? There are many student organizations (30+). If there is one that is missing, you can create an organization and get funded for it easily.

Policies
26. Is there an established protocol for dealing with student exposure to infectious diseases? Is disability insurance provided to cover this exposure? Yes there is a protocol to deal with students exposed to infection.
27. Does this school provide, or does the student pay for, vaccinations against Hepatitis B or prophylactic AZT treatment in case of a needle-stick or accident?
28. Is there a school Honor Code? Is there a grievance process/procedure? Are the students involved? There is an honor code. I don't know how it works (I've never heard of problems).

Residency
29. May I see a list of residency programs to which this school's recent graduates were accepted? http://library.med.cornell.edu/Library/PDF/Matchlist07.pdf
Most people by the time residency applications come around want to stay in New York or go back to wherever they are originally from. This makes interpreting match lists kind of difficult because by the end of fourth year people have heavy geographic requirements. This is something I didn't realize as a med school applicant so you should interpret all match lists (not just ours) with caution.

Questions to Ask Yourself
30. Does this school have strengths in the type of medicine (primary versus specialized care, urban versus rural practice environment, academic medicine versus private practice) that I will want to practice? Most graduates of this school specialize although there are some people who want to do primary care. Students tend to be more academic minded here due to the research opportunities and the clinicians that we interact with on a regular basis. We get exposure to urban medicine.
31. Would I be happy at this school (for at least the next four years)? I think this school provides its students with great opportunities in a fun city to live for four years. You can do anything you want in medicine from here whether it is becoming a prominent researcher or a private practice clinician. There is a lot of money being put into the school right now and the students will reap the benefits of it. I think this is a great place to spend four years (or more with residency).
 
Curriculum
1. Are there any special programs for which this medical school is noted?

Georgetown is one of the few schools to emphasize Complimentary and Alternative Medicine (CAM.) This is the study and understanding of how alternative methods can be integrated into a traditional practice. As many of our patients are becoming more educated about these alternatives, it is crucial that we as physicians can meet their needs.

2. Describe this school's curriculum in the pre-clinical and clinical years. Are there any innovations, like Problem-Based Learning? We have a block schedule in the pre-clinical years.

3. Are there opportunities for students to design, conduct, and publish their own research?
Not only are there opportunities, Georgetown, with the M'11 class has included a research requirement. We are actually being encouraged to do this. There are stipends that are available, especially for summer research projects.

4. Is there a note-taking service? If so, is it university-run or student-run?
Yes there is a service. It is student run and is a summary of lectures.

5. Is there flexibility in the coursework (the number of electives) and the timing of the courses (accelerating, decelerating, and time off) during the pre-clinical and clinical years?
In the pre-clin years, there are a few electives that can be taken (not many) but there is not really any flexibility in the timing of the classes.

6. Has this medical school, or any of its clinical departments, been on probation or had its accreditation revoked?
Not to the best of my knowledge. There was an issue several years back with the solvency of the school, but it has been resolved for years now.

see: http://www.georgetownuniversityhospi....cfm?id=555769
http://www.thehoya.com/news/020207/news4.cfm

7. How do students from this medical school perform on the National Board Examinations? How does the school assist students who do not pass?


Evaluations
8. How are students evaluated academically? How are clinical evaluations performed?

Pre-clin evaluations are Honors/High Pass/Pass/Low Pass/Fail

9. Is there a formal mechanism in place for students to evaluate their professors and attending physicians? What changes have been made recently as a result of this feedback?
Pre-clin - there is a completely anonymous feedback system in place for every course and every preceptor and instructor. The administration often makes changes based on student feedback. For instance, there has been a move to reduce lecture hours and to increase coordination of course material between courses to prevent unnecessary repetition.

Counseling/Student Support
10. What kind of academic, personal, financial, and career counseling is available to students? Are these services also offered to their spouses and dependents/children?

The Student Ombuds Office (http://www3.georgetown.edu/som/stude...udsoffice.html) caters to listen and help student's concerns that can range from sexual harassment; discrimination in one of its many possible forms; professional misconduct; abusive behaviors; interpersonal strife; or the environment for study, teaching, and learning. All information is strictly kept confidential and no record of the cenversation is maintained.

11. Is there a mentor/advisor system? Who are the advisors—faculty members, other students, or both?
There is a formal and informal mentor and advisor system. During First Year orientation, you will be assigned a big sib: a second year student who will be there to answer many of your questions. You will also be placed into a group of 10 and all 10 of you will be assigned an advisor from the faculty. Further, many of the students in the school are more than willing to help others out.

12. How diverse is the student body? Are there support services or organizations for ethnic minorities and women?
Georgetown has a program, GEMS, that only enrolls under-represented minorities and disadvantaged students. Students who are successful in this program will be accepted directly to Georgetown. This adds greatly to the diversity in our school.

Facilities
13. Tell me about the library and extracurricular facilities (i.e., housing and athletic/recreational facilities).

Dahlgren Memorial Library has four floors that includes computer facilities, group study, and quiet study areas. Since the medical campus is located on within Georgetown University campus, there are numerous other libraries at short walking distance to mix study areas. There are no campus housing available for medical students, but most students live 5 to 10 minute walking distance from campus or in Rosslyn, VA, which take about 5 minutes by Georgetown University shuttle bus.


14. Are there computer facilities available to students? Are they integrated into the curriculum/learning?
Computer facilities are available throughout the medical campus, including 3 floors of the library, in some hallways, and student lounge. WiFi is available everywhere around the campus for easy access to the internet. Software called blackboard, which is entirely web-based, is used to distribute lecture slides, audio, syllabus, and to communicate with students.

15. What type of clinical sites—ambulatory, private preceptors, private hospitals, rural settings—are available or required for clerkships? Does this school allow for students to do rotations at other institutions or internationally?
Clinical exposure begins as early as first semester of first year throughout Georgetown University Hopspital and other medical facilities and private clinics througout DC and VA. During fourth year, students can choose to rotate in other medical institutions nationally and internationally. There is a international program department within medical school to facilitate students with international rotations and funding.

16. Is a car necessary for clinical rotations? Is parking a problem?
A car is not necessary, but it is very practical. Realistically, you will probably want one.

Financial Aid
17. What is the current tuition and fees? Is this expected to increase yearly? If so, at what rate?

The tuition for 2007 - 2008 academic year is $39,957.

18. Are there stable levels of federal financial aid and substantial amounts of university/medical school endowment aid available to students?
Yes. The support is very stable and will cover the student budget.

19. Are there students who have an "unmet need" factor in their budget? If so, where do these students come up with the extra funds?

20. Are spouses and dependents/children covered in a student's budget?


21. Is someone available to assist students with budgeting and financial planning?
We have a financial aid department that will hold seminars, send emails and be available in office virtually any time you need to walk you through the process.

22. Does this school provide guidance to its students, and to its graduates/alumni, on debt management?
Yes, again, the financial aid office will handle all of this for its students.

Student Involvement
23. What medical school committees (e.g., curriculum committee) have student representation?

The Student Medical Education Committee (SMEC) works with faculties to listen to students' suggestions and make changes to the curriculum. This can range from introducing new courses, changing exam dates, post-exam gripes.

24. Are students involved in (required or voluntary) community service?
There are 20 hours of community service as a graduation requirement at Georgetown. However, many students are involved above and beyond this minimum. There is a student-run clinic, the HOYA clinic that serves uninsured patients in Southeast DC. It is entirely voluntary and many students participate.

25. How active is the Student Council/Government? Are there other student organizations?
The student council is very active and involved. It has all the standard positions that you would expect and you will hold elections for your class within the first couple months of your first year.

Georgetown is very supportive of student organizations. There are many in place that you can join, but if you want to create one, just say the word and you're set. In fact, first year students this year created a club, the Seinfeld Medical Interest Group (SMIG) where they met once a month for lunch and watched medically related Seinfeld episodes. They also did blood drives, etc.

Policies
26. Is there an established protocol for dealing with student exposure to infectious diseases? Is disability insurance provided to cover this exposure?



27. Does this school provide, or does the student pay for, vaccinations against Hepatitis B or prophylactic AZT treatment in case of a needle-stick or accident?
Hepatitis B vaccination is mandatory before entering clerkship. It is covered through Student Health Insurance and one can receive vaccination through Student Health Center.

28. Is there a school Honor Code? Is there a grievance process/procedure? Are the students involved?
Detailed description of The Student Code of Professionalism can be found by clicking the link.

Residency
29. May I see a list of residency programs to which this school's recent graduates were accepted?
According to class of 2007 residency match, specialties that students matched the most were Anethesiology (12), Opthalmology (11), Radiology (10), and Orthopaedic Surgery (13).

Questions to Ask Yourself
30. Does this school have strengths in the type of medicine (primary versus specialized care, urban versus rural practice environment, academic medicine versus private practice) that I will want to practice?
Georgetown especially has a reputation for turning out Orthopedic Surgeons, among other surgical specialties, but realistically, we produce any type of physician pretty easily.

31. Would I be happy at this school (for at least the next four years)?

Additional comments added by Daeojkim.
 
Members don't see this ad :)
Curriculum
1. Are there any special programs for which this medical school is noted?
OU prides itself on having a more humanistic approach to medicine and does have classes like Spirituality and Medicine and Literature and Medicine. We also generally show up in the USNews primary care rankings, albeit somewhere near the bottom. Overall, though, OU doesn't have a clear emphasis, which I think is a positive. If you want to do rural primary care, great. If you want to specialize in a big city, that's tool, too.
2. Describe this school's curriculum in the pre-clinical and clinical years. Are there any innovations, like Problem-Based Learning?
I can only speak for the pre-clinical curriculum. It is very traditional and largely lecture-based. Grading is ABCDF. We do have some PBL and integrated learning classes, but they're not a big part of the current curriculum, which I've got to admit I like (as do most students). However, the school is discussing curriculum changes, so in a few years, you might see things like more PBL.
The current great thing about OU is that we have ample online resources, including class notes, lecture mp3s, powerpoints and online question banks, which means that you can do most of your studying without ever going to school. We have a long tradition of having "homeschoolers," and it seems like the school is not going to make a move towards requiring attendance in the near future.
3. Are there opportunities for students to design, conduct, and publish their own research?
I'm sure there are, but I don't know about them -- I haven't been involved in research yet.
4. Is there a note-taking service? If so, is it university-run or student-run?
Yes, and it's student run. You don't have to participate if you don't want to. To participate, you pay a fee (about $170 for paper copies over 2 years) and have to take notes for about 3 hours of lecture/semester. You get fined if your notes are flawed or late.
5. Is there flexibility in the coursework (the number of electives) and the timing of the courses (accelerating, decelerating, and time off) during the pre-clinical and clinical years?
As far as I know, not a lot. In the preclinical years, if you get thrown off track, you normally lose the whole year. I've never heard of anybody taking classes in different sequences. I am not at the clinical stage yet and can't compare programs, but it seems like the 4th year is mainly all electives.
6. Has this medical school, or any of its clinical departments, been on probation or had its accreditation revoked?
Not as far as I know.
7. How do students from this medical school perform on the National Board Examinations? How does the school assist students who do not pass?
If someone else could correct me that would be great, but I believe failing the USMLE results in you being pulled from rotations. I'm not sure how students are assisted.
It appears that our board performance is about average. I was told by some students on my interview that board scores seem to be bimodal with a clump of students with high school and a clump with lower yet passing scores. I have no clue how accurate that statement was.

Evaluations
8. How are students evaluated academically? How are clinical evaluations performed?
In the preclinical years, we have ABCDF grading with anything below a C (70%) requiring remediation. There is not a curve, and in my class it seems like the grades are mainly divided between As and Bs. The ABCDF grading holds up the clinical years, but supposedly getting a C is considered to be the equivalent to failing. Again, I'm not there yet, so I don't really know much beyond rumors. :)
9. Is there a formal mechanism in place for students to evaluate their professors and attending physicians? What changes have been made recently as a result of this feedback?
We do have anonymous online course reviews. Some professors seem really receptive to feedback and do make changes. With others, it seems like they only listen to the positive feedback. I don't know how much the administrative is involved in looking at course reviews.

Counseling/Student Support
10. What kind of academic, personal, financial, and career counseling is available to students? Are these services also offered to their spouses and dependents/children?
The school is very upfront about having available personal counseling for students, which I think includes family counseling. I don't know if family members can individually seek counseling, though.
Financial counseling seems to consist mainly of a few sessions throughout the year discussing debt, etc. Like at most schools, I think you largely need to figure that stuff our yourself.
I don't know personally about academic counseling. From what I've heard, professors are pretty receptive to talking to students about academic problems. The school is committed to having you graduate.
I don't know about career counseling because I'm not at that stage yet.
11. Is there a mentor/advisor system? Who are the advisors—faculty members, other students, or both?
First year students are assigned a second year mentor. We do not have faculty advisors.
12. How diverse is the student body? Are there support services or organizations for ethnic minorities and women?
Not very. Classes for the past few years have been stuck at around 40% female, and the school doesn't seem to have much traction changing that. To defend the school, it seems like the problem comes from the applicant pool and not from any discrimination in admissions. Being a conservative, southern state, it's possible most women are deterred from applying to medical school early on. Apparently the school is concerned about this and trying to find a solution.
Being Oklahoma, most of the minority students are Native American, but you probably wouldn't notice that they were a minority. My class had 3 African American students, all of which were female. We do have a decent number of Asian students. The only active organization I know of for minorities is the Asian Student Association. We also have a Women in Medicine group.

Facilities
13. Tell me about the library and extracurricular facilities (i.e., housing and athletic/recreational facilities).
I don't spend much time in the library, but it seems fine if a little small. We have free access to a nice yet small gym and can pay to join a fancy gym on campus. The Student Union is only a few years old and pretty nice. There are intramural fields, a volleyball court and a basketball court.
14. Are there computer facilities available to students? Are they integrated into the curriculum/learning?
There appear to be ample computer labs, but students are "required" to own their own computer (meaning financial aid gives you money to buy one). As mentioned above, we have lots of online resources for coursework.
15. What type of clinical sites—ambulatory, private preceptors, private hospitals, rural settings—are available or required for clerkships? Does this school allow for students to do rotations at other institutions or internationally?
Most of the rotations appear to be at large hospitals but some are ambulatory. In Tulsa, you'll be at private hospitals, and it supposedly has more of a community medicine focus. One rural clerkship is required your 4th year. You can do rotations at other places, and I believe international rotations are a possibility.
16. Is a car necessary for clinical rotations? Is parking a problem?
Yes, a car is always necessary in Oklahoma. :) I don't think parking is a problem for the Oklahoma City students. I doubt it's an issue in Tulsa, but I don't know for sure. Oklahoma is one of those states with lots of land, so parking is usually free and ample.

Financial Aid
17. What is the current tuition and fees? Is this expected to increase yearly? If so, at what rate?
Instate is around $18k, and out of state is $42k. I'm sure like all schools it will increase somewhat. I don't know the rate. It has not increased notably this past year.
18. Are there stable levels of federal financial aid and substantial amounts of university/medical school endowment aid available to students?
Most students seem to fund their schooling through federal financial aid. The school does have lots of scholarships and grants, but it's rare to meet someone who is getting most of their tuition covered through these scholarships or grants -- most seem to be for a few thousand dollars. As a state school, the overall debt level is low.
19. Are there students who have an "unmet need" factor in their budget? If so, where do these students come up with the extra funds?
Not normally. Considering the cost of living in Oklahoma, the school is pretty generous with aid. We are budgeted $17k for 9 months of school, and this increases proportionally for 3rd and 4th year year round schooling. We also get a few thousand dollars each year for books and supplies, etc. From my understanding, the financial aid office is also willing to increase your budget if you have a special need for more money.
I'll note that I am not supporting a family on my aid, and I don't have specifics about how that would work out. I know you can get extra money for day care expenses through getting your financial aid budget increased.
20. Are spouses and dependents/children covered in a student's budget?
No, but you can get money easily for day care expenses. From my understanding, the federal government does not allow OU or any other school to include dependents in the budget.
21. Is someone available to assist students with budgeting and financial planning?
I don't know.
22. Does this school provide guidance to its students, and to its graduates/alumni, on debt management?
We have some debt management seminars. Since the debt level of our students is fairly low because most students are instate, it doesn't seem to be a big emphasis.

Student Involvement
23. What medical school committees (e.g., curriculum committee) have student representation?
Students are going to be on the new curriculum committee. I believe students are also involved in the academic appeal committee.
24. Are students involved in (required or voluntary) community service?
Each class elects a community service representative. Ours has set up volunteer activities for children with cancer and work at a public health clinic. Our class is working on setting up its own clinic, and the Tulsa clinical students have a well established free clinic.
25. How active is the Student Council/Government? Are there other student organizations?
There are several student organizations. Student Council really depends on the quality of the people elected.

Policies
26. Is there an established protocol for dealing with student exposure to infectious diseases? Is disability insurance provided to cover this exposure?
We do have optional needle stick insurance that is pretty cheap (maybe $50/year). I believe there's a protocol, but I'm not at that point yet.
27. Does this school provide, or does the student pay for, vaccinations against Hepatitis B or prophylactic AZT treatment in case of a needle-stick or accident?
I believe you have to buy the needle-stick insurance.
28. Is there a school Honor Code? Is there a grievance process/procedure? Are the students involved?
Yes, and I believe students are involved. I don't know the specifics.

Residency
29. May I see a list of residency programs to which this school's recent graduates were accepted?
Click here to read a post I made on this subject in another forum. Match List for 2007

Questions to Ask Yourself
30. Does this school have strengths in the type of medicine (primary versus specialized care, urban versus rural practice environment, academic medicine versus private practice) that I will want to practice?
31. Would I be happy at this school (for at least the next four years)?
 
Curriculum
1. Are there any special programs for which this medical school is noted?
I do not believe that there are any special programs unique to this school.

2. Describe this school's curriculum in the pre-clinical and clinical years. Are there any innovations, like Problem-Based Learning?
We use a combination of PBL and traditional lecture format. Twice a week we meet in groups of 8 or so with an MD or PhD tutorial leader and
we go through a case from beginning to end and develop learning issues that will be answered in the next session. The traditional lectures are
usually only in the mornings from 8-12 with other things such as clinical skills, ethics and professionalism, or labs in the afternoons.
We learn subjects by blocks such as Human Structure and Development for 10 weeks, followed by Infection and Immunity for 6 weeks, Genetics and
Neoplasia for another 6 weeks, then Neurosciences for 10 weeks, and finally Cardio/Pulm/Renal for 12 weeks. Over summer we do a rural
preceptorship with a FP for 10 weeks. 2nd year has Endocrionology, Human Repro and Sexuality, GI, and then we are given a dedicated USMLE
study time of 8 weeks, followed by a research block of one month to work on a research project of our choice.
Third year is comprised of 6 8-week rotations of FP, IM, Peds, Surgery, OB/Gyn, Neuro/Psych. Fourth year has four required electives with four
of our choice all 4 weeks long.

3. Are there opportunities for students to design, conduct, and publish their own research?
Yes, as mentioned above, there is a required research block of one month in which to work on a research project of your chioce that you can
either choose to publish or not. You work with an MD or PhD mentor throughout.

4. Is there a note-taking service? If so, is it university-run or student-run?
There is not a note-taking service in medical school as far as I know, but there was in undergrad through the university.

5. Is there flexibility in the coursework (the number of electives) and the timing of the courses (accelerating, decelerating, and time off)
during the pre-clinical and clinical years?

You are allowed 6 total years to complete the MD degree meaning that you can take a year off to do a pathology fellowship or a dedicated research
fellowship through the NIH or other organization. Or, you could take a year off for personal reasons. There is no option for accelerating/decelerating.

6. Has this medical school, or any of its clinical departments, been on probation or had its accreditation revoked?
Not to my knowledge.

7. How do students from this medical school perform on the National Board Examinations? How does the school assist students who do not pass?
This year, Class of 2009 had a 100% pass rate for the USMLE with an average score of a 219. This is the highest it has ever been.... Every other year the pass rate has been in the high 80's or low 90's percentile. If you do not pass Step 1 you can float a rotation and study for that period of 8 weeks and retry as you have time fourth year to make it up. There are no designated help groups for those who do not pass.

Evaluations
8. How are students evaluated academically? How are clinical evaluations performed?
During the first two years we are graded on an OGSU system basically corresponding to an A, B, C, and Unsatisfactory with the same number
groupings (90-100= O). Clinical years are similiar with the clinical evaluation equalling 50% of the grade, 25% is the shelf, and the other 25% is made up of rotation specific things such as presentation, quizzes, etc..

9. Is there a formal mechanism in place for students to evaluate their professors and attending physicians? What changes have been made
recently as a result of this feedback?

We are required to fill out evaluations after every block and for attending specifically for every rotation. They are very big on evaluating everything at this school and are constantly trying to improve.

Counseling/Student Support
10. What kind of academic, personal, financial, and career counseling is available to students? Are these services also offered to their spouses
and dependents/children?
They do have academic counselling for students who need extra help that is given by other students who did well in that area. There are a couple options for personal couselling provided through nominated students and via the Dean of Studnents. There is a Financial Aid
officer designated to the SOM that can help with any needs in that area.

11. Is there a mentor/advisor system? Who are the advisors—faculty members, other students, or both?
There is a mentor system made up of a designated physician you can contact for advice, as well as a student run system made up of nominated/elected students in each class.

12. How diverse is the student body? Are there support services or organizations for ethnic minorities and women?
The student body is very diverse, as UNM is in the top 5 for medical school diversity if I remember right. There is a slight majority of females:males. There is support service for minorities in medicine as well as the office of diversity and they are willing to help anyone anytime.

Facilities
13. Tell me about the library and extracurricular facilities (i.e., housing and athletic/recreational facilities).
There is no designated dorm facilities for students which is pretty poor as freshman undergrads getting preference over the grad students. On
main campus across the street they do have a huge athletic facility that can be used for free during certain hours.

14. Are there computer facilities available to students? Are they integrated into the curriculum/learning?
We are encouraged to purchase a laptop at the beginning of school, but there are many, many computers that can be used by students free of
charge in the library and other designated areas around main campus.

15. What type of clinical sites—ambulatory, private preceptors, private hospitals, rural settings—are available or required for clerkships? Does
this school allow for students to do rotations at other institutions or internationally?

We are about 100 feet from New Mexico's only Level 1 trauma center and is a teaching facility. There are also two other hospitals- Lovelace
and Presbyterian that students and residents can do rotations at. We are encouraged to do rural rotations and required after first year.
There are options to do away rotations fourth year as well as options for international rotations.

16. Is a car necessary for clinical rotations? Is parking a problem?
Yes, you will need a car for some rotations. My FP rotation was a good 12 miles away from my house and could be as far as 35 miles for some.
Parking is not really a problem as there are shuttle systems and several close parking lots for students.

Financial Aid
17. What is the current tuition and fees? Is this expected to increase yearly? If so, at what rate?
The current annual tuition is about 16,000/year for residents of NM and approx. 37,000 for non-residents of NM and this has increased some every
year, but only by about 2-3% a year. On top of that is a couple thousand in lab, curriculum, etc.. fees every year. This does not
include housing, food, or computer, etc...

18. Are there stable levels of federal financial aid and substantial amounts of university/medical school endowment aid available to students?
There is as much financial aid available as you could ever need. There is not a large supply of scholarships or grants available to students
that are white, but there are several available for minorities.

19. Are there students who have an "unmet need" factor in their budget? If so, where do these students come up with the extra funds?
You are allowed to get enough to support your entire family if needed.

20. Are spouses and dependents/children covered in a student's budget?
They can be if needed. One person I know gets enough to support his entire family of wife and two kids.

21. Is someone available to assist students with budgeting and financial planning?
I am not sure- never had to use them.

22. Does this school provide guidance to its students, and to its graduates/alumni, on debt management?
To a certain extent. There is a person who comes in on the first few days to offer services on these issues.


Student Involvement
23. What medical school committees (e.g., curriculum committee) have student representation?
Everything under the sun including the AMA and AMSA. There are also student organizations for everything from ethics to religion and medicine, to minorities, to women in medicine, and international interest groups.

24. Are students involved in (required or voluntary) community service?
We do have to do a community project both during our rural rotations and FP in third year.

25. How active is the Student Council/Government? Are there other student organizations?
We do have active student chapters in the AMA that is supported and nominated to national positions every year.

Policies

26. Is there an established protocol for dealing with student exposure to infectious diseases? Is disability insurance provided to cover this exposure?
We do have mandatory coverage including something called specifically needle-stick insurance for this purpose that is something like 70 bucks a year and it will cover these areas.

27. Does this school provide, or does the student pay for, vaccinations against Hepatitis B or prophylactic AZT treatment in case of a needle-stick or accident?
You are required to pay for your own immunizations that insurance will not cover, but the disablility insurance mentioned above would cover things like AZT treatment in needle stick injuries.

28. Is there a school Honor Code? Is there a grievance process/procedure? Are the students involved?
Yes, there is a strict honor code and a board that oversees things such as cheating, and other ethical problems and they decide upon punishments or correction of problems. There are students involved in this and are elected every year.

Residency
29. May I see a list of residency programs to which this school's recent graduates were accepted?
Yes, here is a weblink to the list from this year- http://hsc.unm.edu/som/oss/pdf/2007_Match_List.pdf

Questions to Ask Yourself
30. Does this school have strengths in the type of medicine (primary versus specialized care, urban versus rural practice environment, academic medicine versus private practice) that I will want to practice?
Yes, last year we were ranked in the top 5 for rural medicine nationally and they focus more on primary care. However, I am more interested in a
surgical field such as Ortho and there are plenty of opportunitiies for this. I feel that the better you are at delivering primary care, the better you will be as a specialist.

31. Would I be happy at this school (for at least the next four years)?
I am happy with my school selection thus far.
 
Curriculum
1. Are there any special programs for which this medical school is noted?
GW has a number of optional "track programs." Students can choose from a number of disciplines - Community/Urban Health; Disaster Preparedness; Global Health; Health Policy; Integrative Medicine; Research; Medical Eduation. Students can choose one track and have to fulfill certain requirements: complete an 8-week long project in their discipline between year 1 & 2; attend a few lectures during the year; complete a short project during their 4th year. Students can attend lectures for tracks they are not members of in addition to their own track lectures. Participation is optional.
GW also has a pretty well-know MD/MPH option. It used to be 4 years but recently switched to 5 years. Also, GW has a nationally-respected clinical skills center. Most of the 6th floor of GW's hospital is comprised of simulation exam rooms and simulated patients which are used by students during their entire 4 years.

2. Describe this school's curriculum in the pre-clinical and clinical years. Are there any innovations, like Problem-Based Learning?

pre-clin: traditional lecture set-up; exams are in blocks (3 per semester for 1st year, variable for 2nd); practice of medicine course throughout all 4 years; students work with patients and learn h&p skills beginning in year 1; pbl is a very small component during 1st year (2 hours per week) & increases somewhat during 2nd year

3. Are there opportunities for students to design, conduct, and publish their own research?

Yes, students can take part in the research track. There is also a university program where mentors at gw and children's national create research projects for students which are all funded by summer grants.

4. Is there a note-taking service? If so, is it university-run or student-run?

Yes. The service is run by the medical center, but students take the notes. The service is free to students and note-takers are paid per lecture.

5. Is there flexibility in the coursework (the number of electives) and the timing of the courses (accelerating, decelerating, and time off) during the pre-clinical and clinical years?

Not much during pre-clinical years - students are able to take public health courses at no additional charge (I believe) and there are a few elective courses (at night) such as medical humanities and a new health policy course that just started.

6. Has this medical school, or any of its clinical departments, been on probation or had its accreditation revoked?

7. How do students from this medical school perform on the National Board Examinations? How does the school assist students who do not pass?

GW board scores are usually near the national average. The pass rate has increased in the past few years and I believe our dean told us that it is now around 96%.

Evaluations
8. How are students evaluated academically? How are clinical evaluations performed?

pre-clin: There is an H/P/CP(conditional pass)/F grading system. Grades are NOT based on a curve. Most couse directors set the honors mark at around 90%. So, if half the class finishes with above 90%, all of those people will achieve honors. For practice of medicine, there is both a subjective and objective grade component. For objective, grades are based on written exams, computer exams (evidence based medicine exercises, lit searches, etc), and patient-based exams (history & physical exam skills with standardized patients). The subjective component is comprised of evaluations from physicians and librarians from problem based learning & doctor-patient-society and your clinical apprenticeship preceptor (a physician at gw or in the community).

9. Is there a formal mechanism in place for students to evaluate their professors and attending physicians? What changes have been made recently as a result of this feedback?

Yes, there is an anonymous on-line feedback system for each lecturer in each class we take at the end of each semester. The recent changes I know of have been changes in lecture rooms & getting a professor to agree to print out all of the notesets for us (a huge help).

Counseling/Student Support
10. What kind of academic, personal, financial, and career counseling is available to students? Are these services also offered to their spouses and dependents/children?

The deans' office is very receptive to students in need of academic & career counseling. Student tutors are available for students in academic need. The deans also host a few career lectures each year. The financial aid office is separate from the main university and is available solely for the medical center students.

11. Is there a mentor/advisor system? Who are the advisors—faculty members, other students, or both?

First year students are paired with a second year who acts as a "big sib." I believe 3rd/4th years are paired with faculty advisors for their clinical years.

12. How diverse is the student body? Are there support services or organizations for ethnic minorities and women?

The student body at GW Med is very diverse in terms of ethnicities, academic backgrounds, career backgrounds, etc. It's an important aim for GW to pick a really well-rounded and diverse group of students each year.

Facilities
13. Tell me about the library and extracurricular facilities (i.e., housing and athletic/recreational facilities).

The library is small but it's quiet and there are many places to study. There are some great AV rooms and a great computer lab in the library. The library also has a really extensive on-line resources catalogue which students are taught to use by library staff. The library's on-line materials can be accessed anywhere on campus (there is abundant WiFi) as well as off campus with a VPN. Student housing is very sparse - most live off campus. The medical center is located on the main campus so medical students have access to the gym, student center, etc.

14. Are there computer facilities available to students? Are they integrated into the curriculum/learning?

Yes - see answer #13.

15. What type of clinical sites—ambulatory, private preceptors, private hospitals, rural settings—are available or required for clerkships? Does this school allow for students to do rotations at other institutions or internationally?

Students work with their individual preceptors at GW's Medical faculty associates building or at private offices or hospitals throughout the DC metro area. Students rotate through GW Hospital, Washington Hospital Center, Children's National Medical Center, Holy Cross Hospital, INOVA Fairfax Hospital, to name a few. Several students choose to do away electives internationally.

16. Is a car necessary for clinical rotations? Is parking a problem?

Yes, a car is required for certain rotations.

Financial Aid
17. What is the current tuition and fees? Is this expected to increase yearly? If so, at what rate?

The tuition for 1st year students for 2007-2008 is listed as $44,555.

18. Are there stable levels of federal financial aid and substantial amounts of university/medical school endowment aid available to students?

Yes. And, yes, there is university-funded aid available.

19. Are there students who have an "unmet need" factor in their budget? If so, where do these students come up with the extra funds?

20. Are spouses and dependents/children covered in a student's budget?

21. Is someone available to assist students with budgeting and financial planning?

Yes, because there is a separate financial aid office for the medical center, there is excellent and abundant advising available for students.

22. Does this school provide guidance to its students, and to its graduates/alumni, on debt management?

Student Involvement
23. What medical school committees (e.g., curriculum committee) have student representation?

Honor Council, Medical Center Student Council, Admissions Committee, Student Curriculum Representatives

24. Are students involved in (required or voluntary) community service?

Community Service is not mandatory but the majority of students at GW choose to become involved. There is a structured community service program called ISCOPES that many students are involved in - through the program groups of students visit a chosen service site on a regular basis. GW, in conjunction with a resources center called Bread for the City, is starting a student run clinic that will begin functioning in fall of 2007.

25. How active is the Student Council/Government? Are there other student organizations?

Student Council seems to be fairly active and we are frequently updated on their activities & progress. There are a number of other organizations from various medical specialty clubs to sports-interest clubs and an a capella group.

Policies
26. Is there an established protocol for dealing with student exposure to infectious diseases? Is disability insurance provided to cover this exposure?

Yes, the protocol is dispensed to students and we are required to have it with us at clinical sites.

27. Does this school provide, or does the student pay for, vaccinations against Hepatitis B or prophylactic AZT treatment in case of a needle-stick or accident?

28. Is there a school Honor Code? Is there a grievance process/procedure? Are the students involved?

Yes, there is an honor code which is signed at the white coat ceremony (during orientation) and it is strictly enforced. Violations of the honor code are handled by an honor council and students do participate in that process.

Residency
29. May I see a list of residency programs to which this school's recent graduates were accepted?

This is a nearly complete listing of the most recent match list:
anesthesiology (12); dermatology (2); emergency med (10); family med (5); surgery (9); internal med (31); med-peds (2); neurosurgery (1); obgyn (7); ophtho (4); ortho surgery (6); otolaryngology (5); pathology (1); pediatrics (14); child neuro (1); pm&r (1); plastic surgery (1); psychiatry (4); radiology (3); urology (3)

Questions to Ask Yourself
30. Does this school have strengths in the type of medicine (primary versus specialized care, urban versus rural practice environment, academic medicine versus private practice) that I will want to practice?

Traditionally, many GW students chose to enter primary care fields - many are interested in international health as well as health care for the underpriviledged. There is not as strong of an emphasis on academic medicine, although plenty of students are interested in that route as well. GW students generally match into all or nearly all of the various specialties.

31. Would I be happy at this school (for at least the next four years)?

I am extremely happy I chose GW and I know most of classmates feel the same way.
 
Medical College of Virginia


Curriculum

1. Are there any special programs for which this medical school is noted?
The first 2 years the students get clinical exposure through our Foundations of clinical medicine course where we learn physical exams and basic clinical skills while practicing on standardized patients or on real patients at our preceptor's office (a PCP) In addition our board review is taught by Dr. Linda Costanzo, who for all those who have studied for the boards is the physiology guru. The board review is personalized and very comprehensive which is very nice

2. Describe this school's curriculum in the pre-clinical and clinical years. Are there any innovations, like Problem-Based Learning?
First year is broken down into discrete basic sciences (eg biochem, phys, neuro, histo). We generally have 1-3 courses at a time and they are in blocks.
Second year it is an organ systems based approach
Third year and fourth year are spend on the wards.
Recently PBL has begun to be incorporated into the curriculum but it is a very minimal part.

3. Are there opportunities for students to design, conduct, and publish their own research?
There are many, many opportunities for research at MCV

4. Is there a note-taking service? If so, is it university-run or student-run?
No, but the syllabi are note sets from each professor. All of the info needed is found in the syllabi. There is no information on the tests that is not found in the syllabi. So in all honesty at MCV/VCU no note taking service is needed since the curriculum office ensures that all required information is in the syllabus.


5. Is there flexibility in the coursework (the number of electives) and the timing of the courses (accelerating, decelerating, and time off) during the pre-clinical and clinical years?
There is minimal flexibility in the timing of courses but we can take different electives during pre-clinical and clinical years

6. Has this medical school, or any of its clinical departments, been on probation or had its accreditation revoked?
No, not to my knowledge

7. How do students from this medical school perform on the National Board Examinations? How does the school assist students who do not pass?
Generally there is a 94-99% pass rate on the boards with an average score falling between the mid 210s and low 220s. I am unsure of how the school helps students who do not pass but I am sure that there is assistance. I do know that those who had problems passing step 1 are given tutors and remediation so that they pass step 2. In addition, with the numerous standardized patient encounters through the first 2 years, each student is trained to know how to write a patient encounter note and how to deal with the standardized patients during the Step 2 CS exam.


Evaluations

8. How are students evaluated academically? How are clinical evaluations performed?
Grades are given based on an uncurved scale.
>92- Honors
86-92- HP
~68-86 P
65 +/- 1standard deviation is a Marginal Pass (ie not a passing grade)
65 minus 1SD- Fail

9. Is there a formal mechanism in place for students to evaluate their professors and attending physicians? What changes have been made recently as a result of this feedback?
After every course we have online evaluations to evaluate the course, the syllabus, individual professors and individual parts of the syllabus.


Counseling/Student Support

10. What kind of academic, personal, financial, and career counseling is available to students? Are these services also offered to their spouses and dependents/children?
There are tutors, financial aid counselors and career counseling as well as personal counceling for both student and spouses/significant others. All of this is free of charge.

11. Is there a mentor/advisor system? Who are the advisors—faculty members, other students, or both?
Yes. Each student is paired with a physician advisor and we meet in a group setting every few weeks. In addition each first year is given an M2 buddy to show them the ropes and tell them what to look out for.

12. How diverse is the student body? Are there support services or organizations for ethnic minorities and women?
Not sure of the exact numbers but it is quite diverse. In my class women out number men. There are orgnaizations for women in medicine and extensive support services for ethnic minorities


Facilities

13. Tell me about the library and extracurricular facilities (i.e., housing and athletic/recreational facilities).
There are 2 libraries available with good hours. In addition there are numerous 24 hour study areas which require key card for access (for additional security). CBIL is our computer based learning facility which houses syllabi, board review books, free printing/copying and numerous computers. There are also numerous computer-based training programs that we must complete throughout the first 2 years.

While some students do live in university housing, most have their own apartments.

Rec facilities- there are 3 gyms, 3 pools and numerous basketball courts, and nice astroturf fields easily accessible for students. IM sports are very common as well. There are student commons with ping-pong, billiards etc

14. Are there computer facilities available to students? Are they integrated into the curriculum/learning?
Yes and yes. All syllabi are available online and there is a significant amt of computer based learning. Our histology course is completely digital and there is a significant computer based component to each course.

15. What type of clinical sites—ambulatory, private preceptors, private hospitals, rural settings—are available or required for clerkships? Does this school allow for students to do rotations at other institutions or internationally?
In pre clinical years we are paired with private preceptors. In clinical years most people opt to stay at MCV hospitals and about 24 opt to go to INOVA in fairfax (northern Va). In addition there is a V.A. hospital that people round in.

Some choose to do away rotations in rural settings as well as internationally. We have arrangements with schools in South Africa, Spain and India I believe.

16. Is a car necessary for clinical rotations? Is parking a problem?
A Car is necessary for both pre-clinical years to get to your preceptor's office and for clinical years as you may need to go to the VA hospital.


Financial Aid

17. What is the current tuition and fees? Is this expected to increase yearly? If so, at what rate?
As of now instate is ~26K and OOS is around 39K

18. Are there stable levels of federal financial aid and substantial amounts of university/medical school endowment aid available to students?
Yes

19. Are there students who have an "unmet need" factor in their budget? If so, where do these students come up with the extra funds?
I have no idea.

20. Are spouses and dependents/children covered in a student's budget?
Dont Know

21. Is someone available to assist students with budgeting and financial planning?
Yes, there is a financial aid office that is available to all students and is quite helpful. They put on numerous budget/planning seminars for the students.

22. Does this school provide guidance to its students, and to its graduates/alumni, on debt management?
Yes


Student Involvement

23. What medical school committees (e.g., curriculum committee) have student representation?
In pretty much any decision involving the medical school the students are involved. We have curriculum reps from each class which is the driving force that changes our curriculum. Students choose and run the tour guides service and in addition each class has 4th years sit on the admissions committee

24. Are students involved in (required or voluntary) community service?
Volunteer service is not required but many students volunteer through various activities especially through free clinics (fan free, commonwealth clinic etc). In addition many first years travel to Honduras to help the underserved in the summer between M1 and M2 years.

25. How active is the Student Council/Government? Are there other student organizations?
The student gov't is very active. The other student organizations are too numerous to list.


Policies

26. Is there an established protocol for dealing with student exposure to infectious diseases? Is disability insurance provided to cover this exposure?
Yes

27. Does this school provide, or does the student pay for, vaccinations against Hepatitis B or prophylactic AZT treatment in case of a needle-stick or accident?
I am pretty sure the school pays for this but not sure.

28. Is there a school Honor Code? Is there a grievance process/procedure? Are the students involved?
Yes, Yes, there are student reps from each class.


Residency

29. May I see a list of residency programs to which this school's recent graduates were accepted?
www.medschool.vcu.edu/alumni/documents/Match07M-IVs.doc


Questions to Ask Yourself

30. Does this school have strengths in the type of medicine (primary versus specialized care, urban versus rural practice environment, academic medicine versus private practice) that I will want to practice?
The school maintains a very diverse teaching hospital so that any type of medicine that you would want to participate in be it rural, urban, trauma, academic med, surgery, geriatrics you will have ample access to. The clinical exposure is by far MCV's strongest attribute with a very hands on learning environment.

MCV has a level 1 trauma center and is designated one of 60 Cancer centers in the US. With it's ICU tower it is the 4th largest teaching center in the US.

31. Would I be happy at this school (for at least the next four years)?
Most definitely
 
Curriculum

1. Are there any special programs for which this medical school is noted?
The Penn curriculum includes 1.5 years of basic science followed by a full year of clinics before taking the Step I exam. During the fourth year there is increased time for electives and a required "Scholarly Pursuit" research project. The University of Pennsylvania is noted for its research in almost all areas of medicine and science. There are also almost countless opportunities for programs like global health, public health, community medicine, etc…

2. Describe this school's curriculum in the pre-clinical and clinical years. Are there any innovations, like Problem-Based Learning? Module 1 is absolutely P/F, lasts 6 months, and includes the basic science curriculum (essentially first year at many schools) including anatomy, genetics, biochemistry, etc. Module 2 is H/P/F (there is talk of changing this to P/F at this time) and encompasses the next year. This is arranged in organ blocks where the pathology, pharmacology, physiology, and more of each organ system is reviewed. During both modules the time is split into about 25% required PBL (small group learning) and 75% lectures which are mostly not required. In the clinics, there are required classes each Friday afternoon, which guarantees you Friday evening off for most rotations. Otherwise, the usual basic clerkships apply with the national standardized exams. Grading in clinics is H/HP/P/F.

3. Are there opportunities for students to design, conduct, and publish their own research?
Of course! Some type of project is required of all non-combined degree students during 4th year. The MD/PhD program at Penn is the second largest in the nation and recruits nationally as well as from its medical school classes. The research environment is almost unparalleled here and the fact that it is all on one campus makes Penn unique. There are not multiple campuses like at other institutions, so if you want to do work with any type of science, engineering, law, business, nursing, dentistry, etc etc etc… It's all right here and chances are these are some of the top departments in their fields.

4. Is there a note-taking service? If so, is it university-run or student-run?
Notes and copies of the presentation slides for each lecture are distributed by the medical school. All lectures are video taped and are viewable from any computer with your student ID and password.

5. Is there flexibility in the coursework (the number of electives) and the timing of the courses (accelerating, decelerating, and time off) during the pre-clinical and clinical years? Yes. I don't know about accelerating, but for those who have problems, few students here need decelerated curricula and Penn prides itself on making itself accessible for students who have any issues. It is almost unheard of for a student to fail out or leave the school. Time off is fairly easy to negotiate if necessary.

6. Has this medical school, or any of its clinical departments, been on probation or had its accreditation revoked?
If they have I am unaware.

7. How do students from this medical school perform on the National Board Examinations? How does the school assist students who do not pass? When I started Penn was the #3 medical school for top average Step I scores. My understanding is that we are now #1. Suffice it to say that Penn students do extremely well on the boards. I have never heard of anyone not passing.


Evaluations

8. How are students evaluated academically? How are clinical evaluations performed?
In the pre-clinical years we receive grades in the P/F and H/P/F systems, mostly based off exams. Small group mentors provide evaluation forms about each student which are viewable in a green folder in the back of the medical school offices but are kept completely internal. In the clinics, it varies by rotation. On medicine, for example, students must have a certain number of evaluation cards filled out by their residents and attendings. On other services residents and attendings simply provide feedback that is viewable at the end of the rotation. In house exams are given for the pre-clinical exams and national standardized exams are given for clinics and factor into the grades.

9. Is there a formal mechanism in place for students to evaluate their professors and attending physicians? What changes have been made recently as a result of this feedback?
Online feedback must be given for each lecture and in clinics for each attending and resident. Changes are constantly being made to the curriculum, some of which is student motivated/supported and some of which is not.


Counseling/Student Support

10. What kind of academic, personal, financial, and career counseling is available to students? Are these services also offered to their spouses and dependents/children?
In general we have a very young student body, and I have not personally seen any students over 30 (the oldest in my class), so I don't know what to say about family services. Counseling services are provided confidentially through the general University of Pennsylvania resources for these issues. Medical school ombudsmen are available in the case of medical school specific issues such as harassment.

11. Is there a mentor/advisor system? Who are the advisors—faculty members, other students, or both?
Yes. You are given a faculty mentor and are required to meet with them every 6 months. As you complete clinics you are assigned a mentor in your area of interest with whom you can meet and gain insight into your possible or chosen specialty.

12. How diverse is the student body? Are there support services or organizations for ethnic minorities and women?
The student body at Penn is primarily from middle-upper to upper class backgrounds or children of recent immigrants, so it is mostly, though not entirely, the populations that fit into those categories. Again, for ethnic minorities, LGBT, etc, some services are provided through the medical school, but the large undergraduate campus also includes many services for these groups. Women make up greater than 60% of the medical student body. Penn is situated very close to the center of the diverse city of Philadelphia, the 5th largest city in the USA.


Facilities

13. Tell me about the library and extracurricular facilities (i.e., housing and athletic/recreational facilities).
Penn Med students may use any of the Penn campus libraries, of which there are many and they are a short walk from the medical school. The medical school library itself is small, bland, and a little depressing. Nearby libraries are larger, brighter, and some are architecturally stunning. Few students live in on-campus housing as it is more expensive than comparable housing around Penn. Housing within walking distance is generally affordable and safe. The main Penn gym is 3 blocks from the medical school and is large and recently built. However, membership must be purchased separately and is a bit over $200/year.

14. Are there computer facilities available to students? Are they integrated into the curriculum/learning?
Yes, the small group rooms are always open for students to use the computers in those rooms. The student lounge and library also have computers. There are many electronic resources that you can take advantage of, including supplemental course material and the videotaped lectures.

15. What type of clinical sites—ambulatory, private preceptors, private hospitals, rural settings—are available or required for clerkships? Does this school allow for students to do rotations at other institutions or internationally?
All types of clinical sites are available for students. A majority of rotations are done in the Penn health system, which mostly is set in Philadelphia, but some students elect to or are forced to do rotations in affiliated hospitals or practices as far away as York, Allentown, or southern NJ. Electives may be done in any setting and/or at other institutions, often for credit. There are partnerships with many institutions around the world as well as global health opportunities in several countries around the world.

16. Is a car necessary for clinical rotations? Is parking a problem?
It depends on the rotation, but usually not. Public transportation in and around Philadelphia will usually suffice. If a car is required it becomes an issue because most students do not have parking near their apartments and parking may be a problem at some sites. Also, financial aid is not provided to help you pay for the required transportation to some sites. This only impacts some students and not for long periods of time when it does (say one rotation or one part of a rotation), but it is a longstanding issue.


Financial Aid

17. What is the current tuition and fees? Is this expected to increase yearly? If so, at what rate?
Tuition is currently $39,648, fees are $3,225. This increases yearly at a rate of a few percent per year (depends on the year).

18. Are there stable levels of federal financial aid and substantial amounts of university/medical school endowment aid available to students?
Yes. Penn prides itself on having an average indebtedness tens of thousands of dollars lower than the national averages for private medical schools.

19. Are there students who have an "unmet need" factor in their budget? If so, where do these students come up with the extra funds?
I'm unsure.

20. Are spouses and dependents/children covered in a student's budget?
I don't think so.

21. Is someone available to assist students with budgeting and financial planning?
Yes, the department of financial aid in the medical school is available to help students with these questions.

22. Does this school provide guidance to its students, and to its graduates/alumni, on debt management?
I don't know.


Student Involvement

23. What medical school committees (e.g., curriculum committee) have student representation?
Almost all. Student feedback is always solicited by the medical school administration.

24. Are students involved in (required or voluntary) community service?
There are two clinics within West Philadelphia often staffed by Penn students and physicians that provide volunteer community service, but there are many other opportunities throughout the city of Philadelphia. Students do have a required part of the curriculum where they are assigned a patient with a chronic illness and must meet with them and ask them questions periodically and then hand in assignments based on these meetings. This program receives mixed reviews.

25. How active is the Student Council/Government? Are there other student organizations?
There are many active student organizations.


Policies

26. Is there an established protocol for dealing with student exposure to infectious diseases? Is disability insurance provided to cover this exposure?
I'm sure there is but I'm not familiar with it.

27. Does this school provide, or does the student pay for, vaccinations against Hepatitis B or prophylactic AZT treatment in case of a needle-stick or accident?
The student is required to show proof of hep B vaccination before beginning. I don't know about the case of AZT treatment.

28. Is there a school Honor Code? Is there a grievance process/procedure? Are the students involved?
Yes, we must sign the honor code after each exam. There are procedures for these issues, but it seems so rare that I don't know the details.


Residency

29. May I see a list of residency programs to which this school's recent graduates were accepted?
A 2007 list is here: http://forums.studentdoctor.net/showpost.php?p=4913601&postcount=230


Questions to Ask Yourself

30. Does this school have strengths in the type of medicine (primary versus specialized care, urban versus rural practice environment, academic medicine versus private practice) that I will want to practice?
I am interested in specialty medicine and research, and for me I think that's where Penn excels the most. You can get your medical school training for any type of medicine at any medical school in my opinion. What makes Penn special is its opportunities for doing more than just medicine, whether that's in basic science research, clinical science research, medical business or law, global/public health, or any other interactions that can enhance medicine through outside medicine interactions.

31. Would I be happy at this school (for at least the next four years)?
A wealth of further information is available at http://www.med.upenn.edu/
I don't know that everyone who goes to this school is happy, and I have at times been unhappy with certain parts of the school or the administration. Some students are unhappier than myself and some students love it here. I think I made the right decision because of Penn's opportunities in research and in the areas of medicine in which I'm interested.
 
Curriculum

1. Are there any special programs for which this medical school is noted?
The orthopedics department is fairly well-known and is expanding. There is also a brand new laboratory for endocrinology and diabetes research.

2. Describe this school's curriculum in the pre-clinical and clinical years. Are there any innovations, like Problem-Based Learning?
Pre-clinical years is block-scheduling based (benefits of this would be having a single or few subjects tested simulataneously and in series rather than in parallel). A problem-based learning course runs during the pre-clinical years on a weekly and pass-fail basis. Board pass rates have been 98-99% in the past few years, so the curriculum is solid. I haven't hit clinical years yet, but the new administration is definitely student-centered and is making some excellent changes based upon student feedback (adding an elective rotation during 3rd year to allow you to experience a field you may wish to apply to match for w/ residency).

3. Are there opportunities for students to design, conduct, and publish their own research?
Absolutely; there are many opportunities available and they are all presented in a centralized form for the first summer. Professors are also quite receptive to people who take the initiative and seek out their own opportunities.

4. Is there a note-taking service? If so, is it university-run or student-run?
The environment is quite collaborative here. Definitely more community-based than individual-based. Some years have established note taking services (student run), but ours just shared notes online. Also, the lectures are recorded and posted online in mp3 and podcast form.

5. Is there flexibility in the coursework (the number of electives) and the timing of the courses (accelerating, decelerating, and time off) during the pre-clinical and clinical years?
There is definite flexibility in timing; the school has a 5 year program and some combined degree programs. Electives are becoming more well-structured w/ the new administration as mentioned in #2.

6. Has this medical school, or any of its clinical departments, been on probation or had its accreditation revoked?
To my knowledge, no for the medical school. I don't know about the residency programs.

7. How do students from this medical school perform on the National Board Examinations? How does the school assist students who do not pass?
This school has one of the highest pass rates in the country, in the top 2% for pass rate. I think the average scores are around average. I have not taken them yet, and I don't know what's done to help those who do not pass. The school has never left people hanging as far as I've seen.


Evaluations

8. How are students evaluated academically? How are clinical evaluations performed?
Honors, High Pass, Pass, Fail for all classes except PBL (see #2). I don't know about clinicals.

9. Is there a formal mechanism in place for students to evaluate their professors and attending physicians? What changes have been made recently as a result of this feedback?
Absolutely. Every single lecture has feedback forms and the students are strongly encouraged to fill them out. Response to these forms obviously depends upon the professor. I would say constructive criticism is certainly taken to heart by most, and I've seen professors change notes and processes based upon feedback.


Counseling/Student Support

10. What kind of academic, personal, financial, and career counseling is available to students? Are these services also offered to their spouses and dependents/children?
Counseling is available for all areas, though I've not used any of them as yet. I've heard mixed reviews of the financial counseling, though that is partly because of the nature of money (and the nature of medical school debt). I don't know about families, but I know the perscription plans extend to family members (and they're quite reasonable).

11. Is there a mentor/advisor system? Who are the advisors—faculty members, other students, or both?
The system when I entered was a buddy system with a 2nd year student. The new system being implemented involves "families" of a few members of each class level that periodically meet (at their descretion, but with some structure built in so it should actually happen). This should allow advice from all levels. Faculty members are typically open to offering advice, but there is no formal faculty advising apart from research advisors if you choose to conduct research.

12. How diverse is the student body? Are there support services or organizations for ethnic minorities and women?
The student body is fairly diverse. We get a lot of people from out of state, especially California. The number of minorities accepted into our class was the highest yet, and I think they're trying to continue that trend. I am personally friends with people of all three monotheistic religions, and people from several continents.


Facilities

13. Tell me about the library and extracurricular facilities (i.e., housing and athletic/recreational facilities).
The library is pretty nice and is used by a fair-sized niche of students. I don't use it because I live close and prefer to study at home. The rec. center is central on campus and is fairly old but functional. The merge with UT opened the use of the main-campus rec center which is supposed to be really nice. Plus you get a membership to YMCA/JCC w/ your tuition (the rec center on campus is a YMCA) and you can use that to get into any of them.

14. Are there computer facilities available to students? Are they integrated into the curriculum/learning?
There are several labs which are due to be upgraded soon. The whole campus is wireless. There are mobile computing labs (carts of laptops) used for viewing microbiology slides, etc.

15. What type of clinical sites—ambulatory, private preceptors, private hospitals, rural settings—are available or required for clerkships? Does this school allow for students to do rotations at other institutions or internationally?
All types are available, and the school has tons of partnerships w/ institutions around the area (from Ann Arbor and Detroit down to Columbus) for AHEC sites (away rotations). I've not done clinicals yet but I've heard that the clerkship coordination is exemplary.

16. Is a car necessary for clinical rotations? Is parking a problem?
I've heard clinical rotations are about 30% away on average (you can typically choose where to do your rotations). A car will be necessary for those at the away hospitals besides the University Med Center. Parking is absolutely NOT a problem. The school, while in the city of Toledo, is in a fairly nice and open part of town (suburban, almost) and parking is free.


Financial Aid

17. What is the current tuition and fees? Is this expected to increase yearly? If so, at what rate?
I don't know the exact figure for the upcoming year. It's easy enough to find on the website. However, UT had a 0% increase in tuition this year (a point of pride on the part of the new administration). We'll see what happens next year.

18. Are there stable levels of federal financial aid and substantial amounts of university/medical school endowment aid available to students?
I assume the levels are stable; there are endowments and scholarships available. There are also work-study programs and stipends offered for summer work.

19. Are there students who have an "unmet need" factor in their budget? If so, where do these students come up with the extra funds?
I'm not sure, but I assume that students who come from out of state and pay that tuition during the first year (you can gain Ohio residency after a year) will have unmet need if they have no funds to start with. Private loans would be the source for extra funds, and there are companies w/ partnerships here.

20. Are spouses and dependents/children covered in a student's budget?

I'm not sure on this one.

21. Is someone available to assist students with budgeting and financial planning?
There are financial aid counselors available. See #10.

22. Does this school provide guidance to its students, and to its graduates/alumni, on debt management?
I am not 100% sure, but I think so (haven't gotten to that point yet).


Student Involvement

23. What medical school committees (e.g., curriculum committee) have student representation?
The new administration is heavily student-oriented and wants student participation on many levels. I don't know the names of the specific committees, but there are definitely members of the student body involved in executive steering committees.

24. Are students involved in (required or voluntary) community service?
Yes, mostly voluntary with some required in one class. There's a student-run clinic that is extremely popular and that affords students the opportunity to gain clinical experience as soon as they feel like going.

25. How active is the Student Council/Government? Are there other student organizations?
I would call them moderately to heavily active. There are MANY student organizations. There's an organization fair at the beginning of the year that exposes the new students to these groups.


Policies

26. Is there an established protocol for dealing with student exposure to infectious diseases? Is disability insurance provided to cover this exposure?
I do not know (haven't dealt with this).

27. Does this school provide, or does the student pay for, vaccinations against Hepatitis B or prophylactic AZT treatment in case of a needle-stick or accident?
The student health center administers these vaccinations and all of the required titers, etc. The cost is put on the student account.

28. Is there a school Honor Code? Is there a grievance process/procedure? Are the students involved?
There is a newly-established honor code and Honor Code Committee. It was mandated by the executive office and it was entirely student-written and formed. Grievances do go through a process; those related with the honor code are essentially student run w/ a faculty advisor.


Residency

29. May I see a list of residency programs to which this school's recent graduates were accepted?
Absolutely. There should be a summary sheet available on the school's website. The office of admissions has a big map on the wall with the names of students and their match placed on the map. It's quite an impressive display. Our recent matches have been very successful, with at or very close to 100% match after scramble.


Questions to Ask Yourself

30. Does this school have strengths in the type of medicine (primary versus specialized care, urban versus rural practice environment, academic medicine versus private practice) that I will want to practice?
The school is definitely clinically-oriented. Research will start gaining more strength now that we merged with UT. There are several combined programs already underway (biomedical engineering, for example). There is good exposure to urban and rural medicine. I imagine there is a bit better preparation for private practice, given the heavy need for big research names when it comes to academic medicine (though that door certainly isn't closed for you if you come here).

31. Would I be happy at this school (for at least the next four years)?
I love the school. The city of Toledo takes some getting used to, and the weather can be a bit irksome. If medical school wasn't so time-consuming, I think those things would matter more. I'm definitely happy with my choice; I chose the school over all of the other Ohio schools, WashU, and Duke because of its atmosphere, student-centeredness, and clinical focus (and obviously because of its proven success with training clinicians). Good luck in your search! Find what best fits you and go with it!
 
Curriculum
1. Are there any special programs for which this medical school is noted?
We have MD/PhD and MD/MPH programs. Our MD/PhD students match really well and have lots of options for research at both school and the MD Anderson Cancer Center.

2. Describe this school's curriculum in the pre-clinical and clinical years. Are there any innovations, like Problem-Based Learning?
The 1st 2 years are basic sciences. MS1 is a more traditional curriculum-- for example, you'll take histo, gross, embryo, and biochem your 1st semester, but they'll all roughly cover the same organ system at the same time. You'll look at GI histology when you learn GI development and while you're dissecting the abdomen.
During MS2, you'll have path, pharm, and PBL. This year is essentially organ-based. PBL meets 3 times a week (2 hr sessions) and will present cases which parallel what's happening in class.

3. Are there opportunities for students to design, conduct, and publish their own research?
Yes-- in addition to the MD/PhD program, many students do a research project in the summer between MS1 and MS2.

In addition, many students do research months in fourth year.

4. Is there a note-taking service? If so, is it university-run or student-run?
Nearly all lectures are video taped, and you can watch the lectures later t double speed. Also, the powerpoint slides are posted, too. The video taping is managed by students.

5. Is there flexibility in the coursework (the number of electives) and the timing of the courses (accelerating, decelerating, and time off) during the pre-clinical and clinical years?
There is the Alternative Pathway (AP) program which allows you to spread out MS1 into 2 years. This allows you to take 2 classes at a time instead of 4. Students have a variety of reasons for pursuing this pathway-- new babies, illness, finishing a masters degree, etc.

Third and fourth years have a great deal of flexibility; extra time off may be used for studying for USMLE, pregnancy/maternity leave, family situations, illness, etc. Student Affairs is very lenient in these matters.

6. Has this medical school, or any of its clinical departments, been on probation or had its accreditation revoked?
Not that I know of.

Department of Otolaryngology: on probation
All other departments I checked on ACGME's accredited program search were continued or continued full accreditation. I did not check all subspecialties of internal medicine or pediatrics, however.

7. How do students from this medical school perform on the National Board Examinations? How does the school assist students who do not pass?
I've heard our average step 1 score last year was just above the national average, and we're still waiting on scores from my class. I haven't heard of many people not passing..but I suppose they could delay their 3rd year for another couple months and take it again.

The school will allow students extra time to prepare if they need it. The school also provides all students with Kaplan Qbank for USMLE Step 1.


Evaluations
8. How are students evaluated academically? How are clinical evaluations performed?
Grades for all 4 years are Honors (H), High Pass (HP), Pass (P), Marginal Performance (MP), and Fail (F). The cut-off values for each category are set before the class starts, which fosters a friendly environment-- basically, it doesn't hurt my score if I help my friends do well. Students frequently send out emails to the class with charts or helpful notes.
During the clinic years, each rotation grade is based partly on an evaluation and partly on the shelf exam. The exact percentage of each varies with the rotation.

9. Is there a formal mechanism in place for students to evaluate their professors and attending physicians? What changes have been made recently as a result of this feedback?
Yes-- after each test in the basic science years, we fill out evaluation forms. We have students who function as class representatives for each class, so you could also just talk to one of them and they will pass on the info to the course director. During 3rd year, you evaluate your attending as well. I haven't been around long enough to know what changes have been made.


Counseling/Student Support
10. What kind of academic, personal, financial, and career counseling is available to students? Are these services also offered to their spouses and dependents/children?
UT Counseling and Worklife Services covers all of this: http://www.uthouston.edu/worklife/ . Not sure what services are offered to spouses.

Services are generally offered to spouses.

11. Is there a mentor/advisor system? Who are the advisors—faculty members, other students, or both?
Each new student is assigned to a Master Advisory group with 2 faculty members, 4 MS2 students, and a dozen of their peers. Each group meets a few times a semester, and the faculty members have access to your grades and can help you if you're in trouble.

12. How diverse is the student body? Are there support services or organizations for ethnic minorities and women?
I don't have exact numbers for these, but it seems about 50% women and many non-trad students who had other careers before med school. There are student organizations for asians (APAMSA), latinos (NNLAMS), afr.amer (SNMA), and women (AMWA).


Facilities
13. Tell me about the library and extracurricular facilities (i.e., housing and athletic/recreational facilities).
UT Housing is ~1.5 miles down the road, and the really nice rec center is there too. The Learning Resource Center (LRC) is at school with study rooms, computers, and most of the textbooks you'll need. The Jones Library is the medical library next to school.

14. Are there computer facilities available to students? Are they integrated into the curriculum/learning?
Yes, in the LRC. You'll be using PubMed, UpToDate, etc for PBL and during 3rd years.

15. What type of clinical sites—ambulatory, private preceptors, private hospitals, rural settings—are available or required for clerkships? Does this school allow for students to do rotations at other institutions or internationally?
During the 3rd year required rotations, sites include Hermann hospital, MD Anderson, St.Joseph's, LBJ (county hospital outside the med center). Some rotations have more options than this-- part of pediatrics can be done at Child Protective Services, a week of Ob/Gyn in jail (I've heard this is interesting), the county psych hospital for psych, and family practice is done in clinics around the city (as well as Hermann). I'm leaving out lots-- basically, there are many different hospitals and clinics available for your training, both within and outside of the med center. Away rotations are allowed during the 4th year.

There are fourth year rotations available in Mexico, South America, and China. Away rotations may be in the US or abroad. For rural experience, students may create their own rotations or go through a preceptor matching program; otherwise, rural experience is hard to come by.

16. Is a car necessary for clinical rotations? Is parking a problem?
A car wouldn't be necessary to get to the med center hospitals (if you live along the rail or near a shuttle/bus line). There is only 1 month of 3rd year where you're required to be at LBJ hospital (a medicine month)-- some people carpool if they don't have a car, and there's a bus that goes there (metro line #1).

I would say not having a car is difficult in the third year. It would probably take an hour to get to LBJ hospital from the medical center area by bus. Parking at LBJ is $10/month and the drive is ~20 minutes. For medical center rotations, students are offered garage parking for $70/month, which is especially handy when the rotation requires you to arrive before the school or city bus runs. For pediatrics and family medicine, some of the outlying clinics may be up to 55 minutes away. If you have no vehicle, it is usually possible to make arrangements with another student to carpool, or to arrange for a closer rotation. However, it is not easy; Houston is a car culture kind of place.


Financial Aid
(I didn't answer questions 18-22 b/c the MD/PhD program pays my stipend, so I don't really know anything about financial aid.)

17. What is the current tuition and fees? Is this expected to increase yearly? If so, at what rate?
It's around $10,000 total. I'm not sure how much it increases year to year.

Tuition plus fees is around $11,000 a year now (Texas resident), and increases at the behest of the state of Texas. For a non-resident, the tuition amount could be tripled. Comparatively, tuition is low, especially for a Texas resident, and living expenses in Houston are low.

18. Are there stable levels of federal financial aid and substantial amounts of university/medical school endowment aid available to students?
It has been harder for me to get loans this year; I was required to pay $3000 myself by my FAFSA calculation. I still received all $8500 subsidized Stafford loan and $3000 less unsubsidized Stafford.

19. Are there students who have an "unmet need" factor in their budget? If so, where do these students come up with the extra funds?
I don't know about this. The school tries to budget well for living expenses, books, test fees, and interviewing. Should extra needs arise, there is an emergency loan program, where a student can borrow up to $2000 with 5% interest toward tuition or with zero interest for living expenses. These loans must be repaid within 90 days. If this is still not enough, the student would have to seek out a private source of student loans.

20. Are spouses and dependents/children covered in a student's budget?
If you have dependents, you can request more financial aid.

21. Is someone available to assist students with budgeting and financial planning?
Yes. Financial Aid can assist with this; also, UT Worklife has a financial planning counselor.

22. Does this school provide guidance to its students, and to its graduates/alumni, on debt management?
Yes. During fourth year, there will be a class on "Transitioning to Residency" which will include financial planning and student loan repayment strategies.



Student Involvement
23. What medical school committees (e.g., curriculum committee) have student representation?
The curriculum committee itself has student reps, and each course has reps too.

24. Are students involved in (required or voluntary) community service?
Yes-- in addition to service projects put together by the classes, we share a free, student-run homeless clinic with Baylor. Students volunteer there every Sunday morning.

25. How active is the Student Council/Government? Are there other student organizations?
There is student government and officers in each class. Lots and lots of student organizations for specialty interest groups, faith-based groups, etc. Here's a link: http://med.uth.tmc.edu/students-current/SCAIP/student-organization-leaders.htm


Policies
26. Is there an established protocol for dealing with student exposure to infectious diseases? Is disability insurance provided to cover this exposure?
Haven't run across this, so that's a good question!

There is a protocol. A 24-hour Needlestick Hotline is available to students; all costs of treatment for needlestick or body fluid exposure are covered. Disability insurance is not required.

27. Does this school provide, or does the student pay for, vaccinations against Hepatitis B or prophylactic AZT treatment in case of a needle-stick or accident?
I know there is a needle-stick 24-hour hotline number, but I don't know any more than that.

The school does pay for treatment after an exposure if necessary. Hepatitis B vaccine is a requirement for attendance at the school, as are yearly PPDs (or chest x-rays if PPD is positive), which are NOT free. Yearly influenza vaccines are available for free or cheap at the student health clinic. In other words, prophylaxis is not free, but if you are exposed the school will pay.

28. Is there a school Honor Code? Is there a grievance process/procedure? Are the students involved?
Yes, we have an Honor Code, and students are involved in the SCAIP committee (it's the honor code/ethics committee, forgot what it stands for). There are certainly grievances for those who break the Honor Code-- the committee meets (with faculty, I guess) and comes to a decision.


Residency
29. May I see a list of residency programs to which this school's recent graduates were accepted?
Here's last year's match list: http://med.uth.tmc.edu/students-current/2007-match-list.htm . It's posted online every year.


Questions to Ask Yourself
30. Does this school have strengths in the type of medicine (primary versus specialized care, urban versus rural practice environment, academic medicine versus private practice) that I will want to practice?
With so many hospitals and clinics available for training, you can get a very good exposure to primary and specialized care, academic settings and private practice. We don't really have much rural medicine, since Houston is a big city. But many student do rural preceptorships in the summer after 1st year.

The school tries to place an emphasis on primary care, but a good deal of students match into specialties. Another thing to consider is medical Spanish; it's not a requirement, but it will always come in handy.

31. Would I be happy at this school (for at least the next four years)?
Sure-- most of that depends on you and whether or not you tend to be a happy person, regardless of where you are. I enjoyed my 1st 2 years, and 3rd year hasn't been bad so far!

I've been pretty happy with this school for the past three years; I don't see why my fourth year is going to be any different.


Answers in red - thanks to enanareina
 
Curriculum
1. Are there any special programs for which this medical school is noted?
I don't know of any.

2. Describe this school's curriculum in the pre-clinical and clinical years. Are there any innovations, like Problem-Based Learning?
Our curriculum is systems-based, and divided into blocks that range from 4 to 10 weeks long. Problem-Based Learning is incorporated into the schedule, but accounts for a small percentage of the total class time - we are given cases every few weeks that relate to what we're currently studying, and are asked to discuss/find the answers to certain questions. Within the first few weeks of first year the "Introduction to Clinical Medicine" (ICM) course begins, in which every 6 students is assigned an instructor with whom they spend one morning every week in the hospital taking histories and physicals. We also have a "Professionalism in the Practice of Medicine" (PPM) course that meets weekly to discuss ethical, cultural, etc issues in medicine.

I'm not sure how unique this is, but Pathology is a large part of each of our systems blocks - I've heard that quite a few schools don't teach Pathology in first year. Also, Gross Anatomy is spread out throughout all of 1st year (with a few weeks in 2nd year to cover the reproductive tract), and syncs up very nicely with the systems/subjects we cover.

3. Are there opportunities for students to design, conduct, and publish their own research?
Yes. It's incredibly easy to do your own research - all you need to do is approach a faculty member with your idea and have them agree to be your mentor. Also, there are 3 curriculum tracts you can choose from - one of which is Academic Medicine (the other 2 are Clinical and Community/Underserved). The tracts don't lock you into anything, but if you choose the Academic tract there are several meetings discussing how to do research, how to write an IRB proposal, guest lecturers come and often ask if anyone wants to be a part of their research projects, and the adviser is more than happy to hook anyone up with a mentor. For those who want even more research, the school offers an optional 5th year.

4. Is there a note-taking service? If so, is it university-run or student-run?
There is no note-taking service because we have access to the powerpoint presentations for each lecture, and, more importantly, are given a lecture handout for each lecture. The school policy is that they cannot test us on anything that is not included in the lecture handout - thus if the professor rambles on about their own research, or goes off on some tangent, we're not at all responsible for it. Also, each lecture handout has learning objectives, to let us know what topics the professors believe are most important.

5. Is there flexibility in the coursework (the number of electives) and the timing of the courses (accelerating, decelerating, and time off) during the pre-clinical and clinical years?
I'm not entirely sure. I know that we're free to take time off if we want to, in between any of the years. You can also stop partway through a year, and start up again the next year. You'd have to be insane to want to accelerate. I haven't heard of anyone decelerating either, but it might be an option. In 3rd year we have clerkships in Psych, IM, Ob/Gyn, Peds, FM, Surgery (actually, 2 surgery clerkships), and one 6-week flex block that we can do whatever we want with. Fourth year has tons of time for electives, vacation, TAing, etc.

6. Has this medical school, or any of its clinical departments, been on probation or had its accreditation revoked?
I don't think so.

7. How do students from this medical school perform on the National Board Examinations? How does the school assist students who do not pass?
The average board score for the class of 2007 was 228. The average for the class of 2008 was 232. I'm not sure what the school does for people who don't pass (and I hope I never find out).


Evaluations
8. How are students evaluated academically? How are clinical evaluations performed?
For the first two years grading is Pass/Fail. For 3rd and 4th years it's Honors/High Pass/Pass/Fail. I don't know how clinical evaluations are performed.

9. Is there a formal mechanism in place for students to evaluate their professors and attending physicians? What changes have been made recently as a result of this feedback?
Yes. At the end of each block we have to fill out online evaluations that include every professor. I don't know of any changes that have been made due to these, but I've heard we (the students) have some weight.


Counseling/Student Support
10. What kind of academic, personal, financial, and career counseling is available to students? Are these services also offered to their spouses and dependents/children?
Pretty much everyone in every department has told us they are always available to talk, and they seem pretty approachable. We are required to meet with a career counselor at the beginning of 3rd year, I think, but we're free to see them beforehand, too. There's also a student-run group of peer counselors who are available 24/7. I don't know the policy regarding non-students, but I can't imagine any of our administrators turning someone away. All in all it's a very supportive environment.

11. Is there a mentor/advisor system? Who are the advisors—faculty members, other students, or both?
See above. Also, we are free to ask any faculty we want to to be our adviser. As far as more formal programs, there's a Big Sib system in place where each incoming 1st year is assigned a 2nd year "big sib". The 2nd year is available for answering any questions the 1st year has, offers advice, and gives the 1st year gifts occasionally. Plus, two organizations (American Medical Women's Association, and the Salerni Collegium) match students up with mentors/advisers.

12. How diverse is the student body? Are there support services or organizations for ethnic minorities and women?
The majority of the students are definitely young and white, but not overwhelmingly so. There are support services for minorities, women, and LGBT students.


Facilities
13. Tell me about the library and extracurricular facilities (i.e., housing and athletic/recreational facilities).
The library is pretty small, but there always seem to be enough free places to study in it. I rarely go there, however, since the whole system is online and I can get everything I need from the comforts of my own apartment. There is a dormitory directly over our cafeteria and coffee shop, but it fills up fast - especially with 3rd years (it's directly across the street from the hospital). We don't have a gym! Or a pool. Students are working on changing that, but it won't happen for a few years. We're free to use the facilities on the main campus, but you'd have to drive over there or take a shuttle, since the campuses aren't connected.

14. Are there computer facilities available to students? Are they integrated into the curriculum/learning?
There is a computer lab down the hall from our MDL rooms (where we each have our own desk/area and mailbox), and there are 2 computers and a printer in each MDL. There are also a lot of computers we can use in the library, but printing is free in the MDLs and computer lab and not in the library, so I'd stick to the former.

15. What type of clinical sites—ambulatory, private preceptors, private hospitals, rural settings—are available or required for clerkships? Does this school allow for students to do rotations at other institutions or internationally?
I just finished 1st year, so I don't know much about the clinical years. I do know that we can do rotations at LA County Hospital, University Hospital, and Childrens Hospital LA, and can do away rotations at other institutions. I think we can do international rotations.

16. Is a car necessary for clinical rotations? Is parking a problem?
Yes, a car is necessary. Really, if you live in LA, you have to have a car no matter what. Parking isn't a problem on-campus, and University Hospital and LA County Hospital are both on campus. I'm not sure what parking's like at the other sites.


Financial Aid
17. What is the current tuition and fees? Is this expected to increase yearly? If so, at what rate?
$65,000 for the first 3 years, and $60,000 for 4th year. It will increase slightly each year.

18. Are there stable levels of federal financial aid and substantial amounts of university/medical school endowment aid available to students?
Every student gets a package totaling the tuition+fees minus EFC (estimated family contribution). The school gives out $10,000 interest-free loans, and $5,000 scholarships.

19. Are there students who have an "unmet need" factor in their budget? If so, where do these students come up with the extra funds?
No, see above. Also, the school is pretty flexible about approving additional loans. For example, I've got loans and grants covering the entire tuition+fee amount (yes, I'm going to be up to my eyes in debt), but I had to buy a car - which isn't included in the budget. Once I explained the situation, the financial aid office approved an additional loan to cover part of the cost of the car.

20. Are spouses and dependents/children covered in a student's budget?
I don't know.

21. Is someone available to assist students with budgeting and financial planning?
Yes, they have frequent seminars on financial planning/budgeting, and you're free to talk to the financial aid advisors whenever you want.

22. Does this school provide guidance to its students, and to its graduates/alumni, on debt management?
Yes, there are seminars on that, too.


Student Involvement
23. What medical school committees (e.g., curriculum committee) have student representation?
There are 2 representatives from each class that sit on the curriculum committee, 1 ICM rep, and 1 PPM rep. There are also students on the ethics committee. I can't think of any more committees, but those that exist typically have student representation.

24. Are students involved in (required or voluntary) community service?
Students are involved in community service, but it's voluntary.

25. How active is the Student Council/Government? Are there other student organizations?
Student Government seems pretty active. There are also a ton of clubs that are very active.


Policies
26. Is there an established protocol for dealing with student exposure to infectious diseases? Is disability insurance provided to cover this exposure?
Yes, we had to sit through talks on this during orientation. Yes, we have disability insurance.

27. Does this school provide, or does the student pay for, vaccinations against Hepatitis B or prophylactic AZT treatment in case of a needle-stick or accident?
I don't know.

28. Is there a school Honor Code? Is there a grievance process/procedure? Are the students involved?
Yes. We also went over this during orientation. Honor Code violations are handled by the ethics committee, which does have student representation.


Residency
29. May I see a list of residency programs to which this school's recent graduates were accepted?
2007 Match List
I can't verify the accuracy of this info, but it's better than nothing.


Questions to Ask Yourself
30. Does this school have strengths in the type of medicine (primary versus specialized care, urban versus rural practice environment, academic medicine versus private practice) that I will want to practice?
USC has extremely strong programs in Ophthalmology and Pediatrics, and as far as I know typically does pretty well in the other specialties. A lot of graduates go into specialized care (probably partly because of the huge debt we accrue). If you're looking to practice in an urban environment, I can't imagine a better place to train than Los Angeles County Hospital - this place, and what they allow medical students to do, is legendary.

31. Would I be happy at this school (for at least the next four years)?
I am very happy here. I think the curriculum is practically ideal, I LOVE ICM, and County is really pretty incredible. I'm not so enamored with LA, but I can handle it for 4 years.
 
Curriculum

1. Are there any special programs for which this medical school is noted?

Bascom Palmer Eye Insititute has been ranked the #1 ophthalmology program/hospital by USNews and World Report for the past 3 years. The Ryder Trauma center is one of the busiest in the country, all US army forward medical teams rotate through Ryder before going to Iraq and all medical students get to do 1 month of Trauma Surgery there during their 3rd year.

The school has two seperate medical campuses. The first is in Miami, which is associated with Jackson Memorial Hospital (the largest single site hospital in America) and the second is in Boca Raton (about 40 miles north) which is a more heavily PBL based curriculum. I am at the Miami Campus, and that is what I will speak about.

2. Describe this school's curriculum in the pre-clinical and clinical years. Are there any innovations, like Problem-Based Learning?

The first 2 years are broken down into organ modules. There are PBL blocks in year 2 to help prepare for the transition to the wards and USMLE prep. Years 3 and 4 are ward based with lectures and small groups throughout most rotations. Nearly every rotation in 3rd year has a shelf exam, many have orals. Year 4 is mostly electives and is strictly pass/fail.

We are a modern organ based curriculum. We have PBL for 3 weeks before Christmas and four weeks after regular classes end in the second year.

3. Are there opportunities for students to design, conduct, and publish their own research?

Miami really gives you an opportunity to do whatever you want. If you want to immerse yourself in research experience you have the opportunities and if you want to immerse yourself in clinical skills you can walk into JMH and Ryder whenever you'd like and just spend some time there seeing patients and working shifts in your free time as early as day 1.

There is very little formal aid. However, research opportunities are limitless, and everyone I know with an interest has been more than successful at finding quality opportunities. There is a huge research campus and most departments are VERY accomadating to students.

4. Is there a note-taking service? If so, is it university-run or student-run?

No note taking service. Instead video of every lecture in the first 2 years are digitally recorded and posted to the web that same day. If you'd like to see something again, you can just pull up the video.

No, but the powerpoints are posted online, which would make it really unneccessary.

5. Is there flexibility in the coursework (the number of electives) and the timing of the courses (accelerating, decelerating, and time off) during the pre-clinical and clinical years?

Students can always go to the deans and request a year off for research or anything else they become interested in during med school. The admins are very willing to help you if you need time off.

In the first two years, no. Thereafter, there is a lot of flexibility. Research years are accomadated, but certainly not required or expected. Fourth year is more flexible than third year. Students are assigned a lottery number and are allowed to pick their schedule for third year based on the luck of the draw. The numbers are then reversed for fourth year.

6. Has this medical school, or any of its clinical departments, been on probation or had its accreditation revoked?

Not sure, I think JMH received some citations in the past for minor things like not having enough computers in the resident call rooms and stuff but I dont believe UM/JMH has had any major problems.

No. A couple of the residency programs at Jackson did spend some time on probation, though this should end this year.

7. How do students from this medical school perform on the National Board Examinations? How does the school assist students who do not pass?

My class did about average on step 1 the year we took it. If anyone didnt pass they were given time off in 3rd year to retake the exam and provided with advice from the deans.

We do about average on Step I and well above average on Step II. There is a huge range of scores however.


Evaluations

8. How are students evaluated academically? How are clinical evaluations performed?

The first 2 years uses a numbers and pass/fail system combined. So for example, the school sets 70 for passing a course. If you got an 80, your transcript would show 80/P. ie, your actual grade and that you passed. In year 3 the evals are again number and pass/fail. Here the number is made up based on how your attendings and residents evaluate your performace, your grades on oral exams, shelf exams, quizzes, etc... are also factored in. In year 4 the grades are strictly Pass/Fail, no numbers.

Students are given a P/F along with a number grade. This is compared to the class average.

9. Is there a formal mechanism in place for students to evaluate their professors and attending physicians? What changes have been made recently as a result of this feedback?

Yes, every course and clinical rotation has evals at the end. The curriculum changes from year to year based on that feeback, ie anatomy is now all taught in one block rather than being split across the various organ modules.

There is almost too much feedback. We evaluate every class and every professor. We also give awards to the most popular professors. We even have to evaluate fellow students at times. The school changes the curriculum constantly, and some feedback appears to be taken into account.


Counseling/Student Support

10. What kind of academic, personal, financial, and career counseling is available to students? Are these services also offered to their spouses and dependents/children?

The med school offers counseling with psychiatrists for free for any student that needs to talk to someone, all those sessions are anonymous and nothing goes on a students file. Our financial aid office is wonderful and really has an open door policy for anyone who may have questions. Not sure if the services are offered to family.

All of these counseling services exist for the student, but they are not for the family.

11. Is there a mentor/advisor system? Who are the advisors—faculty members, other students, or both?

The school has this wonderful program called "Academic Societies". Every admitted student is placed into an academic society at random, there are 12 in total. It is in these societies that the students receive all their interview and physical exam training the first couple of years. Whats incredible is that the societies are led by upperclassmen, so you will have 3rd and 4th years there to help you. Makes for a very low stress environment and much easier to ask questions. Each society is also assigned faculty mentors so both upperclassmen and faculty are there for students. Its a really nice way of creating a true community amonst the various classes as you remain in the same society all 4 years and interact with society members ranging from M1's through M4's.

Students are seperated into academic societies, which include both student and faculty mentors. Student mentorship is highly encouraged and produces a variety of results. Mentorship varies within the clinical years.

12. How diverse is the student body? Are there support services or organizations for ethnic minorities and women?

It's Miami so there is definitely diversity. See this page for the profile of the class of 2010 if you want to see specific numbers:

http://www6.miami.edu/UMH/CDA/UMH_Main/0,1770,2600-1;14190-2;45488-3,00.html

Highly diverse. There is an Office of Minority Affairs. African americans are probably underrepresented overall, but latin americans being over represented. I think that the class is close to 50% women.


Facilities

13. Tell me about the library and extracurricular facilities (i.e., housing and athletic/recreational facilities).

The library is a 3 floor building across from the medical school that is being renovated soon. The medical school itself is housed in the Rosenstiel Medical Sciences Building, a facility where students have 24/7 access to including the computer lab. We just built a new gym across the street from the medical school last year that is state of the art, all students have access to it.

There is no student housing. There is a new gym at the medical campus with a panoramic view of downtown Miami from the 13th floor. It's very nice. The library is fine, but it closes overnight, which is a little bit annoying. All students have access to the UM undergraduate campus, which has good intramural sports programs and a better library. The undergraduate campus is a few really annoying miles south though.

14. Are there computer facilities available to students? Are they integrated into the curriculum/learning?

Yes, at the medical school you have the computer lab in Rosenstiel plus computers at the library. You also have access to all the facilities at the main campus in Coral Gables. All lectures are digitally recorded on video and posted to the web with their corresponding powertpoint so the school is very tech friendly.

Yes, Yes

15. What type of clinical sites—ambulatory, private preceptors, private hospitals, rural settings—are available or required for clerkships? Does this school allow for students to do rotations at other institutions or internationally?

Students are assigned clinical preceptors in the community their first month of med school and work with those preceptors starting just a couple weeks after enrolling. In years 3 and 4 most rotations take place either at JMH (public), the VA (military), or Mt.Sinai (private). On your family medicine rotation in year 3 you will work in a private medical office for 6 weeks and on your Generalist Primary Care rotation you will rotate at various public clinics throughout the County for a month. Students ARE allowed to do externships in year 4, either domestically in the uS or internationally.

There are so many answers to this, that I don't have time to write about them all. Every student has community preceptorships the first two years. Options in the third year GPC clerkship include a mobile pediatric clinic, a homeless shelter, a women's shelter, a middle school nursing office, and more. The closest thing we have to a rural rotation is a Florida Keys option for Family Medicine. The keys are actually quite underserved and can be cut off from the rest of the state.
International experience is widespread, with the majority of students going abroad at least once. I've gone twice. In the last two, a variety of groups put on international mission trips to Haiti, The Dominican Republic, Jamaica, Nicaragua, and India. We also sent students on clerkships all over Europe and S. America.


16. Is a car necessary for clinical rotations? Is parking a problem?

A car def makes things easier in Miami and I totally recommend having one. The majority of your rotations occur at JMH which is great so its not like you will be driving around every month going to new hospitals all the time and learning new computer systems, etc... Parking is not a problem, there are several garages on campus and students can buy a monthly parking pass.

It really does help. Parking is only a problem at the main campus, and this is only difficult at the beginning. After you've been assigned a spot, you have to pay $60/month to keep it.


Financial Aid

17. What is the current tuition and fees? Is this expected to increase yearly? If so, at what rate?

http://www.mededu.miami.edu/OSFA/tuition/index_html

Tuition has been held constant the pass 2 or 3 years and has not increased.

Just over $30k for in-state and nearly $40k for out of state. Most OOS students get some sort of scholarship aid that reduces their tution down to in-state levels. Tuition hasn't changed in three years. Tuition is rising rapidly everywhere else within the state, and the administration has hinted that we may soon be similar to our state schools in tuition rates.

18. Are there stable levels of federal financial aid and substantial amounts of university/medical school endowment aid available to students?

Yes, besides the loan programs 40% of all students receive some kind of scholarship or grant from the school.

Yes. I've received money from both.

19. Are there students who have an "unmet need" factor in their budget? If so, where do these students come up with the extra funds?

Not sure, I know that additional private loans can be taken out from the financial aid office up to the maximum amount of the estimated cost of attendance.

I don't know of any such student.

20. Are spouses and dependents/children covered in a student's budget?


Not sure but here is the financial aid office's webpage:

http://www.mededu.miami.edu/OSFA/index_html

No. Budget extensions can be requested, and I have had one granted. A couple of students have families, and everyone seems to figure it out. A working spouse helps.

21. Is someone available to assist students with budgeting and financial planning?

Not sure but I know that the financial aid office is very open to helping and talking to students.

There is an office. I've never really tried to use it.

22.Does this school provide guidance to its students, and to its graduates/alumni, on debt management?

Hmmm.... again, not sure.

Yes for students, but no for alumni.


Student Involvement
23. What medical school committees (e.g., curriculum committee) have student representation?

DOCS (department of community service), Academic societies, and a ton of other campus organizations. As for specific committees, I know that there are a couple of medical students on the admissions committee.

Every committee seems to have some representation


24. Are students involved in (required or voluntary) community service?

Nothing is required as of yet.

Almost all students participate in voluntary service (including local and international). There is no required service.

25. How active is the Student Council/Government? Are there other student organizations?

Very active and there are enough organizations for anyone who is interested to attain a leadership role.

Student government is mildly active. There are many other highly active organizations, including those that put on international trips or run the local free clinic.


Policies

26. Is there an established protocol for dealing with student exposure to infectious diseases? Is disability insurance provided to cover this exposure?

There are protocols in case of needle sticks or student exposure's, I'm not sure how in depth it all is though since I've never had to use it.

Yes and Yes

27. Does this school provide, or does the student pay for, vaccinations against Hepatitis B or prophylactic AZT treatment in case of a needle-stick or accident?

I know the Hep B vaccinations are free, again not sure about the needle stick accidents but we are all given a campus hot line to call in case that happens. Not sure where that goes from there.

It doesn't pay for Hep B vaccines (though they are required). Treatment for accidents is covered I believe.


28. Is there a school Honor Code? Is there a grievance process/procedure? Are the students involved?

There is an honor code and there is a grievance process. Everything is explained in the student handbook:

http://www.mededu.miami.edu/MedEd/d...docs/STUDENT_HANDBOOK_MASTER_2006-7_FINAL.pdf

Yes, yes, and yes. I've been fortunate to never run afoul of the code, so I'm not overly familiar with the exact procedures.

Residency
29. May I see a list of residency programs to which this school's recent graduates were accepted?

Classs of 2007 match results:

http://www6.miami.edu/UMH/CDA/UMH_Main/0,1770,2600-1;14190-2;53868-3,00.html

I don't have a list accessable. We have matched students recently into everything from in-state primary care programs to Neurosurg at Wash. U. or Orthapedics at San Francisco. About 1/2 the students will stay in S. Florida. We also send a lot of students sent to Emory in Atlanta and many of the NYC hospitals. A handful of students from every class are also from California, and many of them will match back in Cali.

Questions to Ask Yourself
30. Does this school have strengths in the type of medicine (primary versus specialized care, urban versus rural practice environment, academic medicine versus private practice) that I will want to practice?

Students get a tremendous exposure to all of those except rural.

Jackson Memorial Hospital is a crazy community hospital with all of the pros and cons that go along. The difference here at Miami, is that there are many academic specialty hopsitals attached. The school is really in a unique position to train a wide variety of practicioners. We have Bascolm Palmer Eye Hospital, which is the #1 eye hospital in America. We have the Miami Project to Cure Paralysis, where huge amounts of neurology and neurosurgery research takes place. We also have top programs in Otolaryngology, transplant surgery, and more. We also have a number of community clinics offering basic services to the uninsured and uninsurable. These include medicine, pediatric, and gynecological services. We also have one of the busiest trauma services in the country, with a stand-alone trauma hospital. A student interested in primary care will do well here. A student interested in opthamologic microsurgery research will also do well here.

The pathology is INCREDIBLY VARIED. I've seen things already that most people only read about. The population is highly diverse. There is a joke that a white anglo-saxon protestant is a minority on the wards (it's really not a joke). We have huge international populations from all of Latin America and the Carribean Islands, with exceptionally large groups from Cuba and Haiti.


31. Would I be happy at this school (for at least the next four years)?

I've been very happy here and am strongly considering staying here for residency. The clinical exposure and training is incredible.

I'm glad that I chose Miami


Red information thanks to D.G.
 
I will try to answer these Q’s to the best of my ability. I am still an M1 who just started 6 weeks ago. I am taking summer anatomy, but I have been talking to older med students. I will modify my answers or add to them over the next few months as I get to know the school better.

Curriculum
1. Are there any special programs for which this medical school is noted?
UW-SMPH is very well known for its research (so I would guess its MSTP program must be good). We have a good MD/MPH program, and sometimes you can get it paid for (yes a free MPH). The school also has a pre-med scholars program (accepts students out of high school into a 7 year program).

I am not sure if this qualifies as a “special program,” but UWSMPH offers anatomy over the summer for its students to get a jump start, and have an easy transition into med school (Your fall semester is 8 credits less than normal!)

I also known that UWSMPH is ranked high in primary care.

One very special program is accommodating students with disabilities; we currently have a BLIND MSTP student who has successfully completed his basic sciences and is working on his Ph.D.

2. Describe this school's curriculum in the pre-clinical and clinical years. Are there any innovations, like Problem-Based Learning?

Curriculum:

FIRST- AND SECOND-YEAR REQUIRED COURSES

YEAR 1
---------------------------------------------------------------------
Semester 1
-Patient, Doctor and Society I
-Integrated Medical Anatomy
-Biomolecular Chemistry
Semester 2
-Patient, Doctor and Society II
-Principles of Human Physiology
-Neuroscience
-Modular Courses*:
Medical Genetics (8 weeks)
Introduction to Psychiatry (10 weeks)
Population Med. and Epidemiology (8 wks)

YEAR 2
------------------------------------------------------------------
Semester 1
-Patient, Doctor and Society III
-Infection and Immunity I
-Pharmacology I
-Pathophysiology/Modular Courses:
General Pathology (3 weeks)
Hematology (4 weeks)
Cardiovascular System (4 weeks)
Neoplastic Diseases (3 weeks)
Respiratory System (4 weeks) Semester 2
-Patient, Doctor and Society IV
-Infection and Immunity II
-Pharmacology II
-Pathology of Organ Systems
(integrated throughout the year)
-Pathophysiology/Modular Courses:
Clinical Nutrition (6 weeks)
Psychiatry: Biopsychosocial and
Interpersonal Aspects (7 weeks)
Renal (4 weeks)
Gastrointestinal and Hepatology (5 weeks)
Endocrine System/Male/Female (5 weeks)

NON-DEGREE CREDIT ELECTIVE COURSES
-Complementary and Alternative Medicine
-Contemporary Issues in Health Care
-Epidemiology
-Fundamentals in International Health Care
-The Healer's Art
-Historical Perspectives in Medicine
-Medical Spanish
-Patient Advocacy
-Physicians as Health Advocates
-Health Care in Diverse Communities

*A full semester is 17 weeks. Modular courses are shorter courses taught concurrently with full semester courses.

-------------------------------------------------------------------------

THIRD- AND FOURTH-YEAR REQUIRED COURSES (Clinical Years)

Total credits (weeks) required for graduation................................80

THIRD YEAR: 46 MINIMUM CREDITS
--------------------------------------------------------------------
Credit distribution:
Credits Courses
40 Core Courses
+6 Required Courses and/or Electives*
Total Credits = 46
3rd-Year Core Courses
Medicine.............8
Surgery...............8
Primary Care.....8
Pediatrics...........6
Ob/Gyn................6
Psychiatry...........4
Total Credits = 40
*Electives:
Fourth-year courses that may be taken during the third year when prerequisites have been satisfied.

THIRD- AND FOURTH-YEAR CREDITS
------------------------------------------------------------------------
Required courses
These courses are required for graduation and may be taken any time during the third or fourth years.

3rd- & 4th-Year
Required Courses
Neuroscience.....6
Anesthesiology...2
Radiology.............2
Total Credits = 10

FOURTH YEAR: 30 MINIMUM CREDITS
--------------------------------------------------------------------------Direct patient care in the UW Clinical Campus:
18 credits of the total fourth-year course requirement must meet the Direct Patient Care standard and must also be spent in the UW Clinical Campus. That includes the clerkships at the La Crosse, Marshfield and Milwaukee sites, as well as the preceptorship. (Note: Research, extramurals and some individualized clerkships do not meet this requirement.)
It is recommended that the fourth-year electives consist of a mixture of experiences with no more than 12 credits (including extramurals and individualized clerkships) in any one discipline.
4th-Year Required Courses
Req. Medicine Sub-Intern.....4
Surgery.....................................4
Preceptorship..........................6
Electives (Direct patient
care in the UW
Clinical Campus)...................4
Total Credits = 18

Electives (UW, extramural, international, individualized clerkship, research**)........12
Total Credits = 30
**A maximum of 8 research credits and 12 extramural credits will be applied toward the 30 fourth-year credits required for graduation. Eight extramural credits may be international experiences. Only research begun after the second year will receive credit.
I am not sure about PB learning. However, based on my anatomy exams and what I hear from students, many exam questions are along the lines of “a patient comes into your office…..etc”

Regarding the curriculum, most prof’s have been there for 10+ years and really have the material organized. It is VERY clear what you need to study, and the prof’s are very friendly. My anatomy prof’s let us call them by their first names.

3. Are there opportunities for students to design, conduct, and publish their own research?
I am pretty sure there is. A fellow med student (taking anatomy this summer) attended UW-Madison as an undergrad and had the opportunity to publish and conduct research as an undergrad…. I would be surprised if Med students could NOT do the same.

4. Is there a note-taking service? If so, is it university-run or student-run?
There is NOT a note-taking service. However, ALL lectures are videotaped (and voice recorded obviously). Also, our course book is a very condensed and concise summary of the original reference. Not much (if anything) is taught in lecture that is not in this course book (about 10 pages per lecture). Also, the PowerPoint slides are available online before lecture (for those who like to take notes on the PPTs). I haven’t been taking ANY notes in lecture, and I have been doing just fine (again this is ONLY summer anatomy). Video taping is University-run.

5. Is there flexibility in the coursework (the number of electives) and the timing of the courses (accelerating, decelerating, and time off) during the pre-clinical and clinical years?
I am not too sure about this yet. Things that I DO know for sure:

1- You can easily defer for a year after being accepted.
2- You have the option of finishing your fist 2 years in 3 years instead (decelerated)….I am not sure of the details. There is a 5 year cap I believe.
3- If you take anatomy over the summer, I heard you have the option of taking a class instead (like one of your 2nd year courses) in the fall.
4- You can take a year off to get an MPH. I know people who did it after 4th year and some before that.

6. Has this medical school, or any of its clinical departments, been on probation or had its accreditation revoked?
NEVER!

7. How do students from this medical school perform on the National Board Examinations? How does the school assist students who do not pass?
I know that our average is above national average (I remember this from my interview back in Sept). I am not sure how the school helps those that do not pass the first time. However, given the number of staff members who do NOTHIG but make sure we do fine throughout our med school education, I am pretty sure there is enough help. I can add more later.

Evaluations
8. How are students evaluated academically? How are clinical evaluations performed?
It is a grade based system. A=4.0; AB=3.5; B=3.0 …..etc. It is very normal for the class average to be at an A or AB (according to older students). Failure in any given class is less than 1%.

I do not know much about clinical evaluations.

9. Is there a formal mechanism in place for students to evaluate their professors and attending physicians? What changes have been made recently as a result of this feedback?
I am not sure of this. However, I know that the prof’s actually want to teach, and so, are usually very open to comments and feedback. I also know that that there are evaluation forms outside the admissions office that you can fill anytime. Again, maybe at the end of this course I’ll know more about eval methods.

Counseling/Student Support
10. What kind of academic, personal, financial, and career counseling is available to students? Are these services also offered to their spouses and dependents/children?
I know that there is a great support system put in place. I have not used it yet, but during orientation, there was more than a half dozen staff personals for different counseling needs. One was for residency/ career planning, one for financial aid, and few for academic support, talking through personal problems confidentially….etc.

I do not have any dependents/ spouse so I can’t say much about that.

11. Is there a mentor/advisor system? Who are the advisors—faculty members, other students, or both?
I know that all students are broken down into “houses.” You stay in your “house” all 4 years. So, each house has an equal number of 1st, 2nd, 3rd, and 4th year students. This is a way to know older students that can give advice…etc.

I am not sure about faculty mentoring advising. I am sure I’ll know more in the fall.

12. How diverse is the student body? Are there support services or organizations for ethnic minorities and women?
Out of the 16 med students taking summer Gross, there is one African American, 2 Asians, 3 Indians, 1 Middle Eastern and 1 Latino student. The other half is white. In terms of gender, I’d say about half the students are women. There is an office of multicultural support/development put in place.

Facilities
13. Tell me about the library and extracurricular facilities (i.e., housing and athletic/recreational facilities).
The library and med school building are both new. There are plenty of rooms for students to study in throughout the building (not in the library). Not sure of the extracurricular facilities yet.

14. Are there computer facilities available to students? Are they integrated into the curriculum/learning?
There is a Do-It lab in the library that helps students with all their technological needs (PDA’s, computer Q’s…etc). We can also check out laptops for a couple of days at a time. I am not sure about integration into the curriculum (our curriculum is all on paper still).

15. What type of clinical sites—ambulatory, private preceptors, private hospitals, rural settings—are available or required for clerkships? Does this school allow for students to do rotations at other institutions or internationally?
Student are allowed to do rotations in other places as well as internationally. Most 3rd and 4th year clerkships are NOT in Madison city; they are scattered all over the state for better patient diversity. You will need a car, but the university has free housing at each site.

16. Is a car necessary for clinical rotations? Is parking a problem?
Yes. Parking is a huge problem in Madison city, but for clerkships outside of Madison it should not be so bad. The bus system in Madison is very organized and easy, so most people just take the bus, walk or bike.

Financial Aid
17. What is the current tuition and fees? Is this expected to increase yearly? If so, at what rate?
Tuition for instate students is a little less than 20 k/year. Out of state is near 35 K. It does NOT seem like tuition is about to increase.

18. Are there stable levels of federal financial aid and substantial amounts of university/medical school endowment aid available to students?
You can get enough loan money to pay tuition and live off of. There are very few scholarship and grant opportunities, and many are conditional (most work in family care in a rural area, for example).

19. Are there students who have an "unmet need" factor in their budget? If so, where do these students come up with the extra funds?
I think everybody gets enough money to meet there needs. I am not sure about students with dependents. I know the office of financial aid is very helpful in helping secure funds.

20. Are spouses and dependents/children covered in a student's budget?
I do not know. I don’t think they are though.

21. Is someone available to assist students with budgeting and financial planning?
Yes. I am not sure of the details.

22. Does this school provide guidance to its students, and to its graduates/alumni, on debt management?
Yes. I am not sure of the details

Student Involvement
23. What medical school committees (e.g., curriculum committee) have student representation?
I don’t know yet.

24. Are students involved in (required or voluntary) community service?
Most students are involved in volunteer work. MEDiC is the biggest volunteer organization, and has over 6 sites in Madison. There are many other organizations that provide services to populations in the Madison area that cannot afford any healthcare otherwise. Again, MEDiC is the biggest player.

25. How active is the Student Council/Government? Are there other student organizations?
There are plenty of student and interest groups/ organizations. The student council is very active. One of its greatest services is providing a host for interviewees. They also hold tournaments with other UW- professional schools.

Policies
26. Is there an established protocol for dealing with student exposure to infectious diseases? Is disability insurance provided to cover this exposure?
I don’t know.

27. Does this school provide, or does the student pay for, vaccinations against Hepatitis B or prophylactic AZT treatment in case of a needle-stick or accident?
Each student has to send immunization records to the school of medicine before matriculation. This includes titers for Hep B. I do know the protocol in the event of accidents.

28. Is there a school Honor Code? Is there a grievance process/procedure? Are the students involved?
There is a school honor code. At this point, my only exposure to it was that we sign our exams saying we have not cheated and we will not cheat because that breaks the school honor code and the trust put in us future physicians.

Residency
29. May I see a list of residency programs to which this school's recent graduates were accepted?
I will upload a complete list of the 2007 graduates before July 20 2007.

Questions to Ask Yourself
30. Does this school have strengths in the type of medicine (primary versus specialized care, urban versus rural practice environment, academic medicine versus private practice) that I will want to practice?
I know that they rank like 6th in the nation in Primary Care, but I also know that they have great residency (specialty) programs. I do not much of the details regarding the other items. However based on its MPH and strong research rep, I believe that academic medicine could be very strong.

31. Would I be happy at this school (for at least the next four years)?
I know that I am going to be happy over the next 4 years, and since I started, it is only getting better. I think YOU would be happy here.
 
Curriculum
1. Are there any special programs for which this medical school is noted?

Big on medical ethics, AMC is the home of the Alden March Bioethics Institute.

2. Describe this school's curriculum in the pre-clinical and clinical years. Are there any innovations, like Problem-Based Learning?
Clinical Skills and Healthcare & Society classes run throughout the themes in preclinical years. Conferences are also interspersed that focus on case studies.

3. Are there opportunities for students to design, conduct, and publish their own research?
From day one, students can become involved in a wide range of projects with instructors and graduate researchers. A large number of students spend their first summers on these projects.

4. Is there a note-taking service? If so, is it university-run or student-run?
Independent student-run note-taking service.

5. Is there flexibility in the coursework (the number of electives) and the timing of the courses (accelerating, decelerating, and time off) during the pre-clinical and clinical years?
Some, but not a large extent.

6. Has this medical school, or any of its clinical departments, been on probation or had its accreditation revoked?
Not to my knowledge.

7. How do students from this medical school perform on the National Board Examinations? How does the school assist students who do not pass?
Seems kind of a silly question, since the pass/fail rates for most Allopathic schools are nearly identical.

Evaluations
8. How are students evaluated academically? How are clinical evaluations performed?
Clinical evaluations are basically pass/fail. Academically, grading is a modified pass fail system comprising of EH, H, G, M, F, with about 80-90% of students receiving G's.

9. Is there a formal mechanism in place for students to evaluate their professors and attending physicians? What changes have been made recently as a result of this feedback?
Theme and instructor evaluations are mandatory for students, although I'm not sure how much actually gets implemented.

Counseling/Student Support
10. What kind of academic, personal, financial, and career counseling is available to students? Are these services also offered to their spouses and dependents/children?
Moderate but not extensive counseling services are available; the Associate Deans often function in a counseling role, which is a very positive experience (you can spend four years at some schools without seeing an administrator on any personal level.)

11. Is there a mentor/advisor system? Who are the advisors—faculty members, other students, or both?
Students are assigned both faculty and student advisors. An effort is made to match given preferences.

12. How diverse is the student body? Are there support services or organizations for ethnic minorities and women?
There is extensive support, including a prominent department of Minority Affairs. The student body is relatively diverse.

Facilities
13. Tell me about the library and extracurricular facilities (i.e., housing and athletic/recreational facilities).
Facilities are decent; the library is large (5 floors) but old. The student lounge is nice, with a new plasma TV, billiards, fooseball, computers, leather couches, etc.

14. Are there computer facilities available to students? Are they integrated into the curriculum/learning?
There are multiple computer labs in the library in addition to computers in the student lounge. There isn't a large extent of computer integration at this point in time, although lecture materials and schedules are all available online, as well as dissection pre-labs and full lecture audio recordings.

15. What type of clinical sites—ambulatory, private preceptors, private hospitals, rural settings—are available or required for clerkships? Does this school allow for students to do rotations at other institutions or internationally?
All of the above.

16. Is a car necessary for clinical rotations? Is parking a problem?
Not necessary but recommended. Parking can be a hassle but is usually manageable.

Financial Aid
17. What is the current tuition and fees? Is this expected to increase yearly? If so, at what rate?
Tuition and fees are around 43,000 a year, as AMC has a small endowment and no state funding.

18. Are there stable levels of federal financial aid and substantial amounts of university/medical school endowment aid available to students?
Financial aid is stable and adequate, though mostly through Stafford and Graduate+ loans.

19. Are there students who have an "unmet need" factor in their budget? If so, where do these students come up with the extra funds?
Students' budgets are usually adequate. Part-time jobs are rare during the year.

20. Are spouses and dependents/children covered in a student's budget?
I don't know.

21. Is someone available to assist students with budgeting and financial planning?
The Financial Aid office is available for budget counseling.

22. Does this school provide guidance to its students, and to its graduates/alumni, on debt management?
See above.

Student Involvement
23. What medical school committees (e.g., curriculum committee) have student representation?
Student council has a large amount of input with the administrators.

24. Are students involved in (required or voluntary) community service?
A large number of students have been involved in voluntary community service by the time they graduate.

25. How active is the Student Council/Government? Are there other student organizations?
Quite active.

Policies
26. Is there an established protocol for dealing with student exposure to infectious diseases? Is disability insurance provided to cover this exposure?
Yes to the first question, I don't know to the second.

27. Does this school provide, or does the student pay for, vaccinations against Hepatitis B or prophylactic AZT treatment in case of a needle-stick or accident?
I believe they provide, although I'm not certain.

28. Is there a school Honor Code? Is there a grievance process/procedure? Are the students involved?
There is an Honor Code, including an Honor Committee comprised of 1st through 4th year students. Grievance procedures are extensive and taken quite seriously, including obtaining official recorded statements from all parties involved.

Residency
29. May I see a list of residency programs to which this school's recent graduates were accepted?
This information is available on the school's web site.

Questions to Ask Yourself
30. Does this school have strengths in the type of medicine (primary versus specialized care, urban versus rural practice environment, academic medicine versus private practice) that I will want to practice?
There are a large number of opportunities to work with local underserved urban populations. There seems to be more of a focus on primary care.

31. Would I be happy at this school (for at least the next four years)?
Above all else, I'm happy at this school because of my classmates. Students are relatively laid back compared to other schools, and there isn't much of a
"cutthroat culture." People regularly share notes and study tips via class-wide emails, and there is a large and active social scene. Aside from sharing the nightlife together, there are tons of intramural and pick-up sporting events, including basketball, flag football, soccer, squash, bowling, rowing, etc.
 
New York Medical College

Curriculum
1. Are there any special programs for which this medical school is noted?

Not really, we have a focus on ethics and we're noted for being affiliated St. Vincent's Hospitals in NYC. Our curriculum is also very board oriented.

2. Describe this school's curriculum in the pre-clinical and clinical years. Are there any innovations, like Problem-Based Learning?
We begin taking full patient histories and conducting psych interviews in the first year, and through second year we learn the complete physical exam so you're not lost on your first day on the wards.

3. Are there opportunities for students to design, conduct, and publish their own research?

Yes, there is a funded summer internship program on campus and the faculty is very welcoming to medical students to come into their labs/clinics to help with research. It depends on your individual motivation.

4. Is there a note-taking service? If so, is it university-run or student-run?
Yes, there is a student run scribing service that offers transcriptions of lectures and access to previous year's exams.

5. Is there flexibility in the coursework (the number of electives) and the timing of the courses (accelerating, decelerating, and time off) during the pre-clinical and clinical years?
The administration is very understanding that medical students have many facets to their lives and allow deceleration and time off as long as you're willing to pay tuition.

6. Has this medical school, or any of its clinical departments, been on probation or had its accreditation revoked?
Not to my knowledge

7. How do students from this medical school perform on the National Board Examinations? How does the school assist students who do not pass?
We tend to have a 99% passage rate on the board exams, and score consistently 5 points above the national average. There is a program in place for getting students to pass.

Evaluations
8. How are students evaluated academically? How are clinical evaluations performed?
In the preclinical years, we are assessed according to Honors, High Pass, Pass, Fail based solely on our exam performance. In the clinical years, we are evaluated by residents and attendings in the same Honors, High Pass, Pass, Fail grading scheme.

9. Is there a formal mechanism in place for students to evaluate their professors and attending physicians? What changes have been made recently as a result of this feedback?
We have a online evaluation system that we are required to complete before we receive grades for a course. Not sure about the clinical years. As a result of this feedback, one of courses was completely restructured for the following year and the course director replaced.

Counseling/Student Support
10. What kind of academic, personal, financial, and career counseling is available to students? Are these services also offered to their spouses and dependents/children?

Our faculty and administration is VERY supportive of the student body and offers all of those kinds of counseling to the students from day one of orientation. They are also open to student's family members as well.

11. Is there a mentor/advisor system? Who are the advisors—faculty members, other students, or both?
There is a relatively informal Big Sibling system for first year med students where a second year is assigned to give them advice on the upcoming year. There is a faculty mentoring service that is made available second year to students, but again, the doors of most faculty members are open any time you need them.

12. How diverse is the student body? Are there support services or organizations for ethnic minorities and women?
The student body is quite diverse. There are student-run organizations for pretty much any cultural group and we have a very active AMWA chapter.

Facilities
13. Tell me about the library and extracurricular facilities (i.e., housing and athletic/recreational facilities).
The library is fairly small since we are a small private medical college, however it is quiet and has adequate space to accommodate the people that use it. On campus housing is apartment style and fairly affordable compared to the surrounding areas. The small on campus athletic facility leaves something to be desired, but there is a NY Sports Club about 3 minutes from campus for those that are really concerned about hitting the gym. As you've probably heard, we have the best anatomy lab in the US in terms of ventilation and the fact that it is naturally lighted by a bank of north-facing windows, which makes you much less depressed about spending so much time secluded in the lab.

14. Are there computer facilities available to students? Are they integrated into the curriculum/learning?
Yes, there are computer facilities and they are integrated into the curriculum. We have 2 computer labs and modular class rooms with computer equipment.

15. What type of clinical sites—ambulatory, private preceptors, private hospitals, rural settings—are available or required for clerkships? Does this school allow for students to do rotations at other institutions or internationally?
There are MANY clinical sites running the gamut from private preceptors in the suburbs and NYC for the first to years. For clinical rotations, there are sites available in settings ranging from the Lower-East Side of Manhattan to the quasi-rural Danbury, CT. We are allowed to do away rotations as well as international rotations in the fourth year.

16. Is a car necessary for clinical rotations? Is parking a problem?
A car is fairly necessary for rotations, but is more necessary to maintain ones sanity during the first 2 years because we are somewhat isolated from the grocery store, bars, etc. Parking is FREE!

Financial Aid
17. What is the current tuition and fees? Is this expected to increase yearly? If so, at what rate?
For on campus housing the tuition and fees for the 2007-08 year come out to about $62,000. This was about a 2% increase over last year's tuition and fees.

18. Are there stable levels of federal financial aid and substantial amounts of university/medical school endowment aid available to students?
All students are eligible for Federal Fin. Aid and there is a limited amount of institutional aid which is awarded on the basis of need.

19. Are there students who have an "unmet need" factor in their budget? If so, where do these students come up with the extra funds?
Our financial aid office ensures that every student has ample funding inorder to live comfortably. If there is a need beyond what the school budget allows for, there are emergency loans available.

20. Are spouses and dependents/children covered in a student's budget?
Yes

21. Is someone available to assist students with budgeting and financial planning?
Yes

22. Does this school provide guidance to its students, and to its graduates/alumni, on debt management?
Yes

Student Involvement
23. What medical school committees (e.g., curriculum committee) have student representation?
There is a student curriculum committee run by the student senate.

24. Are students involved in (required or voluntary) community service?
There are many students actively involved in community service, though it is not mandatory.

25. How active is the Student Council/Government? Are there other student organizations?
Student Sentate is VERY active in the school. There are many student organizations that fall under the umbrella of the senate.

Policies
26. Is there an established protocol for dealing with student exposure to infectious diseases? Is disability insurance provided to cover this exposure?
Not sure, but my guess is yes.

27. Does this school provide, or does the student pay for, vaccinations against Hepatitis B or prophylactic AZT treatment in case of a needle-stick or accident?
The school provides for all medical expenses in the case of needle-sticks/potential exposures.

28. Is there a school Honor Code? Is there a grievance process/procedure? Are the students involved?
Yes there is an honor code, and yes there is a grievance process that allows/requires other students to report violations to the dean of students.

Residency
29. May I see a list of residency programs to which this school's recent graduates were accepted?
Yes, it is posted on the NYMC website

Questions to Ask Yourself
30. Does this school have strengths in the type of medicine (primary versus specialized care, urban versus rural practice environment, academic medicine versus private practice) that I will want to practice?
NYMC allows you to see the full range of practice, from the streets of NYC to the semi-rural setting of Connecticut. It's affiliations with a Level 1 trauma centers, urban, suburban, and semi-rurel, academic and private facilities allow you to tailor your education to your career.

31. Would I be happy at this school (for at least the next four years)?
You'd have to see it to believe it. From day one I have felt at home at NYMC, and from what I've seen thus far in my education, I couldn't ask for more.
 
Columbia University, College of Physicians and Surgeons

Curriculum
1. Are there any special programs for which this medical school is noted?

All the normal double degree programs (i.e. MPH, MBA, PhD, MFA)

2. Describe this school's curriculum in the pre-clinical and clinical years. Are there any innovations, like Problem-Based Learning?

P&S’s philosophy, with which I thoroughly agree, is that PBL only works well if there’s a solid foundation of knowledge underlying it. First year is all lectures, dissections and small-group teaching. Second year is lectures immediately followed by 2 hours of case discussions, which were conducted at a much higher level due to the solid base of 1st yr knowledge. If you jump into PBL right away, you just memorize the cases.

3. Are there opportunities for students to design, conduct, and publish their own research?

Absolutely, in spades. We have so many resources, mentors and lab projects—in structural finance or orthopedics—it’s ridiculous. Ridiculous, I say.

4. Is there a note-taking service? If so, is it university-run or student-run?

Yup, student transcripts. It works out to about 2 transcripts per year apiece. We also podcast our lectures.

5. Is there flexibility in the coursework (the number of electives) and the timing of the courses (accelerating, decelerating, and time off) during the pre-clinical and clinical years?

Nope. We march through with lockstep conformity. Though to be frank we don’t really admit students who are particularly prone to academic difficulties. Electives are only present in the fourth year (which is ALL elective); and loads of students—25-30% of each class—take a year off to do either research or something they find intriguing.

6. Has this medical school, or any of its clinical departments, been on probation or had its accreditation revoked?

Ha ha ha ha.

7. How do students from this medical school perform on the National Board Examinations? How does the school assist students who do not pass?

We have been told that we have amongst the highest averages in the country (usually around 230 or so). If a student doesn’t pass, he’s pulled out of the first rotation of third year, given four weeks to study and retake, and starts MS3 again. If he fails the second time, he sits out the entire third year, though I don’t think that’s ever happened.

Evaluations
8. How are students evaluated academically? How are clinical evaluations performed?
Academics in the first two years are strictly based on examinations, with the exception of Psychiatric Medicine in 2nd year which has some preceptor evaluation component. In the third and fourth years it’s the regular thing: clinical performance, fund of knowledge, enthusiasm, shelf scores.

9. Is there a formal mechanism in place for students to evaluate their professors and attending physicians? What changes have been made recently as a result of this feedback?
They solicit our feedback but I’m not sure how much it’s worth. The Clinical Practice course in the first two years (the “art of doctoring” course) is very responsive to suggestions.


Counseling/Student Support
10. What kind of academic, personal, financial, and career counseling is available to students? Are these services also offered to their spouses and dependents/children?

The Dean meets with students during her office hours. Additionally there is a wonderful ‘advisory dean’ system set up where ¼ of each class is assigned to a mentor who meets with us weekly-monthly to discuss anything on our minds. Financial aid counsels students as well. And the Student Wellness Center is an invaluable resource to deal with stress, pressure, relationships, mental illness and the countless other issues that come up in our personal lives.

11. Is there a mentor/advisor system? Who are the advisors—faculty members, other students, or both?

Advisory deans, as discussed in 10.

12. How diverse is the student body? Are there support services or organizations for ethnic minorities and women?

Quite diverse… though blond rugby players are a bit over-represented. J The minority group BALSO is very active on campus, and most under-represented minority students with multiple offers of admission say they chose Columbia due to its diverse and welcoming atmosphere compared with other top-ranked schools.

Facilities
13. Tell me about the library and extracurricular facilities (i.e., housing and athletic/recreational facilities).

Athletics is huge. We have a paltry gym, fabulous weight club and loads of sports teams. P&S is an extra-curricular school, hands down. Everyone does something, and the proportion of students who have run a marathon is approaching ½. Musical and artistic pursuits are also well-represented.

Housing in the first year is… umm.. adequate. But in the upper years we live in beautiful apartments overlooking the Hudson for unbelievably low prices (for New York at least).

14. Are there computer facilities available to students? Are they integrated into the curriculum/learning?

Yes. We own computers. We use computers. Imagine that.

15. What type of clinical sites—ambulatory, private preceptors, private hospitals, rural settings—are available or required for clerkships? Does this school allow for students to do rotations at other institutions or internationally?

We have our enormous quaternary teaching hospital, New York-Presbyterian, the largest hospital in the US. We have two private hospitals, St Luke’s and Roosevelt, in the City. We have one public hospital, Harlem Hospital. And we have loads of satellites, from Bassett in Cooperstown (upstate NY) and Stamford in CT to Navajo reservations.

We have over 30 formal international exchanges with foreign medical schools and a formal 2-month elective, Medicine in the Tropics, that sends students to any site overseas. Between 1/3 and ½ of MS4s go abroad. And loads of people do away electives in other US cities.

16. Is a car necessary for clinical rotations? Is parking a problem?

No. Unless you’re in New Mexico on a reservation, everything in NYC is available by subway or taxi.

Financial Aid
17. What is the current tuition and fees? Is this expected to increase yearly? If so, at what rate?
Too much to think about… usually about $52,000 a year all-included. It increases no faster than any other school.

18. Are there stable levels of federal financial aid and substantial amounts of university/medical school endowment aid available to students?
The endowment is large so the financial aid is extremely generous.

19. Are there students who have an "unmet need" factor in their budget? If so, where do these students come up with the extra funds?
The stipend is ample, but I think there are slush funds if necessary.

20. Are spouses and dependents/children covered in a student's budget?
Yes

21. Is someone available to assist students with budgeting and financial planning?
Yes, the Financial Aid Office

22. Does this school provide guidance to its students, and to its graduates/alumni, on debt management?
Yes, throughout the years and at a formal exit interview before graduation

Student Involvement
23. What medical school committees (e.g., curriculum committee) have student representation?
Oh, goodness, what doesn’t? P&S students are renowned for their enthusiasm and activism. The Dean’s Advisory Committee meets monthly; Alumni Affairs has students, everyone has student.

24. Are students involved in (required or voluntary) community service?
Community service strictly defined probably pulls in 10% or so of each class. But *everyone* is involved in something extra-curricularly, whether it’s writing, sports, music, cultural societies, or groups formed to enjoy the many delights of living in NY.

25. How active is the Student Council/Government? Are there other student organizations?
The P&S Club is the umbrella org for all the student groups, of which there are over 50. All of them are active. It’s awesome. And yes, the student council does a lot. It’s really cool to go to such an active, enthusiastic medical school, where people have managed to keep their interests outside of medicine alive.


Policies
26. Is there an established protocol for dealing with student exposure to infectious diseases? Is disability insurance provided to cover this exposure?

Yes, we’re forced to get the requisite immunizations and the needlestick protocol and isolation protocols are drummed into us.

27. Does this school provide, or does the student pay for, vaccinations against Hepatitis B or prophylactic AZT treatment in case of a needle-stick or accident?
Yes, of course.

28. Is there a school Honor Code? Is there a grievance process/procedure? Are the students involved?
No honor code. Everything is pretty much on a case-by-case basis, but the school wants everyone admitted to eventually graduate with an MD (for the most part), so there’s lots of second chances for everyone.

Residency
29. May I see a list of residency programs to which this school's recent graduates were accepted?

Sure. It’s a laundry list of hyperspecialization at all the top programs. We usually match 6-10 people in neurosurgery a year (a particular focus); 10-15 in orthopedics; 10 in radiology, 10 in general surgery, 15 in anesthesia, 5 in OB-GYN, 30 in internal medicine, 15 in peds, 10-14 in psych, 1-3 in rad onc, 3-5 in ophtho, 3-4 in derm, 5-6 in neuro, 2-3 in plastics, 3-4 in ENT, and 1 person in family medicine every blue moon.

Questions to Ask Yourself
30. Does this school have strengths in the type of medicine (primary versus specialized care, urban versus rural practice environment, academic medicine versus private practice) that I will want to practice?
We push students into the most competitive specialties, and we hyperspecialize in those specialties. Do not come here if you are interested in family medicine (except for urban immigrant populations) or primary care. It’s sad but the focus is on very technical care. Even on the MS3 clerkships you’re assigned to a specialty—cardiology instead of medicine, for example. We have 15 weeks of surgery and surgical subs and 5 weeks of primary care. If you’re interested in matching in a top program, going into academics or just living in NYC then this is a great pick.

31. Would I be happy at this school (for at least the next four years)?

P&S is a “balls to the wall” medical school that is full of happy, well-adjusted, interesting, active students. Again, the focus is away from rural medicine or primary care, but the international involvement, high-level research and extraordinary array of pathology seen in the teaching hospitals is something unmatched by any other medical school in the United States. New York is the most diverse city in the world, and the stuff that walks through the doors of the hospital is seen only in the textbooks anywhere else. Our ORs turn 200 cases a day; we do 3-5 Whipples a week; we treat loads of cases of malaria and neurocystocercosis; and our patient census seems like the United Nations. It’s a cool and unique clinical education built on a very firm pre-clinical foundation.
 
Curriculum

1. Are there any special programs for which this medical school is noted?
Students have lots of experiences thrown their way. Since the hospitals we rotate through generally do not have residency programs, students become the first assistants to attendings.

2. Describe this school's curriculum in the pre-clinical and clinical years. Are there any innovations, like Problem-Based Learning?
The school uses a mixture of PBL and lecture throughout the first two years. For third year, students use an integrated teaching style. Instead of spending 8 weeks on surgery followed by 8 weeks on surgery, students will spend up to three months at a hospital, involved in every aspect of a patient's care---from the admission from the emergency department to scrubbing in on the surgery.

3. Are there opportunities for students to design, conduct, and publish their own research?
Absolutely. The laboratories are directly upstairs from the classrooms. Students have participated in a wide variety of research opportunities---from genetics to clinical ob/gyn to even ethics. Most notably, we have genetics and neuroscience immediately available for students to get involved with.

4. Is there a note-taking service? If so, is it university-run or student-run?
There is not a note-taking service. Lectures are videotaped and handouts are distributed for each class.

5. Is there flexibility in the coursework (the number of electives) and the timing of the courses (accelerating, decelerating, and time off) during the pre-clinical and clinical years?
The curriculum for the first two years cannot be changed.

6. Has this medical school, or any of its clinical departments, been on probation or had its accreditation revoked?
No

7. How do students from this medical school perform on the National Board Examinations? How does the school assist students who do not pass?
The school admitted its first class in the fall of 2004. That group had an average STEP I score of 229. So far, this year's class has had a 100% pass rate. If a student should fail the USMLE, he will be pulled from rotations and given time to re-take the exam.

Evaluations
8. How are students evaluated academically? How are clinical evaluations performed?
We use multiple-choice tests for the first two years. Clinical grades are based upon a combination of written (Shelf) exams, oral exams, and attending/resident evaluations.

9. Is there a formal mechanism in place for students to evaluate their professors and attending physicians? What changes have been made recently as a result of this feedback?
Students use an anonymous online survey to evaluate every professor. Supposedly, really bad professors are pulled from teaching duties. I am unaware of this system ever having an impact.

Counseling/Student Support
10. What kind of academic, personal, financial, and career counseling is available to students? Are these services also offered to their spouses and dependents/children?

The Dean of Student Affairs is always willing to work with a student's personal issues. The FAU campus also offers a mental health support system should students request its services. Career counseling comes from all layers of administration. Twice each year, the Dean of Medical Education meets with students to discuss career goals and options. The university also employs mentors to guide students. There are no services for families.

11. Is there a mentor/advisor system? Who are the advisors-faculty members, other students, or both?
Formally, the mentors are faculty members. We have an informal big brother/big sister program where the 2nd years help the 1st years.

12. How diverse is the student body? Are there support services or organizations for ethnic minorities and women?
The gender ratio is typically 50% women and 50% men. The Boca campus currently does not have a separate office for minority affairs.

Facilities
13. Tell me about the library and extracurricular facilities (i.e., housing and athletic/recreational facilities).
Students have access to an expansive online collection involving numerous textbooks and journals. The physical library is meant for the undergraduates, but is adequate for medical students wishing to find a place to study. The unique perk of the Boca campus is the medical student offices, called "the nerdery." Each student gets his own cubicle with computer, free unlimited printing and copying, and a lock for storing materials. The nerdery is password-protected, giving extra security.

There is a gym, numerous intramural sports teams, and an Olympic-size swimming pool for students to use without additional cost. There are no housing facilities.

14. Are there computer facilities available to students? Are they integrated into the curriculum/learning?
The nerdery serves as the computer lab. As of right now, e-learning is rarely used.

15. What type of clinical sites-ambulatory, private preceptors, private hospitals, rural settings-are available or required for clerkships? Does this school allow for students to do rotations at other institutions or internationally?

16. Is a car necessary for clinical rotations? Is parking a problem?
A car is definitely required. Parking is not a problem since students get to park in the physician lot.

Financial Aid
17. What is the current tuition and fees? Is this expected to increase yearly? If so, at what rate?
Current tuition is about $30,000

18. Are there stable levels of federal financial aid and substantial amounts of university/medical school endowment aid available to students?

19. Are there students who have an "unmet need" factor in their budget? If so, where do these students come up with the extra funds?

20. Are spouses and dependents/children covered in a student's budget?

21. Is someone available to assist students with budgeting and financial planning?


22. Does this school provide guidance to its students, and to its graduates/alumni, on debt management?

Student Involvement

23. What medical school committees (e.g., curriculum committee) have student representation?
Almost all committees have student involvement---curriculum, teacher evaluation, ethics/honor court.

24. Are students involved in (required or voluntary) community service?
Volunteer service is not required. There are numerous regional and international opportunities for volunteering. The Department of Community Service hosts five health fairs throughout the year. In addition, there are two clinics in South Florida. Internationally, students have traveled to the Dominican Republic, Haiti, Nicaragua, and India.

25. How active is the Student Council/Government? Are there other student organizations?

Policies

26. Is there an established protocol for dealing with student exposure to infectious diseases? Is disability insurance provided to cover this exposure?

27. Does this school provide, or does the student pay for, vaccinations against Hepatitis B or prophylactic AZT treatment in case of a needle-stick or accident?
Every student must be vaccinated against Hep B before matriculating.

28. Is there a school Honor Code? Is there a grievance process/procedure? Are the students involved?
There is an honor code. As part of enforcement, the students elect an ethics rep to serve as a student advisor should behavior ever become questionable.

Residency
29. May I see a list of residency programs to which this school's recent graduates were accepted?

Questions to Ask Yourself

30. Does this school have strengths in the type of medicine (primary versus specialized care, urban versus rural practice environment, academic medicine versus private practice) that I will want to practice?
Students are exposed to all of the listed areas of care except for rural practice. View the Match list to get a better idea.

31. Would I be happy at this school (for at least the next four years)?
That depends. The Boca campus has a small student body (currently 32 students per class) and is very focused on group work. The team approach to medicine is pushed here more than at the Miami campus. Students seeking to work independently should seek admission to the main campus.
 
Uniformed Services University of the Health Sciences

Anonymous Submission

Curriculum
1. Are there any special programs for which this medical school is noted?
It's the only medical school in nation to focus on "military medicine". USU is strong in teaching students skills that will make them better military physicians. From the first year on we are taught battlefield medicine. There is always ongoing research in issues relevant to the military, such as tropical medicine, parasitology, PTSD, etc.

2. Describe this school's curriculum in the pre-clinical and clinical years. Are there any innovations, like Problem-Based Learning?
Mostly traditional lecture format for the first two years and clinical rotations for the last two. Grades are ABCDF for the first two years and parallel, but renamed Honors, High Pass, etc. for the last two. There are some small groups in different classes that give students the opportunity to independently learn a topic and present to their classmates, but they're add-ons to a traditional lecture format. Almost all classes are graded through multiple choice tests, with practical fill-in-the-blanks for laboratory sections. Some classes such as Human Context in Health Care, Medical History, and Ethics require papers.

3. Are there opportunities for students to design, conduct, and publish their own research?
Yes, if you find a faculty member to work with.

4. Is there a note-taking service? If so, is it university-run or student-run?
No. Many classes provide note sets which the tests are obviously primarily based on. Others primarily use textbooks.

5. Is there flexibility in the coursework (the number of electives) and the timing of the courses (accelerating, decelerating, and time off) during the pre-clinical and clinical years?
Not really in the pre-clinical years. There are some summer programs that can be tailored, but the primary academic coursework is set. In third year, only locations and schedules are flexible, but everyone does the same rotations. In fourth year there are required rotations that everyone does and electives that allow for some flexibility.

6. Has this medical school, or any of its clinical departments, been on probation or had its accreditation revoked?
I don't believe so, no.

7. How do students from this medical school perform on the National Board Examinations? How does the school assist students who do not pass?
The most recent class' average was 213 with 14 at or above 240. Pass percentage was 91.4%

Help for those who do not pass seems to consist mostly of a 6 week opportunity to study and the school pays for the retake (they also pay for the first time).

Evaluations
8. How are students evaluated academically? How are clinical evaluations performed?
As mentioned earlier, most evaluation is by multiple choice test, practicals where appropriate (anatomy, neuro, histology). Clinical evaluations are a combination of subjective evaluations from the clinical team and a preceptor plus an objective result from a shelf.

9. Is there a formal mechanism in place for students to evaluate their professors and attending physicians? What changes have been made recently as a result of this feedback?
Yes, there is constant student feedback. Our biochem course changed to a more clinical course, changed textbooks, and added small group discussions on clinical applications of biochemistry based on the previous class' evaluations.

Counseling/Student Support
10. What kind of academic, personal, financial, and career counseling is available to students? Are these services also offered to their spouses and dependents/children?
It mostly seems informal. If you're doing poorly or request it, you can get counseling. Spouses and dependents are all covered by the military benefits system, so they can get professional counseling at no cost.

11. Is there a mentor/advisorMentoring or advising is mostly informal.
You can develop relationships with any faculty member you want to. Most of them are very open and friendly. There is one mandatory "chat" early in first year, but mostly it's voluntary and informal.

12. How diverse is the student body? Are there support services or organizations for ethnic minorities and women?
Very diverse in backgrounds, previous occupations, etc, but not that diverse racially. There is an office to recruit and advise minority students, but I don't know how many use it. There is a "Women in Medicine" interest group with strong female mentors. There are also a number of minority interest groups, like an asian-pacific medical student association (or something similar) that had a conference at USU last year.

Facilities
13. Tell me about the library and extracurricular facilities (i.e., housing and athletic/recreational facilities).
Library is great, lots of computers and study facilities are available. There is no set housing and housing in the area is expensive, but as military officers, students receive a housing allowance that will generally pay most of the cost of renting a nice apartment/condo. Some students buy houses. It's nice to have a salary! There is a gym with indoor pool on the NNMC campus and a smaller gym in the school.

14. Are there computer facilities available to students? Are they integrated into the curriculum/learning?
Yes. There are many computers (approximately 100?) in the library (LRC). There is also wireless internet available throughout the school, including the lecture halls. They're not totally integrated into the curriculum, but classes are often recorded and put on the internet and materials are also often available online.

15. What type of clinical sites—ambulatory, private preceptors, private hospitals, rural settings—are available or required for clerkships? Does this school allow for students to do rotations at other institutions or internationally?
Rotations are done throughout the military medical system around the entire United States. From East Coast to Hawaii, students rotate all over. Of course, if they want to stay locally, most can do most rotations nearby, but some travel is inevitable.

16. Is a car necessary for clinical rotations? Is parking a problem?
Depends on location.

Financial Aid
17. What is the current tuition and fees? Is this expected to increase
yearly? If so, at what rate?

Free and students are paid a good salary as O-1s in the military.

18. Are there stable levels of federal financial aid and substantial amounts of university/medical school endowment aid available to students?
Not really needed when everything is free and we're paid to attend.

19. Are there students who have an "unmet need" factor in their budget? If so, where do these students come up with the extra funds?
N/A

20. Are spouses and dependents/children covered in a student's budget?
Yes, officers with dependents make slightly more and dependents have
full medical coverage.

21. Is someone available to assist students with budgeting and financial planning?
Informally, someone would be able to ask for this type of assistance.

22. Does this school provide guidance to its students, and to its graduates/alumni, on debt management?
Not really. Again, someone could ask and there are numerous financial organizations that specialize in advising officers on investments and debt management. Debt is not a huge problem at USUHS due to free tuition and salary.

Student Involvement
23. What medical school committees (e.g., curriculum committee) have student representation?

Most of the committees have student representation, but how effective it is depends on the students.

24. Are students involved in (required or voluntary) community service?
Generally voluntary. There are no requirements.

25. How active is the Student Council/Government? Are there other student organizations?
They are as active aminor changes, but mostly I think student government is overrated. I don't feel much need for them, but maybe that's because they've been so successful. Faculty takes student government more seriously than I do and that's probably good.

Policies
26. Is there an established protocol for dealing with student exposure to infectious diseases? Is disability insurance provided to cover this exposure?
Yes and students all have full medical coverage.

27. Does this school provide, or does the student pay for, vaccinations against Hepatitis B or prophylactic AZT treatment in case of a needle-stick or accident?
We're all vaccinated against almost everything, including Hep B. I'm not certain about prophylactic AZT, but I'm sure it's available in case of a needle stick.

28. Is there a school Honor Code? Is there a grievance process/procedure? Are the students involved?
Yes, we have student "honor reps". Honor and integrity are emphasized
as part of professional obligations as future physicians and officers. Cheating seems like it's not a big problem here. I've never heard of any problems.

Residency
29. May I see a list of residency programs to which this school's recent
graduates were accepted?
Students are required to do at least their internship in military facilities. Not getting full freedom to pick your own residency is one of the big drawbacks of a career as a military physician. Refer to the mil med forum for much, much discussion of that.

Questions to Ask Yourself
30. Does this school have strengths in the type of medicine (primary
versus specialized care, urban versus rural practice environment, academic medicine versus private practice) that I will want to practice?


31. Would I be happy at this school (for at least the next four years)?
I don't think I knew what to ask prior to coming here and I doubt many prospective students will, either. Med school is unlike anything else I've ever done. If you can be happy in the middle of such a demanding program, then you can certainly be happy here. It's a pretty friendly school. Money is good, classmates are friendly, I would probably still recommend it to anyone who is interested in a career as military physician (and only those who are interested in being a military physician).
 
Jefferson Medical College

Curriculum
1. Are there any special programs for which this medical school is noted?


  • PSU-Jefferson 6 year, guaranteed and accelerated program. Students are accepted into PSU directly out of high school, and start medical school at Jefferson after 2 years of undergrad.
  • Jefferson-Hopkins 5 year MPH program
  • JeffHOPE: Students run homeless shelter health clinics in the city. It's one of the biggest such programs in the nation.

2. Describe this school's curriculum in the pre-clinical and clinical years. Are there any innovations, like Problem-Based Learning?


  • There is some PBL in the Intro to Clinical Medicine 1 course in the first year. Other courses, such as biochemistry and immunology, use team-based learning (students are broken up into groups, and answer a series of questions related to a case study. The groups then compare answers and are invited to defend the answer that they chose.)
  • All courses in the first 2 years have integrated case-based learning into the curriculum. “Real life” applications are heavily stressed.

3. Are there opportunities for students to design, conduct, and publish their own research?


  • Yes – the dean's office is quite active in recruiting students to start doing research, especially in the first year.

4. Is there a note-taking service? If so, is it university-run or student-run?


  • Yes – it is run by Student Council. The service is totally online and absolutely free. Students who participate take turns scribing lectures, which are emailed to a Student Council rep and posted online. These notes are in addition to the pre-printed lecture notes/syllabi that the course directors give to the students.

5. Is there flexibility in the coursework (the number of electives) and the timing of the courses (accelerating, decelerating, and time off) during the pre-clinical and clinical years?


  • Not sure. If you have a particular request, then you can work with your dean to make arrangements. However, I'm not sure if any kind of administrative procedure is outlined in the student handbook.

6. Has this medical school, or any of its clinical departments, been on probation or had its accreditation revoked?


  • No.

7. How do students from this medical school perform on the National Board Examinations? How does the school assist students who do not pass?


  • The school does quite well. The average Step 1 score in past years has been ~ 220-221, which is slightly above the national average of 218. Any instances where students do not pass are probably handled on a case-by-case basis. Students meet with the deans to plot a course of action, but it probably depends on what the individual student wants to do.

Evaluations
8. How are students evaluated academically? How are clinical evaluations performed?


  • In the pre-clinical years, students are graded on an H/P/F scale. Grades are almost entirely objective (based on multiple choice exams).
  • In the clinical years, the grades are on a Honors/Excellent/Good/Marginal/Fail system. As in almost all other schools, the majority of the grade is subjective and is based on attending/resident feedback. I believe that we take the NBME shelf exams for all core rotations except family medicine and surgical-subspecialty electives.

9. Is there a formal mechanism in place for students to evaluate their professors and attending physicians? What changes have been made recently as a result of this feedback?


  • They take student feedback very seriously here - students MUST submit feedback for each course and the professors after each exam. Otherwise, grades will not be sent out to the class.
  • The Intro to Clinical Medicine course for the MS1 year was completely revamped, due to student feedback. Every year, courses are adjusted and tweaked, due to student feedback. They DO listen to student suggestions and take them into consideration.

Counseling/Student Support
10. What kind of academic, personal, financial, and career counseling is available to students? Are these services also offered to their spouses and dependents/children?


  • Personal counseling is available from the Department of Psychiatry. One of the psychiatrists is responsible for student counseling. (He is not responsible for evaluating medical students at any point in their training, which prevents possible conflicts of interest.) From students who have used the service, it is truly helpful.
  • Academic counseling is available from the Office of Student Affairs and also from the Dean's office. One of the student deans specializes in talking to students about improving their study skills.
  • Basically, everyone is willing to help each other out. I've rarely encountered anyone in the administration who has actively tried to “pass the buck.”

11. Is there a mentor/advisor system? Who are the advisors—faculty members, other students, or both?


  • All students are assigned to one of four student deans. Each dean is a physician and is familiar with the needs of medical students.
  • All first years are assigned a “Big Sib” from among the second year class.
  • First and Second Year students have the option of signing up for a clinical mentorship program, in which students state which fields interest them, and are then matched with physicians who practice in that field.

12. How diverse is the student body? Are there support services or organizations for ethnic minorities and women?


  • Jefferson's Office of Diversity and Minority Affairs (ODAMA) and SNMA are very active. They hold frequent festivals and free lectures for students, and discuss cultural sensitivity in a medical context. ODAMA recently won an award for their efforts to increase minority support and diversity at Jefferson.

Facilities
13. Tell me about the library and extracurricular facilities (i.e., housing and athletic/recreational facilities).


  • Student apartments (Barringer and Orlowitz) are available to students. They are exceptionally convenient, since they are next door to the library. These apartments are private apartments, with individual cooking units and bathrooms.
  • The student dormitory (Martin) is also very convenient – it is on the other side of the library. It is a true dormitory, with communal kitchens and bathrooms. However, it is much cheaper than Orlowitz and Barringer.
  • The library is located across the street from the building where classes are held. It is open 24 hours a day, and has controlled access (students must show ID in order to enter the library). The Learning Resource Center in the library is a great resource, with anatomical models, videos, and group study rooms available.
  • (As a note: many of the resources that were previously only available from the library are now being made available to students via the internet. In other words, histology videos, physical diagnosis videos, and anatomy dissection videos are now available online, and no longer have to be borrowed from the library.)
  • The student gym is in the basement of Alumni Hall, and there are smaller gyms available in Orlowitz and Barringer. The gym in Alumni Hall was recently renovated, and includes a pool, a weight room, and a cardio room.
  • The Clinical Skills Center at 833 Chestnut is very well equipped. They have mock exam rooms equipped with video cameras and recording equipment – students in MS1 and MS2 are recorded while doing mock interviews. They then watch the video in small group sessions, and critique their own performance. (Not as intimidating as it sounds.) The CSC also has simulated dummies and a large pool of standardized patients of a variety of races and ethnicities.
  • New facilities will be available in the Dorrance Hamilton building that is currently being constructed behind the library.

14. Are there computer facilities available to students? Are they integrated into the curriculum/learning?


  • Yes – there are computer facilities in the library, Alumni Hall, and the Edison building. Students in the first year are required to use the computer to complete certain assignments. The first week of classes includes a mandatory session that teaches students how to use Ovid and Micromedex. By the second year, you are required to demonstrate competence in medical software programs, such as UpToDate, Micromedex, and Ovid.

15. What type of clinical sites—ambulatory, private preceptors, private hospitals, rural settings—are available or required for clerkships? Does this school allow for students to do rotations at other institutions or internationally?


  • Ambulatory, private hospitals, and rural settings are available for clerkships. You can do away rotations (international or at other institutions) in your 4th year.

16. Is a car necessary for clinical rotations? Is parking a problem?


  • A car is not necessary for clinical rotations, although it can make it easier to get to some sites.
  • Parking is not very convenient in Philadelphia (as is often the case in large cities.) There is parking at any clinical rotation sites that require a car. A car is not necessary for the first 2 years.

Financial Aid
17. What is the current tuition and fees? Is this expected to increase yearly? If so, at what rate?


  • Current tuition and fees is a little over $40,000. There are no expected yearly increases – any increases are done on a case by case basis.

18. Are there stable levels of federal financial aid and substantial amounts of university/medical school endowment aid available to students?


  • I'm not sure how to answer this question best. I can answer from my own personal experience, but I'm sure that it's very different for all different people.

19. Are there students who have an "unmet need" factor in their budget? If so, where do these students come up with the extra funds?


  • Not sure.

20. Are spouses and dependents/children covered in a student's budget?


  • Not sure.

21. Is someone available to assist students with budgeting and financial planning?


  • Yes – the Office of Financial Aid is willing to talk to students who need one-on-one counseling. Susan Batchelor McFadden is the contact person for medical students.

22. Does this school provide guidance to its students, and to its graduates/alumni, on debt management?


  • Yes. Susan Batchelor McFadden routinely sends out emails and sets up workshops to help students manage debt while in school. They also answer questions from recent graduates who need assistance.

Student Involvement
23. What medical school committees (e.g., curriculum committee) have student representation?


  • ALL committees (except the Academic Promotions Committee, for obvious reasons) have student representation. Students are heavily involved in practically every aspect of the Medical College, and have the opportunity to participate in school administrative committees.

24. Are students involved in (required or voluntary) community service?


  • Students not only participate in community service, they actually help run the clinics as well. Students are heavily, heavily involved in community service, from JeffHOPE to JeffYES to JeffSEAL.

25. How active is the Student Council/Government? Are there other student organizations?


  • Student Council is very active, and helps oversee Jefferson's extensive list of student activity groups. Jefferson prides itself on student involvement at all levels, and this is not an overstatement. All student extracurricular groups are led and organized by students. There is an extremely long list of student groups – I would estimate that there are probably about 30-40 active student groups on campus right now.
Policies
26. Is there an established protocol for dealing with student exposure to infectious diseases? Is disability insurance provided to cover this exposure?


  • There is an established protocol for dealing with exposures and sticks. The health insurance that all students are required to have should pay for it.

27. Does this school provide, or does the student pay for, vaccinations against Hepatitis B or prophylactic AZT treatment in case of a needle-stick or accident?


  • The school provides both the Hep B vaccine and needle-stick prophylaxis. University Health Services coordinates both.

28. Is there a school Honor Code? Is there a grievance process/procedure? Are the students involved?


  • There is a school Honor Code, and students are required to sign a copy of it before each exam.
  • The Professional Conduct Committee is run almost entirely by students (with faculty support). They act as intermediaries between students and faculty if problems arise.

Residency
29. May I see a list of residency programs to which this school's recent graduates were accepted?


  • I have not seen a list available online. A printed copy is available to all students on Match Day. The students routinely match well, and send students to a wide variety of specialties, and students match all over the country.

Questions to Ask Yourself
30. Does this school have strengths in the type of medicine (primary versus specialized care, urban versus rural practice environment, academic medicine versus private practice) that I will want to practice?


  • Jefferson has strong departments in both primary and specialty care. Jefferson's Family Medicine department is very strong, with a particular focus on urban community outreach. They do a lot of work in Philadelphia's inner-city and underserved populations. However, Jefferson also has very strong neurosurgery, orthopedics, and ophthalmology departments. Rothman Institute (orthopedics) is strong. Wills Eye is one of the best ophtho hospitals in the nation.
  • The school's focus is probably more urban than rural, just because of our location in Philadelphia.
  • Most universities tend to focus more on academic medicine. However, since Jefferson is quite old and has such a large student body, there is an enormous alumni network (many of whom went into private practice) to draw upon for support and advice.

31. Would I be happy at this school (for at least the next four years)?


  • I can't answer this for anyone else. It's a good school, that gives you the tools you need to succeed and do well. Many of the professors here are phenomenal. They actually listen to student input, and make course changes based on that. If you come here determined to work hard, and if you expect to act like a team player from day 1, then you'll be happy here. They do stress that your classmates will be your colleagues some day, and encourage students to help each other – which they do. Students freely pass around study guides and flashcards, and competitive back-stabbing behavior is strongly discouraged by both upper-year students and by the faculty.
  • The vast majority of the students here are laid back. It's a great place to attend school – it's got fun things to do outside of school, but also great people to hang out with.
  • When the faculty invite you to ask them for help, they really mean it. It's not just an empty invitation, and they remind you (over and over again) that good doctors don't practice “in a vacuum” - that you must stay connected with your colleagues in order to be an effective physician.
 
Ben Gurion Medical School for International Health

Anonymous Contribution

Curriculum
1. Are there any special programs for which this medical school is noted?
Ben Gurion MSIH focuses specifically on international health. The curriculum is equivalent to a standard American medical school course of study but special emphasis is placed on medical issues in developing countries. Several modules are available that focus more deeply on specific regions and issues.

2. Describe this school's curriculum in the pre-clinical and clinical years. Are there any innovations, like Problem-Based Learning?
Year one covers basic sciences: biochemistry, molecular bio, cell bio, micro, physiology etc. Most courses are standard lecture, but nearly every course integrates some problem-based learning and discussion sessions. As classes are smaller than 50 students, even lectures are often participatory, there is plenty of opportunity for discussion and asking questions. Second year is entirely systems-based, and broken into 2-4 week learning units. TA-lead discussion groups and tutoring sessions are common and helpful.

3. Are there opportunities for students to design, conduct, and publish their own research?
All students complete a research project in a third world country during our fourth year. Beginning next year, 1st and 2nd year students will also complete a project or grant proposal as well. For extra motivated students, there are research opportunities within the hospital that are not directly associated with the school..

4. Is there a note-taking service? If so, is it university-run or student-run?
There is nothing official since the program is so small. Some student notes are available online but they are not currently updated.

5. Is there flexibility in the coursework (the number of electives) and the timing of the courses (accelerating, decelerating, and time off) during the pre-clinical and clinical years?
Yes- it is possible to take a year off, or to spread out a preclinical year over two years if you're having trouble keeping up. During first and second year, you are required to take four modules but can schedule those as you'd like to and are free to take more. There is some flexibility for scheduling USMLE step 2 and electives during fourth year, but it is limited due to the required 3rd world rotation.

6. Has this medical school, or any of its clinical departments, been on probation or had its accreditation revoked?
No.

7. How do students from this medical school perform on the National Board Examinations? How does the school assist students who do not pass?
It's hard to compare to American medical schools since our class sizes range from 16-45 students. Our pass rate was 86% when I entered two years ago and it's been rising pretty significantly every year. I'm not sure what support is available for students who don't pass- but no student has failed twice to my knowledge.

Evaluations
8. How are students evaluated academically? How are clinical evaluations performed?
We are on a pass/fail/honors system. Students are also privately notified if they "barely pass" an exam. No number grades are available and exams are not available for review so it's sometimes difficult to get a sense of how you're performing in a given course. You can find out whether you're in the top, middle, or bottom third of the class. I'm not sure how clinical evaluations are performed.

9. Is there a formal mechanism in place for students to evaluate their professors and attending physicians? What changes have been made recently as a result of this feedback?
Yes, there are student-presented evaluations after every class. The faculty is very responsive to our feedback, and several courses have been changed entirely as a result of our comments.

Counseling/Student Support
10. What kind of academic, personal, financial, and career counseling is available to students? Are these services also offered to their spouses and dependents/children?
Support is extremely limited. In my opinion, this is a huge weakness in our school. There is a personal counselor, who many students consult. There is one financial counselor who often seems overwhelmed. Academic and career counseling are effectively unavailable. The professors do have open doors, but there is no official mentor program or career counseling available.

11. Is there a mentor/advisor system? Who are the advisors—faculty members, other students, or both?
There is one faculty adviser for each class, but they're not generally available for personal questions/guidance. There is an academic adviser who helps students with academic difficulties.

12. How diverse is the student body? Are there support services or organizations for ethnic minorities and women?
We have a very international and diverse student body. There are no specific support services for minorities or women.

Facilities
13. Tell me about the library and extracurricular facilities (i.e., housing and athletic/recreational facilities).
There's no official housing, but cheap, nice apartments are easy to find in the community. There is a small medical library with limited hours. The hours are a frustration, but it does have plenty of computers, and all essential textbooks and journals. There is a larger library on campus, again with limited hours. The city has two gyms, a small one associated with the university, and a larger one in the local mall. The university offers multiple clubs, fitness courses, sports teams, and art classes.

14. Are there computer facilities available to students? Are they integrated into the curriculum/learning?
Computers are integrated into the curriculum extensively. There is a 24 hour computer room available, and several other computer labs with limited hours.

15. What type of clinical sites—ambulatory, private preceptors, private hospitals, rural settings—are available or required for clerkships? Does this school allow for students to do rotations at other institutions or internationally?
We work at several major Israeli hospitals, and volunteer opportunities are available on ambulances and at rural and mobile clinics. Fourth year is spent doing rotations all over America and in a third world country.

16. Is a car necessary for clinical rotations? Is parking a problem?
A car is not necessary. Israel is very walking-friendly and vans are provided for away-rotations.

Student Involvement
23. What medical school committees (e.g., curriculum committee) have student representation?
Students are very involved in nearly all curriculum and school decisions.

24. Are students involved in (required or voluntary) community service?
Yes. It is not required, but there are numerous opportunities and most students take advantage of them.

25. How active is the Student Council/Government? Are there other student organizations?
Student government is very active. Our school is teeny- so there aren't a huge amount of other clubs, but there are several social action groups, an emergency medicine club, students for integrative health, a literature and medicine group, etc..

Policies
26. Is there an established protocol for dealing with student exposure to infectious diseases? Is disability insurance provided to cover this exposure?
I'm not sure, but Israel's health care is socialized and our insurance is very comprehensive.

27. Does this school provide, or does the student pay for, vaccinations against Hepatitis B or prophylactic AZT treatment in case of a needle-stick or accident?
Again, I'm not sure, but I think so.

28. Is there a school Honor Code? Is there a grievance process/procedure? Are the students involved?
Yes- we write and sign our own honor code and there is an official procedure for complaints- although some students complain that it is not standardized enough.

Residency
29. May I see a list of residency programs to which this school's recent graduates were accepted?
http://cait.cumc.columbia.edu:88/dept/bgcu-md/documents/AllResidencyPlacements_2002_2007.pdf
 
Status
Not open for further replies.
Top